Mayo Clinic gastroenterology and hepatology board review [4ed.] 9780199827619, 0199827613

1,911 195 8MB

English Pages 459 [474] Year 2011

Report DMCA / Copyright

DOWNLOAD FILE

Polecaj historie

Mayo Clinic gastroenterology and hepatology board review [4ed.]
 9780199827619, 0199827613

Citation preview

Mayo Clinic Gastroenterology and Hepatology Board Review Fourth Edition

Mayo Clinic Scientific Press Mayo Clinic Atlas of Regional Anesthesia and Ultrasound-Guided Nerve Blockade Edited by James R. Hebl, MD, and Robert L. Lennon, DO Mayo Clinic Preventive Medicine and Public Health Board Review Edited by Prathibha Varkey, MBBS, MPH, MHPE Mayo Clinic Internal Medicine Board Review, Ninth Edition Edited by Amit K. Ghosh, MD Mayo Clinic Challenging Images for Pulmonary Board Review Edward C. Rosenow III, MD

Mayo Clinic Gastroenterology and Hepatology Board Review Fourth Edition

Editor-in-Chief

Stephen C. Hauser, MD Consultant, Division of Gastroenterology and Hepatology, Mayo Clinic, Rochester, Minnesota; Associate Professor of Medicine, College of Medicine, Mayo Clinic

MAYO CLINIC SCIENTIFIC PRESS

OXFORD UNIVERSITY PRESS

The triple-shield Mayo logo and the words MAYO, MAYO CLINIC, and MAYO CLINIC SCIENTIFIC PRESS are marks of Mayo Foundation for Medical Education and Research.

1 Oxford University Press, Inc., publishes works that further Oxford University’s objective of excellence in research, scholarship, and education. Oxford New York Auckland Cape Town Dar es Salaam Hong Kong Karachi Kuala Lumpur Madrid Melbourne Mexico City Nairobi New Delhi Shanghai Taipei Toronto With offices in Argentina Austria Brazil Chile Czech Republic France Greece Guatemala Hungary Italy Japan Poland Portugal Singapore South Korea Switzerland Thailand Turkey Ukraine Vietnam

Copyright © 2004, 2006, 2008, 2011 by Mayo Foundation for Medical Education and Research. Published by Oxford University Press, Inc. 198 Madison Avenue, New York, New York 10016 www.oup.com Oxford is a registered trademark of Oxford University Press All rights reserved. No part of this publication may be reproduced, stored in a retrieval system, or transmitted, in any form or by any means, electronic, mechanical, photocopying, recording, or otherwise, without the prior permission of Mayo Foundation for Medical Education and Research. Inquiries should be addressed to Scientific Publications, Plummer 10, Mayo Clinic, 200 First St SW, Rochester, MN 55905 _______________________________________________ Library of Congress Cataloging-in-Publication Data Mayo Clinic gastroenterology and hepatology board review / editor-in-chief, Stephen C. Hauser. – 4th ed. p.; cm. Gastroenterology and hepatology board review Includes bibliographical references and index. ISBN 978-0-19-982761-9 (alk. paper) 1. Gastroenterology–Examinations, questions, etc. 2. Gastrointestinal system–Examinations, questions, etc. 3. Liver–Diseases–Examinations, questions, etc. I. Hauser, Stephen C. II. Hauser, Stephen C. III. Mayo Clinic. IV. Title: Gastroenterology and hepatology board review. [DNLM: 1. Gastrointestinal Diseases–Examination Questions. 2. Liver Diseases—Examination Questions. WI 18.2] RC801.M33 2011 616.3’30076–dc22 2010053146 _______________________________________________

Mayo Foundation does not endorse any particular products or services, and the reference to any products of services in this book is for informational purposes only and should not be taken as an endorsement by the authors or Mayo Foundation. Care has been taken to confirm the accuracy of the information presented and to describe generally accepted practices. However, the authors, editors, and publisher are not responsible for errors or omissions or for any consequences from application of the information in this book and make no warranty, express or implied, with respect to the contents of the publication. This book should not be relied on apart from the advice of a qualified health care provider. The authors, editors, and publisher have exerted efforts to ensure that drug selection and dosage set forth in this text are in accordance with current recommendations and practice at the time of publication. However, in view of ongoing research, changes in government regulations, and the constant flow of information relating to drug therapy and drug reactions, readers are urged to check the package insert for each drug for any change in indications and dosage and for added wordings and precautions. This is particularly important when the recommended agent is a new or infrequently employed drug. Some drugs and medical devices presented in this publication have US Food and Drug Administration (FDA) clearance for limited use in restricted research settings. It is the responsibility of the health care providers to ascertain the FDA status of each drug or device planned for use in their clinical practice. 9 8 7 6 5 4 3 2 1 Printed in China on acid-free paper

To the many persons who have taught, encouraged, and inspired us so that we can provide the best care for our patients and help educate our colleagues to do the same.

This page intentionally left blank

Preface

Gastroenterology and hepatology encompass a vast anatomical assortment of organs that have diverse structure and function and potentially are afflicted by a multiplicity of disease processes. We have designed the Mayo Clinic Gastroenterology and Hepatology Board Review course and the revised fourth edition of this book to assist both physicians-in-training who are preparing for the gastroenterology board examination and the increasing number of gastroenterologists awaiting recertification. Mayo Clinic Gastroenterology and Hepatology Board Review is not intended to replace the many more encyclopedic textbooks of gastroenterology, hepatology, pathology, endoscopy, nutrition, and radiology now available. Nor is this book intended to serve as an “update” to physicians looking for the newest advances in the science and art of gastroenterology and hepatology. Instead, the book provides a core of essential knowledge in gastroenterology, hepatology, and integral related areas of pathology, endoscopy, nutrition, and radiology. Clinical knowledge related to diagnostic and therapeutic approaches to patient management is emphasized. Case-based presentations and short board examination-type, single bestanswer multiple-choice questions with annotated answers are featured. This text also is intended to be used by medical students and residents during their clerkships in internal medicine and gastroenterology and by gastroenterology fellows in training. Physicians in practice should find this book to be a practical review for consolidating their knowledge in gastroenterology and hepatology.

The book is organized by subspecialty topics, including esophageal disorders, gastroduodenal disorders, small-bowel disease and nutrition, colonic disorders, pancreaticobiliary disease, liver disease, and miscellaneous disorders. Numerous color and black-and-white figures support the text. Each subspecialty section concludes with a chapter containing board examination-type, single-best answer multiple-choice questions with annotated answers. (The chapters consisting of questions and answers are not included in the index.) The faculty responsible for the book (at the time it was produced) all are Mayo Clinic gastroenterologists and hepatologists who spend the majority of their time caring for patients but have a commitment to teaching medical students, house officers, fellows, nurses, and physicians. Most of the faculty have particular interests in subspecialty areas of clinical gastroenterology and hepatology, which provides broad expertise. We want to thank the staffs of the Section of Scientific Publications, Media Support Services at Mayo Clinic, and the Mayo School of Continuous Professional Development for their contributions. The support of Mayo Clinic Scientific Press and the publisher, Oxford University Press, also are greatly appreciated. We want to give special thanks to our secretaries and to Greg Gores, MD, for his ongoing enthusiasm and support for our faculty and teaching mission.

This page intentionally left blank

Contents

Section I: Esophagus

1. Gastroesophageal Reflux Disease 3 Joseph A. Murray, MD 2. Barrett’s Esophagus and Esophageal Cancer Ganapathy A. Prasad, MD, MS 3. Normal and Abnormal Esophageal Motility Dawn L. Francis, MD, MHS

Section V: Colon

19 29

Questions and Answers Section II: Stomach

4. Peptic Ulcer Disease 47 Stephen C. Hauser, MD 5. Gastritis 55 Stephen C. Hauser, MD 6. Gastric Neoplasms and Gastroenteropancreatic Neuroendocrine Tumors 63 Mark V. Larson, MD 7. Gastrointestinal Motility Disorders 81 G. Richard Locke III, MD Questions and Answers

Questions and Answers

Section III: Small Bowel and Nutrition

8. Clinical Features of Malabsorptive Disorders, Small-Bowel Diseases, and Bacterial Overgrowth Syndromes 101 Amy S. Oxentenko, MD 9. Nutritional Disorders: Vitamins and Minerals Stephen C. Hauser, MD

Section VI: Liver

115

Questions and Answers Section IV: Miscellaneous Disorders

10. Gastrointestinal Manifestations of Human Immunodeficiency Virus Infection 127 Stephen C. Hauser, MD 11. Nonvariceal Gastrointestinal Tract Bleeding 135 Jeffrey A. Alexander, MD 12. Vascular Disorders of the Gastrointestinal Tract 141 Stephen C. Hauser, MD 13. Gastrointestinal Manifestations of Systemic Disease 149 Stephen C. Hauser, MD Questions and Answers

14. Inflammatory Bowel Disease: Clinical Aspects 167 David H. Bruining, MD, and William J. Tremaine, MD 15. Inflammatory Bowel Disease: Therapy 173 Sunanda V. Kane, MD 16. Inflammatory Bowel Disease: Extraintestinal Manifestations and Cancer 185 Edward V. Loftus Jr, MD 17. Gastrointestinal Infections, Clostridium difficileAssociated Disease, and Diverticular Disease 193 Conor G. Loftus, MD 18. Colorectal Neoplasms 205 Lisa A. Boardman, MD, and Paul J. Limburg, MD, MPH 19. Irritable Bowel Syndrome 213 G. Richard Locke III, MD 20. Constipation and Disorders of Pelvic Floor Function 219 Adil E. Bharucha, MBBS, MD 21. Gastrointestinal Disease and Pregnancy 231 Sunanda V. Kane, MD

22. Approach to the Patient With Abnormal Liver Tests and Acute Liver Failure 261 John J. Poterucha, MD 23. Chronic Viral Hepatitis 269 John J. Poterucha, MD 24. Clinical Approach to Liver Mass Lesions 281 Lewis R. Roberts, MB, ChB, PhD 25. Alcoholic Liver Disease 295 Vijay H. Shah, MD 26. Vascular Diseases of the Liver 305 Patrick S. Kamath, MD 27. Portal Hypertension-Related Bleeding 311 Patrick S. Kamath, MD 28. Ascites, Hepatorenal Syndrome, and Encephalopathy 317 J. Eileen Hay, MB, ChB 29. Metabolic Liver Disease 327 William Sanchez, MD

x

Contents

30. Cholestatic Liver Disease 339 Keith D. Lindor, MD 31. Drug-Induced Liver Injury 343 Jayant A. Talwalkar, MD, MPH, and John J. Poterucha, MD 32. Autoimmune Hepatitis 351 Kymberly D. Watt, MD, and Albert J. Czaja, MD 33. Nonalcoholic Fatty Liver Disease 359 Kymberly D. Watt, MD, and Paul Angulo, MD 34. Liver Disease and Pregnancy 367 J. Eileen Hay, MB, ChB 35. Liver Transplantation 377 J. Eileen Hay, MB, ChB Questions and Answers

Section VII: Pancreas and Biliary Tree

36. Acute Pancreatitis 403 Randall K. Pearson, MD 37. Chronic Pancreatitis 411 Suresh T. Chari, MD 38. Pancreatic Neoplasms 417 Randall K. Pearson, MD 39. Gallstones 425 Ferga C. Gleeson, MB, BCh Questions and Answers

Index

445

Contributors

Jeffrey A. Alexander, MD Consultant, Division of Gastroenterology and Hepatology, Mayo Clinic, Rochester, Minnesota; Assistant Professor of Medicine, College of Medicine, Mayo Clinic

Dawn L. Francis, MD, MHS Consultant, Division of Gastroenterology and Hepatology, Mayo Clinic, Rochester, Minnesota; Assistant Professor of Medicine, College of Medicine, Mayo Clinic

Paul Angulo, MD Formerly, Consultant, Division of Gastroenterology and Hepatology, Mayo Clinic, Rochester, Minnesota; Formerly, Associate Professor of Medicine, College of Medicine, Mayo Clinic; Presently, Professor of Medicine and Section Chief of Hepatology, University of Kentucky Medical Center, Lexington, Kentucky

Ferga C. Gleeson, MB, BCh Senior Associate Consultant, Division of Gastroenterology and Hepatology, Mayo Clinic, Rochester, Minnesota; Assistant Professor of Medicine, College of Medicine, Mayo Clinic

Adil E. Bharucha, MBBS, MD Consultant, Division of Gastroenterology and Hepatology, Mayo Clinic, Rochester, Minnesota; Professor of Medicine, College of Medicine, Mayo Clinic Lisa A. Boardman, MD Consultant, Division of Gastroenterology and Hepatology, Mayo Clinic, Rochester, Minnesota; Associate Professor of Medicine, College of Medicine, Mayo Clinic David H. Bruining, MD Senior Associate Consultant, Division of Gastroenterology and Hepatology, Mayo Clinic, Rochester, Minnesota; Assistant Professor of Medicine, College of Medicine, Mayo Clinic Suresh T. Chari, MD Consultant, Division of Gastroenterology and Hepatology, Mayo Clinic, Rochester, Minnesota; Professor of Medicine, College of Medicine, Mayo Clinic Albert J. Czaja, MD Emeritus Consultant, Division of Gastroenterology and Hepatology, Mayo Clinic, Rochester, Minnesota; Emeritus Professor of Medicine, College of Medicine, Mayo Clinic

Stephen C. Hauser, MD Consultant, Division of Gastroenterology and Hepatology, Mayo Clinic, Rochester, Minnesota; Associate Professor of Medicine, College of Medicine, Mayo Clinic J. Eileen Hay, MB, ChB Consultant, Division of Gastroenterology and Hepatology, Mayo Clinic, Rochester, Minnesota; Professor of Medicine, College of Medicine, Mayo Clinic Patrick S. Kamath, MD Consultant, Division of Gastroenterology and Hepatology, Mayo Clinic, Rochester, Minnesota; Professor of Medicine, College of Medicine, Mayo Clinic Sunanda V. Kane, MD Consultant, Division of Gastroenterology and Hepatology, Mayo Clinic, Rochester, Minnesota; Professor of Medicine, College of Medicine, Mayo Clinic Mark V. Larson, MD Consultant, Division of Gastroenterology and Hepatology, Mayo Clinic, Rochester, Minnesota; Assistant Professor of Medicine, College of Medicine, Mayo Clinic

xii

Contributors

Paul J. Limburg, MD, MPH Consultant, Division of Gastroenterology and Hepatology, Mayo Clinic, Rochester, Minnesota; Professor of Medicine, College of Medicine, Mayo Clinic

John J. Poterucha, MD Consultant, Division of Gastroenterology and Hepatology, Mayo Clinic, Rochester, Minnesota; Professor of Medicine, College of Medicine, Mayo Clinic

Keith D. Lindor, MD Consultant, Division of Gastroenterology and Hepatology, Mayo Clinic, Rochester, Minnesota; Professor of Medicine, College of Medicine, Mayo Clinic

Ganapathy A. Prasad, MD, MS Consultant, Division of Gastroenterology and Hepatology, Mayo Clinic, Rochester, Minnesota; Assistant Professor of Medicine, College of Medicine, Mayo Clinic

G. Richard Locke III, MD Consultant, Division of Gastroenterology and Hepatology, Mayo Clinic, Rochester, Minnesota; Professor of Medicine, College of Medicine, Mayo Clinic

Lewis R. Roberts, MB, ChB, PhD Consultant, Division of Gastroenterology and Hepatology, Mayo Clinic, Rochester, Minnesota; Professor of Medicine, College of Medicine, Mayo Clinic

Conor G. Loftus, MD Consultant, Division of Gastroenterology and Hepatology, Mayo Clinic, Rochester, Minnesota; Assistant Professor of Medicine, College of Medicine, Mayo Clinic

William Sanchez, MD Consultant, Division of Gastroenterology and Hepatology, Mayo Clinic, Rochester, Minnesota; Assistant Professor of Medicine, College of Medicine, Mayo Clinic

Edward V. Loftus Jr, MD Consultant, Division of Gastroenterology and Hepatology, Mayo Clinic, Rochester, Minnesota; Professor of Medicine, College of Medicine, Mayo Clinic

Vijay H. Shah, MD Consultant, Division of Gastroenterology and Hepatology, Mayo Clinic, Rochester, Minnesota; Professor of Medicine and of Physiology, College of Medicine, Mayo Clinic

Joseph A. Murray, MD Consultant, Division of Gastroenterology and Hepatology, Mayo Clinic, Rochester, Minnesota; Professor of Medicine, College of Medicine, Mayo Clinic

Jayant A. Talwalkar, MD, MPH Consultant, Division of Gastroenterology and Hepatology, Mayo Clinic, Rochester, Minnesota; Associate Professor of Medicine, College of Medicine, Mayo Clinic

Amy S. Oxentenko, MD Consultant, Division of Gastroenterology and Hepatology, Mayo Clinic, Rochester, Minnesota; Assistant Professor of Medicine, College of Medicine, Mayo Clinic

William J. Tremaine, MD Consultant, Division of Gastroenterology and Hepatology, Mayo Clinic, Rochester, Minnesota; Professor of Medicine, College of Medicine, Mayo Clinic

Randall K. Pearson, MD Consultant, Division of Gastroenterology and Hepatology, Mayo Clinic, Rochester, Minnesota; Associate Professor of Medicine, College of Medicine, Mayo Clinic

Kymberly D. Watt, MD Consultant, Division of Gastroenterology and Hepatology, Mayo Clinic, Rochester, Minnesota; Associate Professor of Medicine, College of Medicine, Mayo Clinic

Mayo Clinic Gastroenterology and Hepatology Board Review Fourth Edition

This page intentionally left blank

Section I Esophagus

This page intentionally left blank

1 Gastroesophageal Reflux Diseasea Joseph A. Murray, MD

Gastroesophageal reflux is the reflux of gastric contents other than air into or through the esophagus. Gastroesophageal reflux disease (GERD) refers to reflux that produces frequent symptoms or results in damage to the esophageal mucosa or contiguous organs of the upper aerodigestive system and occasionally the lower respiratory tract.

Etiologya

Gastroesophageal reflux results from several factors that lead to symptoms or injury of the mucosa of the esophagus or the airway by reflux of corrosive material from the stomach (Table 1.1). These factors include a weak or defective sphincter, transient lower esophageal sphincter relaxations (TLESRs), hiatal hernia, poor acid clearance from the esophagus, diminished salivary flow, reduced mucosal resistance to injury, increased acid production, delayed gastric emptying of solids, and obstructive sleep apnea (Fig. 1.1). The relative contribution of these varies from patient to patient.

Table 1.1 Etiologic Factors of Gastroesophageal Reflux Disease

Motility disorders Transient lower esophageal relaxationsa Weak lower esophageal sphinctera Weak esophageal peristalsis Scleroderma and CREST Delayed gastric emptying Damaging factors Increased gastric acid production Bile and pancreatic juice Resistance factors Reduced saliva and HCO3 production Diminished mucosal blood flow Growth factors, protective mucus Others Hiatal herniaa Obstructive sleep apnea Abbreviation: CREST, calcinosis cutis, Raynaud’s phenomenon, esophageal dysfunction, sclerodactyly, and telangiectasia. a

Major/common factors.

Factors Contributing to Gastroesophageal Reflux Disease Barrier Function of the Lower Esophageal Sphincter

a Portions of this chapter were adapted from Szarka LA, DeVault KR, Murray JA. Diagnosing gastroesophageal reflux disease. Mayo Clin Proc. 2001 Jan;76(1):97-101. Used with permission. Abbreviations: CREST, calcinosis cutis, Raynaud’s phenomenon, esophageal dysfunction, sclerodactyly, and telangiectasia; GERD, gastroesophageal reflux disease; H2, histamine2; TLESR, transient lower esophageal sphincter relaxation

The lower esophageal sphincter and its attached structures form a barrier to reflux of material across the esophagogastric junction and is the central protection against pathologic reflux of gastric contents into the esophagus. This barrier has several components, including the smooth muscle lower esophageal sphincter, the gastric sling fibers, and the striated muscle crural diaphragm. The lower esophageal sphincter maintains tone at rest and relaxes with swallowing and gastric distention as a venting reflex. This latter relaxation has been termed transient 3

4

Section I Esophagus

Defective esophageal clearance Lower esophageal sphincter ‘dysfunction’

The presence of hiatal hernia has an important role in defective barrier function, both by removing the augmentation that the crural diaphragm provides the lower esophageal sphincter and lowering the threshold for TLESRs to occur. Acid Clearance

Hiatal hernia

Delayed gastric emptying Increased intra-abdominal pressure

The clearance of acid from the esophagus is a combination of mechanical volume clearance (gravity and peristalsis) and chemical neutralization of the lumen contents (saliva and mucosal buffering). This may be delayed in patients with reflux because of either impaired esophageal peristalsis or reduced buffering effects of swallowed saliva. The defective peristalsis can be a primary idiopathic motor disorder or, occasionally, it can result from a connective tissue disorder such as CREST (calcinosis cutis, Raynaud’s phenomenon, esophageal dysfunction, sclerodactyly, and telangiectasia) syndrome or scleroderma. Many drugs and Sjögren’s syndrome can decrease salivary flow. Normally, salivary flow is decreased at night; thus, if reflux occurs during the night when the person is supine, acid will not be cleared by either gravity or saliva. This is why episodes of reflux at night are long-lasting and have a greater chance of causing severe injury to the mucosa.

Figure 1.1 Causes of increased exposure of the esophagus to

gastric refluxate. (Adapted from AstraZeneca Pharmaceuticals LP [Internet]. Wilmington [DE]. Available from: http://www. astrazeneca.com. Used with permission.)

lower esophageal sphincter relaxation (TLESR). In persons with mild reflux disease, acid liquid contents instead of air alone are vented, resulting in many episodes of acid reflux. In patients with severe reflux, the resting pressure of the lower esophageal sphincter usually is diminished and easily overcome.

A

B

Intrinsic Mucosal Factors

The mucosa of the esophagus has intrinsic factors that protect the esophageal lining against acid damage. These include the stratified squamous mucosa, intercellular tight junctions, growth factors, buffering blood flow, and production of mucin, bicarbonate, and epidermal growth factors. When these factors are overcome, GERD causes reflux esophagitis (Fig. 1.2 and 1.3).

C

Acid Pepsin Bicarbonate

Nerve ending

Tight cell junction

Widened cell junction

Figure 1.2 Mechanism of action of refluxate in gastroesophageal reflux disease. The sequence of events hypothesized to lead to symptoms and tissue damage in gastroesophageal reflux disease is as follows: A and B, Acid-peptic attack weakens cell junctions and, C, widens the cell gaps, thus allowing acid penetration. Exposure to gastric acid and pepsin can cause microscopic damage to the esophageal mucosa, which may not be visible endoscopically but still result in heartburn. (Adapted from AstraZeneca Pharmaceuticals LP [Internet]. Wilmington [DE]. Available from: http://www.astrazeneca.com. Used with permission.)

1 Gastroesophageal Reflux Disease

A

5

B

Nerve ending

Acid Pepsin Bicarbonate

Figure 1.3 Mechanism of action of refluxate in gastroesophageal reflux disease. A, Penetration of acid and pepsin into the mucosa allows contact of acid with epithelial nerve endings (which may result in heartburn). B, Additional influx of acid and pepsin into the mucosa triggers a cascade of events ultimately leading to cell rupture and mucosal inflammation. (Adapted from AstraZeneca Pharmaceuticals LP [Internet]. Wilmington [DE]. Available from: http://www.astrazeneca.com. Used with permission.)

Gastric Factors

Delayed gastric emptying or increased gastric production of acid is less frequently part of GERD. Reflux esophagitis is rarely a manifestation of Zollinger-Ellison syndrome. The availability of corrosive gastric contents in the cardia of the stomach is necessary for reflux to occur during TLESR or when a defective lower esophageal sphincter is overcome during recumbency or abdominal straining. The cardia is often submerged under liquid gastric contents in the recumbent, especially in the right lateral decubitus, position. It has been suggested recently that what differentiates patients with GERD from normal subjects is not the number of actual reflux events but the reflux of acidic gastric contents instead of the release of air alone. The timing of reflux is also important. Because gastric acid is buffered by food during the first hour after eating, normal physiologic reflux that may occur during maximal gastric distention is not as harmful as the reflux that occurs later after the stomach pH has again decreased. Any obstruction of the outflow from the stomach increases the propensity to reflux, although this is often associated with nausea and vomiting. Pure bile reflux may occur in patients who have had gastric surgery. More common is pathologic reflux associated with a restrictive bariatric procedure such as vertical banded gastroplasty. If too much acid-producing mucosa is present above the restriction, pathologic reflux may occur. Helicobacter pylori and Gastroesophageal Reflux Disease

Whether chronic Helicobacter pylori infection protects against GERD is a matter of controversy. Duodenal ulcers and distal gastric cancer (both caused by H pylori infection) are becoming

rare in the developed world, and adenocarcinoma of the proximal stomach and esophagus is becoming more common as the carriage rates of H pylori decrease. Patients with GERD symptoms may be less likely to carry H pylori than the population without GERD symptoms. Reports that symptoms of GERD developed after the eradication of H pylori have led to a reexamination of those treatment trials of duodenal ulcers, which included H pylori eradication, for the new development of GERD symptoms. The evidence is conflicting whether the symptoms of GERD are more common in those in whom H pylori eradication has been successful or in those with persistent infection. In some persons, H pylori infection may cause chronic atrophic gastritis that affects the corpus of the stomach, resulting in diminished acid secretion. It is this relative hypochlorhydria that protects against GERD. Indeed, it has been suggested that acid suppression heals reflux esophagitis faster in patients with H pylori infection (Fig. 1.4). Other explanations for the apparent occurrence of GERD after the eradication of H pylori may include unrecognized GERD injury or symptoms present before eradication, rebound acid secretion after cessation of potent acid suppression, or other unrelated factors. Connective Tissue Disease

Scleroderma, CREST syndrome, or mixed connective tissue diseases are rare causes of reflux, but these should be considered in young women who have Raynaud’s phenomenon or subtle cutaneous features of scleroderma in the hands or face. Occasionally, GERD may be the first manifestation of these disorders. Esophageal manometry usually demonstrates a low-pressure lower esophageal sphincter and decreased amplitude of contractions in the esophagus (Fig. 1.5).

Section I Esophagus

% Patients healed at 4 weeks

6

P=0.0005

100

87 76

80 60 40 20 0

H pylori +

H pylori −

Figure 1.4 The efficacy of proton pump inhibitor therapy may be

greater in patients with gastroesophageal reflux disease who are positive for Helicobacter pylori (H pylori +) than in those negative for H pylori (H pylori –).

Figure 1.6 Laryngeal stenosis. (Courtesy of Dana M. Thompson,

Mechanism of Extraesophageal Symptoms

Epidemiology of Gastroesophageal Reflux Disease

Upper esophagusnormal 23 28 34

0 50

35

0 50

Absent peristalsis

38

34

24

0 50

36

0 50

39

0 50

29

0 50

GERD can be defined as chronic symptoms of heartburn, acid regurgitation or dysfunction, or injury to the esophagus or other organs because of abnormal reflux of gastric contents. Symptoms suggestive of GERD are common: 40% of the adult population in the United States report heartburn monthly and 18% report it weekly (Fig. 1.7). GERD becomes more common with increasing age (Fig. 1.8). Previously, GERD

19

The mechanism for extraesophageal manifestations of GERD, such as wheeze or cough, may not always be direct aspiration or damage of mucosa in the respiratory tract but a vagally mediated reflex triggered by acidification of the distal esophageal mucosa. Subglottic stenosis and granuloma of the vocal cords are very serious consequences of reflux caused by direct contact injury of the delicate mucosa of the airway, resulting in stridor, cough, or dysphonia (Fig. 1.6).

MD, Department of Otorhinolaryngology, Mayo Clinic, Rochester, Minnesota.)

LES-hypotonic 0

30 sec

21:34.9

Figure 1.5 Esophageal manometric tracing illustrating complete absence of peristalsis or absence of lower esophageal sphincter (LES) pressure

consistent with involvement of the esophagus by scleroderma.

25 20

14

15 10

18

17

10

10

11

10

8 4

5 0 Italy n=999

Japan n=500

Nordic Canada USA n=1010 n=1036 n=1020

15 10 5 0 70

Age, years

Figure 1.7 Prevalence of gastroesophageal reflux disease worldwide.

Note that the prevalence varies markedly from country to country, largely because of differences in physicians’ awareness and understanding of the condition.

and its complications were rare in China, Japan, and other Asian countries, but this is changing rapidly with the adoption of a Western diet. A protective role of H pylori–induced hypochlorhydria has been suggested as a protective influence in countries with high carriage rates of infection. However, actual organ damage is observed less frequently, and fewer than 50% of patients who present for medical attention for reflux symptoms have esophagitis. Of patients who have endoscopy for GERD, 10% have benign strictures and only 3% to 4% have Barrett’s esophagus; an extremely small number

Psychiatric patients

Figure 1.8 Incidence of gastrointestinal reflux disease increases with

age. Note that the incidence increases markedly after age 40 years. (Adapted from Brunnen PL, Karmody AM, Needham CD. Severe peptic oesophagitis. Gut. 1969 Oct;10[10]:831–7. Used with permission.)

have adenocarcinoma. Complications of GERD may be more common in males and whites and with advancing age. Whether reflux is becoming more common is not clear, but it certainly is diagnosed more frequently than in the past. Also, because of direct-to-consumer advertising and public education campaigns, the public is more aware of GERD. For patients with GERD, the quality of life may be impaired even more than for those with congestive heart failure or angina pectoris (Fig. 1.9). Treatment of GERD has important

67

Erosive esophagitis, untreated

84

Duodenal ulcer, untreated

85

Angina pectoris

87

Heart failure (mild)

94

Normal female

101

Normal male

103

Hypertension, untreated 60

7

20

22

Heartburn Regurgitation

Annual incidence (cases per 100,000)

3-Month prevalence, %

1 Gastroesophageal Reflux Disease

105 70

80

90

100

110

Psychological General Well-being Index (PGWBI) score Figure 1.9 Gastroesophageal reflux disease has a greater effect on quality of life than other common diseases. Quality of life, assessed by the

PGWBI, was compared between patients with untreated gastroesophageal reflux disease and those with other disorders. For example, the mean PGWBI score of patients with untreated erosive esophagitis is similar to that of patients with untreated duodenal ulcer and lower (ie, worse) than that of patients with angina pectoris or mild heart failure. Normal scores are 101 for women and 103 for men, but they vary slightly from country to country. (Data from Dimenas E. Methodological aspects of evaluation of Quality of Life in upper gastrointestinal diseases. Scand J Gastroenterol Suppl. 1993;199:18–21.)

8

Section I Esophagus

health economic effects because, currently, proton pump inhibitors are among the most commonly prescribed and most expensive drugs.

Presentation

The classic symptoms of GERD, that is, heartburn and acid regurgitation, are common in the general population and usually are readily recognized. GERD may be manifested in a wide array of esophageal and extraesophageal symptoms. GERD may contribute to many clinical syndromes, either as a common factor or a rare culprit (Table 1.2).

Symptoms Esophageal Symptoms

The cardinal symptoms of GERD are heartburn (defined as retrosternal burning ascending toward the neck) and acid regurgitation (the unpleasant return of sour or bitter gastric contents to the pharynx). This is to be differentiated from the nonacid (bland) regurgitation of retained esophageal contents in an obstructed esophagus, as occurs in achalasia or the almost volitional regurgitation of recently swallowed food which is remasticated and again swallowed that typifies rumination. Patient symptoms of “GERD,” “reflux,” and “heartburn” should be differentiated from the burning epigastric sensation of dyspepsia. Patients may report relief of symptoms with antacids or milk. The symptoms of heartburn and especially acid regurgitation are specific for GERD. Their presence with sufficient frequency and severity alone usually justifies medical therapy.

Objective confirmation is required before surgery or endoscopic treatment is recommended. Although regurgitation of acid is a specific symptom highly suggestive of GERD, heartburn may have many different meanings for patients, and, indeed, patients may use different and imprecise terms to describe their symptoms, such as “indigestion,” “stomach upset,” and “sour stomach.” Less common symptoms suggestive of but not diagnostic of GERD include water brash (hypersalivation associated with an episode of esophageal acid exposure), dysphagia (difficulty swallowing), odynophagia (painful swallowing), and chest discomfort not identified as heartburn. Reflux is more common after eating. Although reflux symptoms can occur at any time, they tend to aggregate in the period 1 to 3 hours after eating, when acid production overcomes the buffering effects of food (Fig. 1.10). It has been reported that a layer of acid may remain unbuffered on the surface of the gastric meal contents. Reflux may occur also at night or when a person with a weak lower esophageal sphincter is supine or, especially, in the right lateral decubitus position. Esophageal Chest Pain

GERD is the most common esophageal cause of noncardiac chest pain. The pain may be referred to any point on the anterior or posterior chest, with radiation to the neck, arm, or back. It may be indistinguishable from cardiac pain. Because of the potential fatal significance of cardiac pain, it is imperative that cardiac investigation precede esophageal investigation. Frequently, patients who have both cardiac and esophageal diseases cannot distinguish between reflux-associated pain and real angina. GERD may decrease the threshold for coronary ischemia, further confusing the clinical picture. This emphasizes the importance of first investigating the heart and, when appropriate, other vital structures. Extraesophageal Symptoms

Table 1.2 Symptoms of Gastroesophageal Reflux Disease

Esophageal symptoms Heartburn Acid regurgitation Odynophagia Dysphagia Angina-like chest pain Water brash (hypersalivation) Airway symptoms Cough Wheezing Hoarseness Throat clearing Globus Tracheal stenosis Aspiration pneumonia Pulmonary fibrosis Apnea in infants

GERD may contribute to symptoms originating in other areas of the upper aerodigestive system. These symptoms, which can occur without the classic symptoms of heartburn and acid regurgitation, include cough, wheeze, hoarseness, sore throat, repetitive throat clearing, postnasal drip, neck or throat pain, globus, apnea, and otalgia. They are not specific for GERD. Indeed, GERD is only one of many causes of most of these symptoms. Like GERD, cough and wheezing are very common and likely to coexist by chance alone. Whether these symptoms are due to GERD needs to be confirmed by investigation or by the response to an empiric trial of potent acid-blocking therapy. Ideally, the demonstration of a pathologic degree of GERD and a response of the atypical symptoms to an adequate antireflux regimen are needed to conclude that GERD is the cause. GERD may produce extraesophageal symptoms in one of two ways. The first is by direct irritation or inflammation of the delicate mucosa of the larynx, trachea, or bronchi. The second is by reflex-mediated changes in function. Both mechanisms may operate in some patients.

1 Gastroesophageal Reflux Disease

Breakfast

Lunch

9

Dinner

Number of episodes of reflux symptoms/hour (mean x 102)

45

n=105 30

15

am

am 6

id 12

m

3

ni gh t

pm 9

pm 6

pm 3

12

no on

am 9

6

am

0

Figure 1.10 Distribution of symptoms of gastroesophageal reflux disease over 24 hours in 105 patients who took their major meals at the

same time of day. Note that food intake was associated with a marked increase in the number of symptom episodes and relatively few episodes occurred during the night. (Adapted from Johnsson L, Adlouni W, Johnsson F, Joelssson B. Timing of reflux symptoms and esophageal acid exposure. Gullet. 1992;2:58–62. Used with permission.)

Establishing a Diagnosis Therapeutic Trial

Several studies have investigated the usefulness of empirical trials of acid-suppressive therapy with proton pump inhibitors (Table 1.3). Typical Symptoms of Gastroesophageal Reflux Disease

Patients who present with typical symptoms without alarm symptoms should be given acid-suppressive therapy. Complete resolution of the symptoms with treatment and relapse when Table 1.3 Empirical Trials of Acid-Suppressive Therapy With Proton Pump Inhibitors for Diagnosis Symptom

Treatment

Sensitivity,a %

Heartburn and Omeprazole twice 80 regurgitation daily for 7 days Noncardiac Omeprazole twice 75 chest pain daily for 14 days Extraesophageal Proton pump inhibitor twice daily for 3 months a

For the confirmation of gastroesophageal reflux disease.

Specificity, %

56

85

treatment is discontinued confirm the diagnosis and suggest the need for a long-term management strategy. However, even in these patients, the specificity of a response to potent acid suppression is not specific for GERD because other acid peptic disorders respond to acid-suppressive therapy. If symptomatic improvement is limited, either an increase in dose or additional diagnostic testing is needed. If there is little or no symptomatic improvement with acid-suppressive therapy, further investigation is indicated. Atypical Symptoms of Gastroesophageal Reflux Disease

GERD may cause or contribute to many different clinical syndromes. The more common or dangerous causes of these syndromes should be evaluated first. For example, patients with chronic cough or hoarseness should be evaluated for asthma or laryngeal neoplasm, respectively. If GERD is a possible cause, a therapeutic trial of acid suppression may be attempted. For esophageal symptoms such as chest pain, a 2-week trial of therapy usually is sufficient. For extraesophageal symptoms, a more prolonged therapeutic trial (2-3 months) may be necessary. The acid-suppression test uses a potent regimen of acid suppression, for example, proton pump inhibitors (omeprazole, 40 mg in the morning and 20 mg in the evening). If the symptoms resolve, the patient should receive long-term treatment, with an attempt at dose reduction or cessation. For atypical symptoms, it is important to consider that they may have had alternative causes that resolved spontaneously. However, if there are reversible factors that are altered and if GERD is the major cause, the symptoms are likely to recur when therapy

10

Section I Esophagus

is discontinued. If the symptoms do not resolve completely, further evaluation with upper endoscopy or 24-hour ambulatory esophageal pH monitoring with symptom-reflux correlation (or both) is indicated. Ideally, the test should be conducted when the patient is not taking a proton pump inhibitor. If GERD is confirmed, long-term acid-suppressive therapy is indicated. If symptoms persist, ambulatory esophageal pH monitoring may be repeated to document that the esophagus is no longer exposed to acid.

Diagnostic Tests for Gastroesophageal Reflux Disease

Diagnostic tests are unnecessary for most persons with GERD. Investigations should be conducted in patients who have alarm symptoms, equivocal results on a treatment trial, or atypical symptoms of sufficient importance to warrant confirmation of GERD and in those undergoing surgical or endoscopic therapy for GERD. For most patients, the endoscopic demonstration of esophagitis is sufficient proof of GERD and further investigation is unnecessary. However, more than 50% of patients with symptoms typical of GERD have normal endoscopic findings, and additional tests are required to identify increased esophageal exposure to acid. This is done either directly with ambulatory pH monitoring or indirectly by showing the reflux of a detectable material such as barium or a radiolabeled compound during a provocative maneuver or by reproducing symptoms through the instillation of acid (Table 1.4). Endoscopic Examination

Endoscopic examination allows direct visualization of the esophageal mucosa. In reflux esophagitis, the characteristic

Table 1.4 Uses of Diagnostic Tests for Gastroesophageal Reflux Disease

Endoscopy Differentiate from other causes of reflux esophagitis Biopsy Barrett’s esophagus, adenocarcinoma Dilate strictures Endoscopic therapy (?) Contrast radiography Hiatal hernia Identify strictures Reproduce reflux of barium (?) Ambulatory 24-hour pH studies Quantify acid reflux in the absence of esophagitis Determine temporal correlation between gastroesophageal reflux and atypical symptoms Bernstein test Provoke symptoms with acid Gastroesophageal scintigraphy Quantify gastroesophageal reflux Identify aspiration

finding is linear erosions in the distal esophagus. These usually start at the esophagogastric junction and extend for various distances. The degree of severity varies. By their appearance alone, these erosions usually are readily differentiated from rarer infectious, allergic (eosinophilic), or corrosive causes of inflammation. If the diagnosis is in question, biopsy specimens should be obtained, not primarily to confirm reflux but to identify alternative pathologic conditions. Several grading schemes, generally based on the extent of involvement, have been used. The Los Angeles classification system is the one used most commonly worldwide (Fig. 1.11). Erythema and increased vascularity are nonspecific features, and a break in the mucosa is required to make the diagnosis of reflux esophagitis. Careful scrutiny of the esophagogastric junction with adequate air insufflation is needed to examine the mucosa in its entirety. Endoscopy identifies the esophageal complications of GERD, including esophageal ulceration and stricture, Barrett’s esophagus, and esophageal adenocarcinoma. Alarm symptoms that suggest these complications include long duration (>10 years) of typical symptoms, dysphagia, hematemesis or melena, and weight loss. The presence of these symptoms is a strong indication for diagnostic testing, especially endoscopy. Male sex, middle age, and nocturnal heartburn may be associated with a higher risk of esophagitis and its complications. Barium Upper Gastrointestinal Tract Series

Although the barium contrast study is a readily available test, it is of limited usefulness in the evaluation of patients with GERD. Its major usefulness in GERD is in identifying strictures and large hiatal hernias. It is insensitive for detecting erosions or superficial mucosal changes. The ability to reflux barium while at rest or in response to a provocative maneuver or postural change is not a sensitive test for GERD because most patients have a normal-pressure lower esophageal sphincter. In patients with extraesophageal symptoms, reflux of barium to or above the level of the aortic arch suggests the possibility of proximal reflux. The contrast study has limited value in detecting mucosal changes other than the most pronounced inflammation, which requires a double contrast study. The sensitivity for GERD is only 20%. When provocative maneuvers are added, the sensitivity increases but at great cost to specificity. A barium contrast study may be useful in delineating postoperative anatomical relationships and the intactness of an antireflux repair. Its use is discouraged in the evaluation of uncomplicated GERD. Prolonged Ambulatory Esophageal pH Monitoring Studies

Ambulatory pH monitoring of the esophageal lumen, a wellestablished test, was introduced in the early 1970s. It provides objective evidence of the degree of GERD and its timing. For most patients with symptoms of GERD and for whom the diagnosis is not in doubt, this test is not needed. The indications for ambulatory esophageal pH monitoring are listed in Table 1.5.

LA Grade B

11

LA Grade D

LA Grade C

LA Grade A

1 Gastroesophageal Reflux Disease

Figure 1.11 Erosive esophagitis. Summary of Los Angeles (LA) classification. Grade A, one or more mucosal breaks not more than 5 mm in

maximal length. Grade B, one or more mucosal breaks more than 5 mm in maximal length, but not continuous between the tops of two mucosal folds. Grade C, mucosal breaks that are continuous between the tops of two or more folds but involve less than 75% of the esophageal circumference. Grade D, mucosal breaks that involve at least 75% of the esophageal circumference. (Adapted from AstraZeneca Pharmaceuticals LP [Internet]. Wilmington [DE]. Available from: http://www.astrazeneca.com. Used with permission.)

Table 1.5 Indications for Ambulatory Esophageal pH Monitoring

Atypical symptoms: respiratory, ear, nose, and throat Frequent atypical chest pain Refractory symptoms in well-established GERDa Preoperative confirmation of GERD Abbreviation: GERD, gastroesophageal reflux disease. a

Done on acid blockade.

The test is performed with a probe that has a pH sensor at its tip. The tip is placed 5 cm above the proximal border of the lower esophageal sphincter. Accurate location of this sphincter is critical because normal values for acid exposure apply only if the distance between the pH probe and the sphincter is 5 cm. The position of the lower esophageal sphincter usually is determined manometrically with a standard stationary esophageal manometry study or with a combined single waterperfused pressure transducer with a pH probe that can locate accurately the proximal border of the sphincter and requires only a single intubation. Other methods such as endoscopic measurement and pH step-up on withdrawal are not sufficiently accurate for the placement of the nasoesophageal probe. The pH is recorded by a small portable recorder. A newer method uses a tubeless pH capsule that is pinned to the distal esophagus 6 cm above the endoscopically determined squamocolumnar junction. It transmits the pH measurements to a recorder worn on the chest. Its advantages are that it can record for prolonged periods and patients may eat more normally, without the discomfort of the nasal tube. The patient should maintain his or her usual diet, activity, and habits during the study to allow the assessment of findings relative to the patient’s normal lifestyle. The recorders have a patientactivated event button (or buttons) to indicate meals, changes in posture, and symptom events. The duration of the recording

must be long enough to reflect all periods of the day, especially postprandial periods. Ideally, 20 hours or more of analyzable recordings are made. The recordings are analyzed initially by visual inspection of the graphs and then by computer-assisted quantitative analysis of the number and duration of reflux episodes and the relation to any symptoms the patient may have recorded (Fig. 1.12). Reflux of acid is defined as a sudden decrease in intraesophageal pH 8 mm Hg)

Chest pain, dysphagia

Medications Bougie dilation Botulinum toxin

Chest pain, dysphagia

Medications Botulinum toxin

Reflux, regurgitation, dysphagia

Treat underlying disorder Acid-suppressing medications Fundoplication (typically partial)

Reflux, regurgitation

Acid-suppressing medications Fundoplication (typically partial)

Achalasia

Nonspecific esophageal dysmotility Discoordinated motility Diffuse esophageal spasm

Hypercontractile esophagus Nutcracker esophagus

Hypertensive LES

Hypocontractile esophagus Ineffective esophageal motility

Hypotensive LES

Low-amplitude peristalsis (>50% wet swallows) Low mean distal peristaltic amplitude (6 seconds) (Fig. 3.5). Barium esophagram findings are variable in diffuse esophageal spasm. Classically, severe tertiary contractions can produce a “corkscrew” esophagus (Fig. 3.6). However, because the results of radiographic studies may be normal in diffuse esophageal spasm, a normal study does not rule out the diagnosis. The diagnosis is made on the basis of clinical symptoms and manometric findings. Nutcracker Esophagus

Hypercontractile Esophageal Disorders Diffuse Esophageal Spasm

Diffuse esophageal spasm is likely a rare disease. The true prevalence is uncertain because it can be difficult to diagnose. The typical symptom complex for this condition is intermittent chest pain (it may radiate to the back or throat) associated with dysphagia. Nearly 10% of patients with noncardiac chest pain or unexplained dysphagia have diffuse esophageal spasm. The pathophysiology of the condition is poorly understood. Over the past several decades, it has become clear that diffuse esophageal spasm is a misnomer, because the spastic component affects only the smooth muscle in the distal two-thirds of the esophagus. A small study of patients with diffuse esophageal spasm has suggested there may be dysfunction in the endogenous synthesis or degradation of nitric oxide. This finding

The term nutcracker esophagus is used to describe the manometric finding of high-pressure peristaltic contractions in the distal esophagus, with an average amplitude of pressure higher than 220 mm Hg on 10 or more liquid swallows, with otherwise normal peristalsis. Nutcracker esophagus was first described more than 3 decades ago, and since that time, there has been considerable controversy about the association of this manometric finding with symptoms. In a study that examined the manometric findings from patients with unexplained chest pain, nutcracker esophagus was the most common abnormal finding, occurring in 12% of patients. Many patients with the manometric finding of nutcracker esophagus do not have chest pain. In an unpublished study at Mayo Clinic, 15% of asymptomatic patients who received esophageal manometry before having 24-hour impedance pH testing met the manometric criteria for nutcracker esophagus.

mm Hg

Simultaneous, multipeaked contractions

Intermittent peristalsis

Normally relaxing LES

Figure 3.5 Esophageal manometry in patient with diffuse esophageal spasm. Note the simultaneous, multipeaked contractions of prolonged

duration with intermittent peristalsis. LES indicates lower esophageal sphincter.

36

Section I Esophagus

Figure 3.6 Barium esophagram of patient with diffuse esophageal

spasm showing “corkscrew” esophagus.

A study with 24-hour ambulatory esophageal manometry found that patients who had nutcracker esophagus or diffuse esophageal spasm according to stationary manometry frequently switch patterns over 24 hours, suggesting that nutcracker esophagus may be a marker for esophageal spasm. With the use of high-frequency intraluminal ultrasonography to examine patients who have nutcracker esophagus, asynchrony was found in the contractions of the circular and longitudinal muscle layers of the esophagus, and this was reversed with atropine, suggesting that the manometric abnormality may be due to a hypercholinergic state. Hypertensive Lower Esophageal Sphincter

The manometric finding of a hypertensive LES (>45 mm Hg, LES resting pressure) without other abnormalities denotes a primary esophageal motility disorder of the hypercontractile subtype. Some patients with a hypertensive LES also have a poorly relaxing LES. As many as 50% of patients with a hypertensive LES also have high-amplitude distal esophageal peristalsis consistent with nutcracker esophagus. Treatment

Because the pathophysiology of discoordinated and hypercontractile esophageal motility disorders is poorly understood and the association with symptoms is unclear, only a few appropriately designed and powered studies have evaluated therapy for these disorders. Generally, the medications that have been used with success have been used to treat all these disorders (ie, diffuse esophageal spasm, nutcracker esophagus, and hypertensive LES).

Nitrates and calcium channel blockers both have some effect. Diltiazem was studied in a randomized, double-blind crossover study (60–90 mg 4 times daily), and it relieved chest pain, compared with placebo, in patients with nutcracker esophagus. There have been anecdotal reports of the usefulness of nitrates and anticholinergic agents, but these have not been studied in a controlled trial. Gastroesophageal reflux disease has been suggested to have a role in spastic disorders of the esophagus, and some small studies have shown symptomatic improvement with proton pump inhibitor therapy. Trazodone (100–150 mg/day) and imipramine (50 mg/day) have been shown to be effective in improving chest pain of patients with esophageal motility disorders, likely by modifying visceral sensation. Sildenafil inactivates nitric oxidestimulated cyclic guanosine monophosphate and, as a result, can cause the LES to relax. Several investigators have studied the effect of sildenafil, at a dose of 50 mg, on the LES and have found that it relieved symptoms in a group of 11 patients who had nutcracker esophagus or diffuse esophageal spasm. Injection of botulinum toxin also has been studied in this group of patients. The largest series included 29 patients who had chest pain or dysphagia and the diagnosis of a spastic esophageal motility disorder (diffuse esophageal spasm, hypertensive LES, or nutcracker esophagus). The patients received 100 units of botulinum toxin injected at the Z-line (given as five circumferential injections of a 20 units/mL injection), and 70% of the patients had some improvement and nearly 50% had complete relief. Patients who have profound symptoms (eg, severe dysphagia associated with weight loss or aspiration associated with severe dysphagia) refractory to other treatments may benefit from long esophageal myotomy. Hypocontractile Disorders

Hypocontractile esophagus disorders are characterized by either hypotensive LES alone or in combination with increased lowamplitude peristalsis (>50% wet swallows and low mean distal peristaltic amplitude [10 years) reflux symptoms have evidence on endoscopy of long-segment Barrett’s esophagus I.7. Following the diagnosis of long-segment Barrett’s esophagus with no dysplasia on initial endoscopy, a 70-year-old patient who has gastroesophageal reflux well-controlled with once-a-day PPI returns for consultation. Which of the following would be the most appropriate next recommendation? a. Return in 1 year for repeat endoscopy and surveillance biopsies b. Undergo fundoplication c. Increase dose of PPI to twice a day d. Undergo endoscopic ablative therapy e. Return in 3 months for additional biopsy and endoscopic ultrasonography I.8. High-grade dysplasia is diagnosed with surveillance endoscopy in a 75-year-old woman. Endoscopic evaluation shows a 3-cm segment of Barrett’s esophagus, with a 1-cm nodular lesion in the distal esophagus. Her medical history is remarkable for diabetes mellitus, coronary artery disease, and cardiac bypass surgery. Physical examination findings include marked obesity, with a body mass index of 37. CT does not show any evidence of metastatic disease. She is concerned about the premalignant potential of her condition but wants to avoid surgery. She is interested in pursuing endoscopic therapy. Which of the following facts about endoscopic and surgical therapy for high-grade dysplasia is true?

a. Endoscopic mucosal resection is indicated to assess accurately the nodular lesion in the distal esophagus b. Surgical mortality and morbidity do not depend on hospital or surgical volume c. Endoscopic therapy can be achieved in a single procedure without need for surveillance endoscopy thereafter d. The survival of patients treated surgically is superior to that of those treated endoscopically e. Recurrence of neoplasia after endoscopic therapy is negligible I.9. A 65-year-old man is referred with a history of longsegment Barrett’s esophagus and an outside diagnosis of low-grade dysplasia. He is very concerned about further progression and development of esophageal adenocarcinoma. He has no medical problems and takes no medication except for a PPI. His symptoms are well controlled, and there is no evidence of esophagitis. He has no visible lesion on endoscopy. He asks numerous questions about further approach to the management of low-grade dysplasia and the risk of progression of low-grade dysplasia to esophageal adenocarcinoma. The next most appropriate step in management is: a. Increase dosage of PPI to twice a day and have patient return for endoscopy in 3 months b. Consider endoscopic ablative therapy c. Have an expert gastrointestinal pathologist confirm the diagnosis of low-grade dysplasia and have the patient return for repeat endoscopic surveillance in 6 months d. Request thoracic surgery consultation for esophagectomy e. Enroll patient in chemoprevention trial I.10. You evaluate a 55-year-old man who presented with new-onset dysphagia and was found to have distal esophageal adenocarcinoma, evident as a 3-cm mass on endoscopy. Staging has been performed with positron emission tomography and CT of the chest and abdomen, which showed no distant metastatic lesions. Endoscopic ultrasonography demonstrated a T2N1 lesion, and fine-needle aspiration cytology of a distal periesophageal lymph node was positive for malignant cells. Initially, he lost 10 lb in weight but weight loss has now stabilized with modifications in diet and the use of nutritional supplements. The suggested course for treatment after staging would be: a. Consideration of endoscopic therapy with endoscopic mucosal resection b. Esophagectomy, followed by postoperative chemotherapy c. Preoperative chemoradiotherapy, followed by surgery

Questions and Answers

d. Esophageal stent placement for palliation of dysphagia e. Placement of a PEG tube for nutritional support I.11. A 38-year-old professional woman in good health presents with a 6-week history of esophageal dysphagia, chest pain, and regurgitation. Because of the dysphagia, she has altered her diet and has lost 10 lb in weight. EGD with esophageal biopsies was performed, and the findings were normal. What is the next best test? a. No further testing is required; the patient should take a PPI daily and symptom response should be assessed in 8 weeks b. No further testing is required; the patient likely has psychogenic dysphagia and should be referred to a behavioral therapist c. Stationary esophageal manometry d. 24-Hour ambulatory esophageal pH study e. 24-Hour ambulatory esophageal manometry I.12. A 48-year-old woman presents with solid food esophageal dysphagia and typical heartburn symptoms. Both symptoms have developed gradually over the past 2 years, along with other vague symptoms of myalgias and arthralgias. She has Raynaud’s phenomenon. EGD shows Los Angeles grade C esophagitis. Which of the following is the most likely finding on esophageal manometry? a. Hypertensive lower esophageal sphincter b. Hypotensive lower esophageal sphincter that relaxes normally c. Increased amplitude of the peristaltic wave in the distal esophagus (>220 mm Hg) d. Multipeaked contractions with a prolonged duration (>6 seconds) e. Complete absence of peristalsis I.13. A 76-year-old man presents with intermittent chest pain that is associated with dysphagia. Occasionally, the pain has been very severe, prompting evaluation

41

in the emergency department for cardiac causes, of which none has been found. EGD showed Los Angeles grade B esophagitis. A barium esophagram showed a “corkscrew” esophagus. Which of the following would you expect to find on esophageal manometry? a. Absence of peristalsis b. Low distal peristaltic amplitude (average 6 seconds), and intermittent peristalsis. One would not expect to see an absence of peristalsis; low distal peristaltic amplitude; a hypertensive, poorly relaxing lower esophageal sphincter; or a hypotensive lower esophageal sphincter. I.14. Answer e. Patients who have oropharyngeal dysphagia and clear evidence of a cricopharyngeal bar that obstructs easy passage of the food bolus are treated best with cricopharyngeal myotomy, although bougie dilation can be tried initially. For this young patient, endoscopic cricopharyngeal myotomy would not be the best choice because of the relative lack of published experience with this technique. Medications have little role in the treatment of cricopharyngeal bar. Heller myotomy is reserved for the treatment of achalasia.

I.15. Answer d. Recent data suggest that the most durable treatment of achalasia with the fewest complications in young patients who are good surgical candidates is laparoscopic Heller myotomy with Dor fundoplication. If a dilation is to be performed, it should be with a rigid pneumatic balloon, not a bougie dilator. Injection of botulinum toxin into the lower esophageal sphincter works well and has few risks but is short-lived, with a limit on the effectiveness of repeated dilations. Thus, it would be inappropriate for this young patient. There is good evidence that the combination of myotomy with partial fundoplication (eg, Dor fundoplication) is better than myotomy alone because it prevents most complications of reflux disease that previously were commonplace when only myotomy was performed. Esophagectomy is reserved for patients who have a very dilated esophagus and for whom myotomy alone failed.

Section II Stomach

This page intentionally left blank

4 Peptic Ulcer Disease Stephen C. Hauser, MD

A peptic ulcer is a persistent 5 mm or larger break in the gastrointestinal mucosa of the stomach or duodenum that penetrates through the muscularis mucosa. At endoscopy, an ulcer should be readily apparent with perceivable depth. Smaller and more shallow mucosal breaks represent erosions. Most ulcers are due to infection with Helicobacter pylori (HP) or the administration of nonsteroidal antiinflammatory drugs (NSAIDs). The term peptic ulcer disease (PUD) refers to ulceration that depends in part on the acid and peptic (pepsins) activity of gastric juice. Gastric malignancy is an example of ulceration that may occur in the absence of acid and peptic activity. Most PUD involves the stomach or duodenal bulb. Ulceration distal to the duodenal bulb should suggest Zollinger-Ellison syndrome, Crohn’s disease, ischemia, or malignancy. Epidemiologya

The lifetime prevalence of PUD in the United States may be as high as 10% among the general population and up to 20% for persons infected with HP. The incidence of duodenal ulcers in the United States increased during the twentieth century until the 1960s and since then has decreased steadily, as has the incidence of HP infection. The frequency of gastrointestinal tract hemorrhage due to PUD is decreasing among younger persons but increasing among the elderly. Pathophysiology

Peptic ulcers occur when there is an imbalance between processes that damage the gastrointestinal mucosa and mechanisms that Abbreviations: COX-2, cyclooxygenase-2; ECL, enterochromaffinlike; HP, Helicobacter pylori; MALToma, mucosa-associated lymphoid tissue lymphoma; NSAID, nonsteroidal antiinflammatory drug; PPI, proton pump inhibitor; PUD, peptic ulcer disease

protect it. Mucosal defense mechanisms include the surface mucus layer; the secretion of bicarbonate, mucus, and phospholipid by gastroduodenal epithelial surface mucus cells; the epithelial barrier; mucosal blood flow; epithelial cell restitution; and epithelial cell renewal (Fig. 4.1). Many of these defense mechanisms are prostaglandin-dependent. Processes that can be injurious to the gastrointestinal mucosa include ischemia (shock, cardiovascular disease, and vasoconstrictive agents such as cocaine), infection (HP, cytomegalovirus, and syphilis), drugs (NSAIDs, aspirin, antiplatelet drugs, sirolimus, and bisphosphonates), chemotherapy, radiation, gastricoutlet obstruction and other acid-hypersecretory states (Zollinger-Ellison syndrome and systemic mastocytosis), infiltrative disorders (Crohn’s disease, sarcoidosis, and malignancy), cigarette smoking, and vasculitis. Both cirrhosis and chronic obstructive lung disease are associated with PUD, but the pathophysiologic mechanisms are unclear. Ulceration may occur in critically ill patients (stress-related PUD); this usually involves the stomach, with ischemia being the most likely mechanism. Helicobacter pylori Infection

Worldwide, HP infection is the major cause of both duodenal and gastric ulcers. More than one-half of all persons in the world are infected with HP, and 5% to 20% of them have ulcers. In developing countries, the majority of children (>70%) are infected with HP before the age of 10 years, and more than 90% of adults by age 50. In the United States, HP infection is uncommon in persons born after 1945. Improved sanitation and the increasing use of antibiotics during childhood may be largely responsible for this. Persons born before 1945 are more likely to have been infected with HP (60% of them by age 60). Lower socioeconomic status and household crowding (including children sharing beds) are risk factors for HP infection. It is likely that the transmission of HP is from person to person, through oral-oral and fecal-oral routes. 47

48

Section II Stomach

Exogenous factors: NSAIDs, alcohol

Acid + Pepsin

Endogenous factors: bile, lysolecithin

First-line defense: mucus/bicarbonate barrier Second-line defense: epithelial cell mechanisms • Barrier function of apical plasma membrane • Intrinsic cell defense • Extrusion of acid Third-line defense: blood-flow–mediated • Removal of back-diffused H+ • Supply of energy A

Epithelial cell injury First-line repair: restitution Second-line repair: cell replication Acute wound formation Third-line repair: wound healing • Formation of granulation tissue • Angiogenesis • Remodeling of basement membrane

B

Ulcer

Figure 4.1 Cascade of mucosal defense (A) and repair (B)

mechanisms. Damaging effects on epithelial cells of exogenous and endogenous factors are amplified by peptic acid activity. If the three sets of lines of defense mechanisms fail, epithelial cell injury occurs that is repaired by restitution and cell replication. If these repair mechanisms fail, an acute wound forms. Ulcers form only with failure of acute wound healing mechanisms. NSAID indicates nonsteroidal antiinflammatory drug. (Adapted from Soll AH. Peptic ulcer and its complications. In: Feldman M, Sleisenger MH, Scharschmidt BF, editors. Sleisenger & Fordtran’s gastrointestinal and liver disease: pathophysiology, diagnosis, management. Vol 1. 6th ed. Philadelphia: WB Saunders Company; 1998. p. 620–78. Used with permission.)

HP is a gram-negative, spiral-shaped microaerophilic bacterium with multiple flagella. The organisms exist as dormant coccoid forms in culture. HP survives only on gastric-type mucosa in the stomach or metaplastic gastric-type epithelium in the duodenum, esophagus (Barrett’s epithelium), or small bowel (Meckel’s diverticulum). In the stomach, HP lives within the mucus layer, surviving the acid pH of the stomach in part by its urease activity, which converts urea (ubiquitous) to ammonia, and by its motility, its ability to adhere to epithelial cells, its microaerophilic properties, and its proteases, which may digest mucus (Fig. 4.2). HP does not often survive in an alkaline environment in the stomach (eg, after gastroenterostomy). HP damages gastric-type mucosa through its production of ammonia, proteases, lipases, phospholipases, and mucinases

Figure 4.2 Section of stomach showing Helicobacter pylori (black, rod-shaped structures). (Wenger-Angritt stain.) (Adapted from Emory TS, Carpenter HA, Gostout CJ, Sobin LH. Atlas of gastrointestinal endoscopy & endoscopic biopsies. Washington, DC: Armed Forces Institute of Pathology, American Registry of Pathology; 2000.)

and by its induction of a local immune response (chemotactic factors for neutrophils and monocytes, as well as a host T-cell response). HP-specific virulence factors associated with ulcer disease include a cag pathogenicity island whose product (CagA) enters host epithelial cells, and certain vacA loci that encode a bacterial toxin (VacA). Many persons infected with HP have higher than normal levels of gastrin because of antralpredominate active chronic gastritis that decreases the number of antral D cells and the level of somatostatin, thus resulting in a slightly increased rate of gastric acid secretion and the greater likelihood of a duodenal ulcer developing. Higher than normal levels of gastric acid secretion may damage the duodenum and result in gastric metaplasia, allowing HP to colonize the duodenum. This pattern appears to be more common in persons who are infected with HP later in life. In contrast, other persons infected with HP, more often early in life, have a more multifocal and pangastric gastritis, with parietal cell damage, decreased production of acid, and a greater likelihood of a gastric ulcer developing.

4 Peptic Ulcer Disease

Eradication of HP in infected patients with PUD both eliminates gastric and duodenal ulcers (along with antisecretory therapy) and prevents recurrence. Currently, in the United States, recurrent infection with HP is less than 3% per year. Nonsteroidal Antiinflammatory Drugs

Nearly 20 million persons in the United States take NSAIDs daily, and up to 200,000 of these will develop serious complications such as gastrointestinal tract bleeding, perforation, or death (about 16,000 each year). Between 5% and 30% of persons who ingest full-dose NSAIDs long term have an ulcer found on endoscopy, often without symptoms. NSAIDs and aspirin can directly damage the gastric epithelium upon oral ingestion (topical effects) and can systemically, when present in the blood, inhibit the production of prostaglandins by gastroduodenal epithelial cells, thus adversely affecting multiple defense mechanisms. Risk factors for NSAID-related gastrointestinal ulceration and its complications include age (older than 60–70 years); pervious history of PUD (especially if complicated); the first 30 to 90 days of therapy; high-dose NSAIDs; simultaneous use of multiple NSAIDs; concomitant use of corticosteroids, aspirin, other antiplatelet drugs, alendronate, or anticoagulants; history of comorbid conditions; and HP infection. Specific Agents

The risk of injury to the gastroduodenal mucosa by aspirin is dose-related and can occur even with administration of lowdose aspirin. Because of the antiplatelet effects of aspirin and other NSAIDs, complications such as ulceration with bleeding or perforation can occur throughout the gastrointestinal tract. Cyclooxygenase-2 (COX-2) selective inhibitors have been associated with a decreased risk of clinically apparent PUD, including complications, but similar to nonselective NSAIDs, there are concerns about cardiovascular risks. Up to 5% of patients who take COX-2 inhibitors may develop PUD, and ulceration in persons who subsequently ingest COX-2 inhibitors may be slow to heal. COX-2 activity appears to be important for ulcer healing. Persons who take both low-dose aspirin and a COX-2 inhibitor are at increased risk for PUD and its complications compared with those who take either drug alone. Antiplatelet agents such as clopidogrel have been linked to a high rate of complications such as gastrointestinal tract bleeding, particularly in patients who previously have had gastrointestinal complications, whether taken alone or with NSAIDs. Platelets, similar to COX-2 activity, appear to be important for ulcer healing.

have increased levels of gastric acid secretion and those who have active or subclinical PUD are likely to be at increased risk for PUD and its complications after initiation of NSAID therapy. Two controlled trials have shown that eradication of HP infection before treatment with NSAIDs decreases these risks. In contrast, persons who take NSAIDs long term and who have a history of complicated PUD and also are infected with HP are not protected from additional PUD complications after eradication of HP infection. On-going ingestion of NSAIDs perpetuates the increased risk for additional complications. Gastric Acid

The presence of acid in the upper gastrointestinal tract along with proteolytic pepsins is necessary for the development of PUD. Indeed, partial inhibition of gastric acid secretion can heal ulcers, albeit temporarily, in persons infected with HP. Acid secretion occurs through gastric proton pumps (hydrogen, potassium-ATPase) located in parietal cells. At rest, these pumps are located intracellularly. With stimulation of parietal cells by the vagus nerve (acetylcholine, a neurocrine mediator), gastrin (from antral G cells, an endocrine mediator), and histamine (from enterochromaffin-like, or ECL, cells, a paracrine mediator), the proton pumps are translocated to the apical secretory canalicular (luminal) membrane, where they become functional (Fig. 4.3). During the cephalic phase of meal-stimulated acid secretion, vagal activity stimulates ECL cells, G cells, and parietal cells. During the gastric phase, distention of the stomach augments vagal output and short peptides, amino acids, and calcium, as well as alkaline pH, stimulate gastrin release by G cells. Gastrin release is inhibited by a gastric pH less than 3. Acid pH also stimulates somatostatin-producing D cells in the antrum and body of the stomach, with somatostatin (as a paracrine mediator) inhibiting gastrin release from G cells and acid secretion from parietal cells. Gastric acid

Stimulants

Inhibitors

Histamine: ECL, mast cells

Somatostatin: D cells Prostaglandins: epithelial and nonepithelial cells of the stomach

Acetylcholine: vagus nerve Gastrin: G cells

Helicobacter pylori Infection and Nonsteroidal Antiinflammatory Drugs

The relationship between NSAID use and HP infection is complex and controversial. Patients infected with HP who

49

Parietal cell Figure 4.3 Physiology of the parietal cell. ECL indicates

enterochromaffin-like.

50

Section II Stomach

Hypersecretion of Gastric Acid

Infection and Miscellaneous Conditions

Mild hypersecretion of gastric acid occurs in many but not all patients with PUD who are infected with HP, especially in those with duodenal ulcers. Zollinger-Ellison syndrome is a rare cause of excessive hypersecretion of gastric acid. Most of these patients develop ulceration of the upper gastrointestinal tract, often involving the esophagus, stomach, and duodenum, including ulceration of the duodenum beyond the duodenal bulb. Gastric acid hypersecretion with PUD is found in about one-third of patients who have systemic mastocytosis because of the increased release of histamine by mast cells. Increased release of histamine, acid hypersecretion, and PUD have been identified also in patients who have basophilic leukemia and chronic myelogenous leukemia with basophilia. Uncommonly, PUD patients with gastric acid hypersecretion due to hypergastrinemia who do not have Zollinger-Ellison syndrome (negative secretin test) have been found to have antral G-cell hyperplasia. Many, but not all, of these patients also are infected with HP, and their PUD and gastric acid hypersecretion resolve with eradication of HP. Gastric outlet or duodenal obstruction due to various causes produces gastric distention, hypergastrinemia, hypersecretion of acid, and ulceration proximal to the obstruction. Severe PUD, hypersecretion of gastric acid, and hypergastrinemia are common in the few patients who have retained antrum syndrome after a Billroth II operation, when a small cuff of gastric antrum is left as part of the afferent limb (Fig. 4.4). Finally, patients who undergo substantial small-bowel resection can develop hypergastrinemia, hypersecretion of gastric acid, and severe PUD postoperatively because of the sudden loss of inhibitors of gastrin and acid secretion.

Ulceration of the gastroduodenal area rarely occurs with infections other than HP. In immunocompromised patients, cytomegalovirus infection is a not uncommon cause of ulceration, often producing large, deep, and multiple ulcers and frequently complicated by bleeding, perforation, or obstruction. Rarely, ulceration can occur from infections such as syphilis or tuberculosis (often antral) and from postradiation therapy, sarcoidosis, Crohn’s disease, vasculitis, and ischemia (rare in the unoperated stomach). Ulceration can be due to malignancy, usually in the stomach, including gastric adenocarcinoma, lymphoma, sarcoma (gastrointestinal stromal tumors), and metastatic malignancies. PUD is more common in persons who smoke cigarettes and is more likely to be complicated, to require surgery, to be slow to heal, and to recur than in nonsmokers. Although alcohol can directly damage the gastroduodenal mucosa and stimulate acid secretion, there is no evidence that alcohol is a risk factor for PUD. Similarly, there is no clear link between either diet or psychologic factors and ulcer disease.

Retained antral tissue with gastrinsecreting cells

Clinical Features

PUD often is asymptomatic, particularly in patients with PUD due to NSAIDs or aspirin. Symptoms of a duodenal ulcer include dyspepsia, which is an epigastric burning or hunger-like sensation, especially 1 to 3 hours after a meal or during the night, that improves after the ingestion of food, antacids, or antisecretory agents. Thus, these patients tend to gain weight. In comparison, persons with gastric ulcers are more likely to experience discomfort after eating and may have weight loss. Patients who have nonulcer dyspepsia can have the same symptoms as those with PUD, and many persons with PUD have atypical symptoms. Also, the overlap of symptoms between patients with gastroesophageal reflux disease and those with PUD is considerable. PUD usually is diagnosed with endoscopy, which in contrast to upper gastrointestinal tract radiography, allows for biopsy of gastric ulcers to exclude malignancy and to diagnose HP infection.

Diagnosis of Helicobacter pylori Infection

Figure 4.4 Retained antrum after Billroth II operation.

(Adapted from Costamagna G. ERCP after Billroth II reconstruction [Internet]. In: UpToDate, Basow DS, editor; Waltham [MA]: c2010 [cited 2008 May 9]. Available from http://www.uptodate.com/. Used with permission.)

Many diagnostic tests are available for making the diagnosis of HP infection (Table 4.1). In patients who have PUD but no obvious cause of disease (eg, NSAID use), a negative test for HP should be confirmed with a second independent diagnostic test, because false-negative results are not uncommon. Many tests (culture, histology, breath and stool antigen tests, and urease testing) depend on the number of organisms, and false-negative results are frequent if the patient has been exposed recently to antibiotics, bismuth, or a proton pump inhibitor (PPI). Patients should not receive treatment with

4 Peptic Ulcer Disease Table 4.1 Diagnostic Tests for Helicobacter pylori Test

Nonendoscopic tests In-office antibody test ELISA on serum Stool antigen test Urease breath test Endoscopic tests Biopsy urease test Histology Culture

Sensitivity, %

Specificity, %

51

Invasive Tests Cost, $

88–94

74–88

10–30

86–94 94 90–96

78–95 92 88–98

40–100 180 250–300(13C) 20–65(14C)

88–95 93–96 80–98

95–100 98–99 100

6–20 60–250 150

Abbreviation: ELISA, enzyme-linked immunosorbent assay.

antibiotics and bismuth for 4 to 6 weeks or PPIs for 2 weeks (H2-receptor blockers can be taken) before testing. Testing can be divided into noninvasive tests and invasive (requiring endoscopy) tests. Noninvasive Tests Serology

Serologic tests for antibodies against HP are noninvasive, rapid, simple, and inexpensive. Serum-based tests are more accurate than whole-blood tests. Tests that measure antibody levels in saliva are not as reliable. Serologic testing is as good as any other test for the initial diagnosis of HP infection in untested persons. However, because antibody levels may remain positive indefinitely in some patients after successful treatment, serologic testing is not useful after treatment. Office-based serologic testing is less sensitive and specific than laboratory-based testing.

Culture

HP culture after endoscopic biopsy is not widely available, is not rapid, and is less sensitive than noninvasive tests and histologic examination. However, with culture, antimicrobial sensitivity can be determined in infected persons with HP infection resistant to therapy or who have recurrent relapse of disease after treatment. Rapid Urease Tests

Biopsy specimens obtained during endoscopy can be tested for urease with rapid urease testing. These tests are rapid, highly sensitive, highly specific, and inexpensive. Bacterial overgrowth can produce false-positive results (ureaseproducing Proteus), as can non-HP Helicobacter species. Gastrointestinal tract bleeding can increase false-negative results. Histology

Histologic examination is highly sensitive and highly specific, but expensive and invasive. Biopsy specimens (5 or 6) should be taken from the greater and lesser curvatures of the antrum and fundus as well as from the incisura. Silver staining is recommended to facilitate the detection of HP. Endoscopy also allows additional evaluation of PUD (eg, biopsy of gastric ulceration to exclude malignancy or to detect gastric metaplasia or dysplasia). Treatment

The HP stool antigen test is noninvasive, rapid, highly sensitive and highly specific, but relatively expensive. In contrast to serologic testing, it can be used to evaluate active infection as well as response to therapy; thus, it is cost-effective.

The treatment of PUD depends on the underlying pathogenesis. Specific treatment is available for PUD in patients infected with HP or cytomegalovirus or who have syphilis or tuberculosis. Patients with gastroduodenal ulceration due to Crohn’s disease, vasculitis, ischemia, gastroduodenal obstruction, other acid hypersecretory states, or malignancy require specific therapy directed at the underlying condition. Treatment of drug-induced gastroduodenal ulceration is nonspecific, usually consisting of acid-suppressive medication and often withdrawal of use of the offending drug. Elimination of cigarette smoking will help all PUD patients.

Urease Breath Test

Treatment of Helicobacter pylori Infection

Because HP is practically the only infectious organism in the upper gastrointestinal tract capable of producing urea via urease, carbon 13- or carbon 14-labeled urea can be administered as a breath test. The urease breath test is relatively noninvasive, rapid, highly sensitive, and highly specific, but expensive. Like stool antigen testing, it is highly reliable after treatment to confirm eradication of HP.

Virtually all persons with a history of PUD and all those with active PUD who also are infected with HP should be given therapy to eradicate HP. Therapy also is indicated for HP-infected patients with gastric mucosa-associated lymphoid tissue lymphoma (MALTomas) or gastric adenocarcinoma. Also, HP-infected first-degree relatives of patients with gastric adenocarcinoma should receive treatment. Some HP

Stool Antigen

52

Section II Stomach

infected patients with nonulcer dyspepsia, idiopathic thrombocytopenic purpura, or anticipated prolonged treatment with NSAIDs may benefit from eradication of HP. Indeed, because HP is classified as a carcinogen, one could consider treating all those with documented HP infection. First-line therapy for eradication of HP requires 10 to 14 days of combination therapy, or triple therapy, including a PPI plus two antibiotics, preferably clarithromycin and amoxicillin, with metronidazole in place of amoxicillin for patients allergic to penicillin (Table 4.2). In areas where clarithromycin resistance is high (>15%-20%), the effectiveness of this triple combination therapy is less than 70%. If this is known, metronidazole should be given in place of clarithromycin. Although metronidazole resistance is not uncommon, giving it in combination with other agents (PPI, bismuth, or tetracycline) besides clarithromycin and prescribing a higher dose (500 mg twice daily) can diminish the rate of treatment failure. Second-line therapy, or quadruple therapy, for those whom first-line therapy failed or as an alternative first-line therapy for those in areas where clarithromycin resistance is high includes PPI, bismuth, metronidazole, and tetracycline. Resistance to amoxicillin, tetracycline, or bismuth, is rare. Third-line therapy can be designed on the basis of results of

Table 4.2 American College of Gastroenterology First-Line Regimens for Helicobacter pylori Eradication

Patients

Regimen

Eradication rate, %

70–85 Patients who are not Standard dose PPI twice daily (or esomeprazole allergic to once daily) plus penicillin and have clarithromycin 500 mg not previously twice daily, and amoxicillin received 1,000 mg twice daily for a macrolide 10–14 days 70–85 Standard dose PPI twice Patients who are allergic to penicillin daily, clarithromycin 500 mg twice daily, and who have not metronidazole 500 mg previously received twice daily for 10–14 days a macrolide or are unable to tolerate bismuth quadruple therapy 75–90 Bismuth subsalicylate Patients who are 525 mg orally 4 times daily, allergic to metronidazole 250 mg penicillin and who orally 4 times daily, have previously tetracycline 500 mg orally received a 4 times daily, ranitidine macrolide 150 mg orally twice daily (or standard dose PPI once to twice daily) for 10–14 days Abbreviation: PPI, proton pump inhibitor. Adapted from Chey WD, Wong BCY. American College of Gastroenterology Guideline on the management of Helicobacter pylori infection. Am J Gastroenterol. 2007;102:1808–25. Epub 2007 Jun 29. Used with permission.

antibiotic sensitivity testing after culture of gastric biopsy specimens, if available, or the more recently proposed (but not completely validated) triple combination of PPI, amoxicillin, and levofloxacin (or furazolidone). So-called sequential therapy has not been tested adequately in the United States. After treatment, eradication of HP should be confirmed. Antisecretory Treatment

Treatment with antisecretory agents not only can heal gastroduodenal ulceration but also can decrease recurrent ulceration and diminish the risk of complications. Even malignant ulcers of the stomach can decrease in size or even heal temporarily with the administration of acid-reducing agents. Antacids and sucralfate are used rarely to treat PUD. An exception is PUD in pregnant women, because magnesium-containing antacids (except when given near delivery) and sucralfate are safe to administer during pregnancy. For most patients, sodium-containing antacids should be avoided (because of salt load and systemic alkalosis), as should calcium-containing antacids (because of systemic alkalosis, hypercalcemia, and rebound acid hypersecretion). Magnesium-containing antacids may cause diarrhea and should be avoided for persons with renal dysfunction. Aluminum-containing antacids and sucralfate may cause constipation. Prostaglandin analogues (misoprostol), which are antisecretory at pharmacologic doses, are used rarely to treat PUD. Adverse effects, including diarrhea, headache, and abdominal pain, are particularly common and troublesome. These agents are uterotropic and thus contraindicated for potentially fecund women. H2-Receptor Antagonists

Four H2-receptor antagonists (cimetidine, ranitidine, nizatidine, and famotidine) are available. All of them are highly effective for the treatment of PUD. Cimetidine rarely is prescribed, because of its short half-life, several drug-drug interactions, which are due to its binding to cytochrome P450 (decreased metabolism of warfarin, lidocaine, theophylline, and phenytoin), and adverse effects (eg, dose-dependent gynecomastia and breast tenderness). Ranitidine and nizatidine are longer acting, and famotidine has the longest duration of action. However, healing of ulceration can be accomplished with an 8-week regimen of single bedtime dosing with any H2 blocker. H2-receptor blockers do not have anti-HP activity. Proton Pump Inhibitors

PPIs are the most potent antisecretory agents available. They are enteric coated or combined with sodium bicarbonate to protect them from acid inactivation. After being absorbed in the upper small bowel, PPIs are taken up by parietal cells and secreted into the canalicular (luminal) space, where they are converted to a metabolite that binds covalently with proton pumps and irreversibly inactivates them. These proton pumps must be active, and the parietal cells must be stimulated for the

4 Peptic Ulcer Disease

PPI-proton pump interaction to occur. Thus, PPIs should be taken 15 to 30 minutes before a meal. They are not as effective if administered while acid secretion is being inhibited by H2-receptor blockers. Several PPIs are available, including omeprazole, lansoprazole, pantoprazole, rabeprazole, and esomeprazole. All PPIs are capable of healing duodenal ulcers in 4 weeks. Gastric ulcers, depending on size, may require a considerably longer duration of treatment.

Follow-up and Maintenance Therapy

Patients who have uncomplicated duodenal ulcers without HP infection do not require follow-up endoscopy after therapy if they do not have recurrent or persistent symptoms. If duodenal ulcer is due to the use of an NSAID, discontinuation or reduction in the dose of the NSAID is recommended. If this is not possible (eg, treatment with aspirin or other antiplatelet agents in patients with cardiovascular disorders), on-going prophylactic use of a PPI may be required after treatment. Generally, follow-up endoscopy is recommended for all patients with gastric ulcers, to exclude malignancy. However, about 95% of gastric cancers associated with gastric ulceration can be diagnosed at initial endoscopy when an adequate number of biopsy specimens are obtained from the edge (at least four) and base (at least one) of the ulcer. Patients at high risk for gastric cancer (eg, born in an endemic area with a high prevalence of gastric adenocarcinoma or first-degree relative with gastric adenocarcinoma) or gastric ulcers with worrisome features at endoscopy (eg, ulceration within a mass lesion or distortion of the adjacent normal fold pattern) should have follow-up endoscopy to document complete healing of the ulcer. Persons with gastric or duodenal ulcers who are not infected with HP or do not take NSAIDs (idiopathic ulcers) may benefit from maintenance PPI therapy to prevent ulcer relapse. Similarly, patients with large (>2 cm in diameter) ulcers or ulcers complicated by fibrosis or hemorrhage may benefit from prolonged antisecretory therapy. Also, patients who have HP infection and complicated PUD (eg, hemorrhage) should receive maintenance acid-suppressive therapy after eradication of HP and healing of the ulceration.

53

Prevention of Peptic Ulcer Disease

Persons about to be prescribed aspirin or NSAIDs for longterm therapy should be tested for HP and treated if infected. If they are at high risk for PUD (see above), prophylactic therapy with misoprostol or a PPI should be considered. H2-receptor blockers have not been useful for these patients. Patients at highest risk are those with a previous history of ulcer disease; the very elderly; patients also receiving treatment with warfarin, corticosteroids, or other antiplatelet agents; and those with clinically important comorbid conditions. SUGGESTED READING

Chan FK, Abraham NS, Scheiman JM, Laine L; First International Working Party on Gastrointestinal and Cardiovascular Effects of Nonsteroidal Anti-inflammatory Drugs and Anti-platelet Agents. Management of patients on nonsteroidal anti-inflammatory drugs: a clinical practice recommendation from the First International Working Party on Gastrointestinal and Cardiovascular Effects of Nonsteroidal Anti-inflammatory Drugs and Anti-platelet Agents. Am J Gastroenterol. 2008 Nov;103(11): 2908–18. Epub 2008 Oct 1. Choung RS, Talley NJ. Epidemiology and clinical presentation of stress-related peptic damage and chronic peptic ulcer. Curr Mol Med. 2008 Jun;8(4):253–7. Kiltz U, Zochling J, Schmidt WE, Braun J. Use of NSAIDs and infection with Helicobacter pylori: what does the rheumatologist need to know? Rheumatology (Oxford). 2008 Sep;47(9):1342–7. Epub 2008 May 13. Malfertheiner P, Chan FK, McColl KE. Peptic ulcer disease. Lancet. 2009 Oct 24;374(9699):1449–61. Epub 2009 Aug 13. McColl KE. How I manage H. pylori-negative, NSAID/aspirin-negative peptic ulcers. Am J Gastroenterol. 2009 Jan;104(1):190–3. Napolitano L. Refractory peptic ulcer disease. Gastroenterol Clin North Am. 2009 Jun;38(2):267–88. O’Connor A, Gisbert J, O’Morain C. Treatment of Helicobacter pylori infection. Helicobacter. 2009 Sep;14 Suppl 1:46–51. Rokkas T, Sechopoulos P, Robotis I, Margantinis G, Pistiolas D. Cumulative H. pylori eradication rates in clinical practice by adopting first and second-line regimens proposed by the Maastricht III consensus and a third-line empirical regimen. Am J Gastroenterol. 2009 Jan;104(1):21–5.

This page intentionally left blank

5 Gastritis Stephen C. Hauser, MD

Inflammation of the stomach can manifest itself as gastritis or gastropathy. Gastritis refers to inflammatory processes of the stomach in which inflammation is the predominant feature. In comparison, gastropathy refers to epithelial damage with little or no inflammation. Gastritis may or may not have identifiable endoscopic findings or clinical symptoms. There are several classifications of gastritis and gastropathy. Most of them distinguish between acute and chronic disease in terms of duration (short term vs long term) and the predominant inflammatory infiltrate seen in biopsy specimens (neutrophilic vs mononuclear cell infiltration). The etiology of the gastritis or gastropathy (not always known) also may be an important factor in the classification, as may the topography or specific areas of involvement in the stomach. The Sydney System (Table 5.1) often is used to classify chronic gastritis. It requires two biopsy specimens from the antrum (greater and lesser curvatures) and the body (greater and lesser curvatures) of the stomach and 1 or 2 biopsy specimens from the incisura (Fig. 5.1). In some cases, duodenal biopsies may assist in the overall diagnosis, for example, celiac disease in patients with lymphocytic gastritis, Crohn’s disease in patients with granulomatous gastritis, and eosinophilic gastroenteritis in patients with eosinophilic infiltration of the antrum. Gastritisa Acute Gastritis

By definition, acute gastritis is an acute inflammatory process that involves the stomach, with a predominate neutrophilic infiltrate, with or without intramucosal hemorrhage or superficial

Abbreviations: CMV, cytomegalovirus; GAVE, gastric antral vascular ectasia; HP, Helicobacter pylori; NSAID, nonsteroidal antiinflammatory drug

Table 5.1 Sydney System for Classification of Chronic Gastritis Type of Gastritis Etiologic Factors

Nonatrophic

Gastritis Synonyms

Superficial Diffuse antral gastritis Chronic antral gastritis Interstitial follicular Type B AtrophicAutoimmunity Type A autoimmune H pylori Diffuse corporeal Pernicious anemia Type B AtrophicH pylori Environmental multifocal Metaplastic Atrophic pangastritis Progressive intestinalizing pangastritis Reactive Chemical/ Bile Reflux radiation Nonsteroidal antiinflammatory drugs Radiation Alcohol or other agents? Radiation Varioliform Lymphocytic Idiopathic Celiac-associated Autoimmune Gluten H pylori Isolated granulomatous Granulomatous Crohn’s disease Sarcoidosis Wegener’s granulomatosis Infectious Allergic Eosinophilic Food sensitivities Allergies Idiopathic Phlegmonous Infectious Bacteria Cytomegalovirus gastritides Viruses Anisakiasis Fungi Parasites Helicobacter pylori

55

56

Section II Stomach

Figure 5.1 Gastric biopsy protocol to diagnose gastritis. Biopsy

specimens (circles) should be obtained from the greater and lesser curvatures of the body and the antrum and from the incisura (dotted line).

mucosal sloughing (erosion) or both. Most often, acute gastritis is related to a toxic substance (aspirin and nonsteroidal antiinflammatory drugs [NSAIDs], excess alcohol intake, uremic toxins, heavy tobacco use, or cancer chemotherapeutic drugs), ischemia (severe stress, trauma, shock, sepsis, burns, hypothermia, or radiation), or infection (Helicobacter pylori [HP], cytomegalovirus [CMV], phlegmonous gastritis [multiple bacteria], or anisakiasis). Acute gastritis due to HP infection is identified rarely. A few patients may develop abdominal pain, nausea, or vomiting. Endoscopic findings are nonspecific (erythema and erosion) or absent. Biopsy specimens show mucosal neutrophilic infiltration in the antrum, with or without desquamation and erosions. The great majority of persons acutely infected with HP go on to develop an active chronic gastritis. Aspirin and NSAIDs can produce an acute injury to the gastric mucosa, with little to no inflammation (gastropathy). Hemorrhagic and erosive changes can be seen in biopsy specimens, with or without endoscopic findings (petechial hemorrhage or erosions) or clinical symptoms (abdominal pain, nausea, vomiting, or gastrointestinal tract bleeding). Similar findings occur after the ingestion of large amounts of alcohol and other toxic substances and after ischemia (Fig. 5.2). All these insults impair mucosal barrier function by affecting prostaglandin synthesis (NSAIDs) or mucosal blood flow (alcohol or ischemia) or by direct injury to the surface mucosal cells (NSAIDs, other drugs, or infection). Treatment for acute gastritis includes management of the underlying condition, withdrawal of any offending drug or toxin, and acidsuppression therapy with a proton pump inhibitor.

Figure 5.2 Stomach—Acute hemorrhagic gastritis. (From Emory TS, Carpenter HA, Gostout CJ, Sobin LH. Atlas of gastrointestinal endoscopy & endoscopic biopsies. Washington, DC: Armed Forces Institute of Pathology, American Registry of Pathology; 2000. p. 101.)

and gastropathy. Mucosal injury occurs in all forms of chronic gastritis. The atrophy in the atrophic forms of chronic gastritis can be autoimmune or multifocal, and, subsequently, metaplasia, dysplasia, and carcinoma may develop in some cases. Nonatrophic Chronic Gastritis

This type of chronic gastritis is typical in HP-infected persons with acute gastritis who do not clear the infection (Fig. 5.3). It usually is most evident in the antrum. The inflammatory infiltrate includes acute inflammatory cells (neutrophils) and chronic inflammatory cells (lymphocytes, plasma cells, and eosinophils). Lymphoid aggregates or germinal centers may be seen. Gastric atrophy, metaplasia, and dysplasia are not seen. Persons with this antral-predominate, active chronic gastritis due to HP infection are at increased risk for duodenal ulcer disease. Multifocal Atrophic Gastritis

Multifocal atrophic gastritis may develop in persons infected with HP. This type of active chronic gastritis includes the loss of glands and metaplastic change, principally involving the body and antrum of the stomach (Fig. 5.4). Inflammation consists of both acute and chronic inflammatory cells. These patients are at increased risk for gastric ulcer disease and gastric adenocarcinoma.

Chronic Gastritis

Autoimmune Atrophic Gastritis

Chronic gastritis is classified by the Sydney System as nonatrophic or atrophic, together with special forms of gastritis

This type of chronic atrophic gastritis is much less common than other forms of chronic gastritis (Fig. 5.5). Autoantibodies

5 Gastritis

57

A

B

Figure 5.3 Active (nonatrophic) chronic gastritis of the antrum of

the stomach. (From Emory TS, Carpenter HA, Gostout CJ, Sobin LH. Atlas of gastrointestinal endoscopy & endoscopic biopsies. Washington, DC: Armed Forces Institute of Pathology, American Registry of Pathology; 2000. p. 90.)

Figure 5.5 Chronic atrophic autoimmune gastritis. A, Gross appearance. B, Section through the body. (Hematoxylin and eosin). (From Emory TS, Carpenter HA, Gostout CJ, Sobin LH. Atlas of gastrointestinal endoscopy & endoscopic biopsies. Washington, DC: Armed Forces Institute of Pathology, American Registry of Pathology; 2000. p. 94.)

Figure 5.4 Multifocal atrophic gastritis. (From Emory TS,

Carpenter HA, Gostout CJ, Sobin LH. Atlas of gastrointestinal endoscopy & endoscopic biopsies. Washington, DC: Armed Forces Institute of Pathology, American Registry of Pathology; 2000. p. 93.)

to parietal cells, intrinsic factor, and H+, K+-ATPase result in mucosal and glandular atrophy, which is severe and involves the body and fundus of the stomach. Diffuse infiltration of lymphoplasmacytic cells, enterochromaffin-like cell hyperplasia, metaplasia, and in some patients, microcarcinoids, and gastric adenocarcinoma may be seen. Hypochlorhydria, achlorhydria, iron deficiency, hypergastrinemia, or pernicious anemia may develop in some but not all patients. Patients or their relatives may have other autoimmune disorders including Hashimoto’s thyroiditis, Graves’ disease, Addison’s disease, diabetes mellitus, and vitiligo. There is an association with HLA-B8 and HLA-DR3. On endoscopy, the loss of gastric folds and prominent submucosal vasculature may be apparent in the body and fundus of the stomach.

58

Section II Stomach

Special Forms of Gastritis

A

Chemical Gastropathy

Long-term exposure to substances that can damage the gastric mucosa can result in chronic gastropathy. Biopsy specimens show inflammation (little in contrast to the active chronic gastritis with HP infection), edema, foveolar hyperplasia with loss of mucin and glandular regenerative changes, proliferation of lamina propria smooth muscle, and vascular congestion. These histologic changes are most prominent in the antrum. Common causes of this chronic chemical or reactive gastropathy include NSAIDs, bile salts, and alcohol. Endoscopic findings include edema, erythema, vascular congestion, erosions, and, less often, ulceration (especially with NSAIDs). Bile Reflux Gastropathy

Bile reflux gastropathy is usually due to surgical intervention (gastroenterostomy) but can occur in persons with gastric emptying disorders or in stomachs that have not had a surgical procedure. Symptoms such as epigastric discomfort, nausea, and vomiting do not correlate well with endoscopic or histologic findings. Treatment with cholestyramine, sucralfate, and aluminum-containing antacids has, by and large, been unsuccessful. Ursodiol has been shown to decrease symptoms without improving histologic findings. Surgical Roux-en-Y revision to divert bile from the stomach in patients with previous gastroenterostomy and symptomatic bile reflux gastropathy has been found to reduce symptoms in 50% to 90% of these patients.

B

Infectious Gastritis

Infection with bacteria is usually due to HP (see Chapter 4). Phlegmonous gastritis is a rare, life-threatening condition associated with full-thickness purulent necrosis of the gastric wall. Multiple bacteria are responsible, and it usually occurs in immunocompromised patients, including alcoholics and patients with diabetes mellitus. Invasive procedures may trigger the onset, with fever, chills, abdominal pain, and hypotension being common. If gas-forming organisms are involved, emphysematous changes may be apparent on imaging studies. Mortality is substantial, and surgery may be required to remove the necrotic portion of the stomach. Syphilis (secondary or tertiary) and mycobacteria can involve the stomach, especially the antrum. Biopsy specimens from ulcerated or nodular areas, with special studies (dark-field examination for syphilis and acid-fast stains for mycobacteria), can be used to diagnose these infections. The most commonly recognized viral infection of the stomach is due to CMV (Fig. 5.6). This infection may or may not have endoscopic findings (edema, erythema, erosions, or ulcers). Patients with CMV infection of the stomach may or may not have symptoms, such as fever, abdominal pain, nausea, vomiting, or bleeding, and they usually are immunosuppressed.

Figure 5.6 Cytomegalovirus (CMV) infection of the stomach.

A, Gross appearance of CMV ulcers. B, CMV gastropathy. (From Emory TS, Carpenter HA, Gostout CJ, Sobin LH. Atlas of gastrointestinal endoscopy & endoscopic biopsies. Washington, DC: Armed Forces Institute of Pathology, American Registry of Pathology; 2000. p. 123.)

Biopsy specimens from macroscopically involved and apparently normal areas may show typical cytomegalic cells and inclusions. In the case of ulceration, biopsy specimens from the center of the ulcer (vascular endothelial involvement) are more likely to be diagnostic than specimens from the edge. In cases in which the diagnosis is in doubt, immunohistochemistry enhances the diagnostic yield. Fungal and parasitic infections of the stomach are uncommon. In immunosuppressed patients, gastric infections may be caused by Histoplasma, Cryptococcus, Torulopsis, or mucormycosis. Ingestion of raw fish can result in anisakiasis. Cryptosporidium and Strongyloides infections do not commonly involve the stomach, especially in immunosuppressed patients.

5 Gastritis

A

59

Noninfectious Granulomatous Gastritis

Granulomatous gastritis (Fig. 5.7) in the absence of infection occurs most commonly with Crohn’s disease. An inflammatory infiltrate may be found, most often in the antrum, in patients with Crohn’s disease. Granulomatous gastritis also occurs in some patients with sarcoidosis, collagen vascular disease, vasculitis, drug reactions, gastric foreign bodies (eg, sutures), or gastric malignancies. Many patients with granulomatous gastritis are asymptomatic. In a few patients, particularly those with Crohn’s disease or sarcoidosis, symptoms of gastric outlet obstruction due to ulceration and scarring of the antrum and pylorus can develop. Nonspecific treatment (acid-reduction therapy) or therapy directed toward the underlying disorder usually is indicated. Lymphocytic Gastritis

B

Lymphocytic gastritis often is asymptomatic. On endoscopy, edema, erythema, or erosions are seen in some patients. Histologically, there is active chronic pangastritis with epithelial infiltration by mature lymphocytes, usually T lymphocytes (Fig. 5.8). Most often, lymphocytic gastritis is found in patients with celiac disease (more antral predominant), but it also can occur in patients with HP infection (more body predominant and often with polymorphonuclear cells), microscopic colitis, or Ménétrier’s disease. Therapy is directed toward the underlying condition. Eosinophilic Gastritis

A wide variety of disorders are associated with eosinophilic infiltration of the stomach, which usually involves the mucosa as well as deeper layers. Eosinophilic gastroenteritis, also called allergic gastroenteritis, often includes peripheral eosinophilia and increased serum levels of immunoglobulin E. In the most

Figure 5.7 Granulomatous gastritis. A, Gross appearance. B, Histologic appearance of granuloma. (From Emory TS, Carpenter HA, Gostout CJ, Sobin LH. Atlas of gastrointestinal endoscopy & endoscopic biopsies. Washington, DC: Armed Forces Institute of Pathology, American Registry of Pathology; 2000. p. 98, 99.)

Figure 5.8 Lymphocytic gastritis. Intraepithelial lymphocytes can be

seen without any destruction of surrounding epithelial cells. (From Owen DA. Gastritis and carditis. Mod Pathol. 2003;16:325–41. Used with permission.)

60

Section II Stomach

common mucosa-predominant form (vs the serosa- and muscularis-predominant forms), biopsy specimens typically show patchy but dense eosinophilic infiltration (Fig. 5.9). Mucosal eosinophilic involvement of the stomach also can occur with collagen vascular disease, drug or food reactions,

vasculitis, or Anisakis infection or in patients with other causes of substantial peripheral eosinophilia (hypereosinophilic syndrome).

Gastropathy

A

B

Vascular Gastropathies

Vascular gastropathies are defined as abnormalities of the gastric vasculature affecting mucosal blood vessels, with little or no inflammation. Typical examples include congestive gastropathy from congestive heart failure, portal hypertensive gastropathy, and gastric antral vascular ectasia (GAVE), also known as watermelon stomach. In patients with portal hypertension, dilatation and sclerosis of small mucosal and submucosal venules and capillaries can produce an endoscopically recognizable mucosal mosaic pattern, usually most prominent in the fundus and body of the stomach (Fig. 5.10). Nodularity and punctate erythema also may be seen. Clinically, some patients with portal hypertensive gastropathy may have iron deficiency anemia or melena. Treatment involves attempts to lower portal pressure; endoscopic therapy is not effective (Chapter 26). Patients with GAVE develop vascular ectasias, often with occlusion by fibromuscular hyperplasia (not seen in portal hypertensive gastropathy). Endoscopic findings typically include linear or nodular erythematous streaks in the antrum, without a mosaic pattern, usually not involving the body or fundus of the stomach (Fig. 5.11). GAVE tends to occur in patients who have an array of underlying conditions, including pernicious anemia, collagen vascular disease, cirrhosis, renal failure, bone marrow transplantation, and antral mucosal trauma (as in pyloric prolapse). Similar to patients with portal hypertensive gastropathy, some patients with GAVE have iron deficiency anemia or melena. Endoscopic therapy with argon plasma coagulation can diminish blood loss in patients with anemia (Chapter 11). Of note, liver transplantation can benefit patients with GAVE who have cirrhosis, but transjugular intrahepatic portosystemic shunts are not helpful (Chapter 26). Hypertrophic Gastropathy

Figure 5.9 Eosinophilic gastritis. Note marked eosinophilic

infiltration. A, High-power and, B, low-power views. (From Emory TS, Carpenter HA, Gostout CJ, Sobin LH. Atlas of gastrointestinal endoscopy & endoscopic biopsies. Washington, DC: Armed Forces Institute of Pathology, American Registry of Pathology; 2000. p. 97.)

Hypertrophic gastropathy refers to a group of conditions with giant enlargement of the rugal folds. This includes some cases of chronic gastritis (HP infection and lymphocytic gastritis), Zollinger-Ellison syndrome, infiltrative disorders (sarcoidosis and malignancy), and Ménétrier’s disease. Patients with Ménétrier’s disease often have such symptoms as epigastric pain, nausea, vomiting, diarrhea, edema, and weight loss. Many patients have evidence of a protein-losing enteropathy (low serum level of albumin and increased stool clearance of alpha1antitrypsin). A full-thickness gastric biopsy specimen or endoscopic snare biopsy specimen from an enlarged fold usually is necessary to make the diagnosis, which often is suggested by the presence of giant gastric rugal folds (often sparing the antrum)

A

B

C

Figure 5.10 Portal hypertensive gastropathy. A and B, Gross specimens showing the mosaic mucosal pattern. C, Histologic section showing

dilated tortuous blood vessels. (From Emory TS, Carpenter HA, Gostout CJ, Sobin LH. Atlas of gastrointestinal endoscopy & endoscopic biopsies. Washington, DC: Armed Forces Institute of Pathology, American Registry of Pathology; 2000. p. 116.)

A

B

Figure 5.11 “Watermelon” stomach (antrum). A, Gross specimen, and, B, histologic section showing thick-walled ectatic vessels. (From Emory TS, Carpenter HA, Gostout CJ, Sobin LH. Atlas of gastrointestinal endoscopy & endoscopic biopsies. Washington, DC: Armed Forces Institute of Pathology, American Registry of Pathology; 2000. p. 118.)

61

62

Section II Stomach

with a cobblestone pattern seen on endoscopy. Histologic examination shows extreme surface mucous cell hyperplasia with deeper glandular atrophy. Often, gastric hypochlorhydria or achlorhydria is present together with excessive secretion of mucus. Increased levels of transforming growth factor-alpha may be responsible for the histologic changes. Improvement in symptoms after the subcutaneous administration of octreotide has been inconsistent. In addition to gastric resection, treatment with investigational agents to counteract the effects of transforming growth factor-alpha has been successful. It is unclear if patients with Ménétrier’s disease have an increased risk of gastric adenocarcinoma. SUGGESTED READING

Atten MJ, Attar BM, Teopengco E, Nadimpalli V. Gastric syphilis: a disease with multiple manifestations. Am J Gastroenterol. 1994 Dec;89(12):2227–9. Chan FK, Abraham NS, Scheiman JM, Laine L; First International Working Party on Gastrointestinal and Cardiovascular Effects of

Nonsteroidal Anti-inflammatory Drugs and Anti-platelet Agents. Management of patients on nonsteroidal anti-inflammatory drugs: a clinical practice recommendation from the First International Working Party on Gastrointestinal and Cardiovascular Effects of Nonsteroidal Anti-inflammatory Drugs and Antiplatelet Agents. Am J Gastroenterol. 2008 Nov;103(11):2908–18. Epub 2008 Oct 1. Chinitz MA, Brandt LJ, Frank MS, Frager D, Sablay L. Symptomatic sarcoidosis of the stomach. Dig Dis Sci. 1985 Jul;30(7):682–8. Dixon MF, Genta RM, Yardley JH, Correa P. Classification and grading of gastritis: the updated Sydney System. International Workshop on the Histopathology of Gastritis, Houston 1994. Am J Surg Pathol. 1996 Oct;20(10):1161–81. El-Zimaity H. Gastritis and gastric atrophy. Curr Opin Gastroenterol. 2008 Nov;24(6):682–6. Kakizoe S, Kakizoe H, Kakizoe K, Kakizoe Y, Maruta M, Kakizoe T, et al. Endoscopic findings and clinical manifestation of gastric anisakiasis. Am J Gastroenterol. 1995 May;90(5):761–3. Moosvi AR, Saravolatz LD, Wong DH, Simms SM. Emphysematous gastritis: case report and review. Rev Infect Dis. 1990 Sep–Oct; 12(5):848–55.

6 Gastric Neoplasms and Gastroenteropancreatic Neuroendocrine Tumorsa Mark V. Larson, MD

Gastric neoplasms are an important contributor to cancerrelated mortality. Of the various neoplasms that can affect the stomach, adenocarcinoma is the most common and accounts for up to 95% of all gastric neoplasms. Less common are gastric lymphomas, gastrointestinal stromal tumors, neuroendocrine tumors, and metastatic disease involving the stomach (Table 6.1). This chapter considers the epidemiology, pathogenesis, clinical presentation, diagnostic evaluation, treatment, and prognosis of these neoplastic diseases.

Gastric Adenocarcinoma

Table 6.1 Frequency of Different Types of Gastric Neoplasms

Tumor Type

Adenocarcinoma Lymphomas (diffuse large B cell and extranodal marginal zone B cell) Gastroenteropancreatic neuroendocrine tumors (including carcinoids), gastrointestinal stromal tumors, and metastatic disease to the stomach

Percentage of Gastric Neoplasms

90–95 5 48 h Avoid except for ERCP Gargle and spit Use in low doses Use in low doses Only for severe narcotic overdoses No human studies available

B C C

Avoid in 1st and 2nd trimesters Can be avoided but low risk Low risk with one-time use

No human data Limited human data: probably compatible Compatible Limited human data: potential toxicity No human data: probably compatible Probably safe

Abbreviations: ERCP, endoscopic retrograde cholangiopancreatography; FDA, US Food and Drug Administration.

is a major consideration in the choice and dosage of endoscopic medications. For particularly high-risk endoscopy such as therapeutic endoscopic retrograde cholangiopancreatography (ERCP), an anesthesiologist may be helpful in titration of medications and patient monitoring. For patients who present with gastrointestinal tract hemorrhage, in which diagnosis and therapeutic intervention are necessary, therapeutic scopes should be used. The use of medications for endoscopy is summarized in Table 21.2.

prior to the labor period, unless in the judgment of the physician the potential benefits outweigh the possible hazards, because safe use in pregnancy prior to labor has not been established relative to possible effects on fetal development.” Meperidine is preferred to diazepam or midazolam as an endoscopic premedication during pregnancy. Meperidine dosage should be titrated to produce calmness, restfulness, and mild analgesia without somnolence. Fentanyl

Meperidine

Meperidine is transferred rapidly across the placenta. Physicians have extensive experience prescribing meperidine during pregnancy, particularly during labor. Two large studies did not show teratogenicity from the administration of meperidine during the first trimester. The Collaborative Perinatal Project, a national study with the primary aim of documenting the teratogenicity of drugs taken during the first 4 months of pregnancy, followed more than 50,000 women in 12 US centers between 1959 and 1965 and reported no teratogenicity from meperidine use in 268 mothers with first-trimester exposure. Meperidine is preferred to morphine for obstetric pain because it is slower to cross the fetal blood-brain barrier. Meperidine can cause diminished fetal beat-to-beat cardiac variability that lasts for approximately 1 hour after being administered intravenously to the mother and is a common cause of decreased fetal cardiac variability during endoscopy. The labeling approved by the US Food and Drug Administration (FDA) for meperidine carries the following warning: “Meperidine should not be used in pregnant women

Fentanyl is rated a category C drug during pregnancy, and accumulated anecdotal experience suggests that it may be used in low doses for endoscopy. Fentanyl is sometimes used as an alternative to meperidine during endoscopy because of its more rapid onset of action. In several human studies, administration of fentanyl to the mother during labor produced no neonatal toxicity. Propofol

Propofol is a category B drug and is now a preferred agent for sedation in some endoscopy centers. However, it has not been studied extensively in women in the first and second trimesters and, thus, is not recommended for use during this time, based on the dearth of studies in the obstetrical literature. Propofol rapidly transfers across the placenta near term. In one study, 20 infants exposed to propofol during parturition had depressed Apgar scores at birth compared with unexposed controls, but the neurodepression rapidly reversed. Numerous other studies have failed to demonstrate any neonatal toxicity

21 Gastrointestinal Disease and Pregnancy

when propofol was administered during parturition; however, the safety of exposure in the first trimester has not been studied adequately. Naloxone

The FDA classifies naloxone as a category B drug during pregnancy. In subjects who are opiate-dependent, small doses of naloxone precipitate a syndrome resembling that produced by opiate withdrawal. Symptoms include restlessness, anxiety, insomnia, irritability, hyperalgesia, nausea, and muscle cramps. Because opiates cross the placenta, the administration of naloxone is dangerous and contraindicated in a pregnant patient who is specifically opiate dependent. The FDA-approved labeling for naloxone has the following precaution about use during pregnancy: “Naloxone should be used in pregnancy only if clearly needed.” The administration of naloxone is appropriate, however, for pregnant patients who have serious signs of potential meperidine toxicity, such as respiratory depression, systemic hypotension, or unresponsiveness.

233

Flumazenil

Flumazenil, a category C drug, is a benzodiazepine antagonist that rapidly reverses the central effects of benzodiazepines. Little is known about the safety of flumazenil during pregnancy or in infants; thus, it should be used during pregnancy only if the potential benefit clearly outweighs the risks. The need for flumazenil can be prevented by careful and slow titration of benzodiazepine dosage and by using the minimal benzodiazepine dosage required for endoscopic examination. Simethicone

The FDA has classified this agent as a category C drug. Many endoscopists use simethicone to reduce gastric foam before upper endoscopy. The Michigan Medicaid Study failed to show a statistically significant difference between pregnant women exposed to the drug and those not exposed to it. Simethicone use is low risk during endoscopy because it is not absorbed systemically.

Benzodiazepines

Glucagon

Diazepam and midazolam, category D drugs, should have restricted use during endoscopy, particularly during the first trimester. Benzodiazepines, including diazepam and midazolam, are commonly administered before gastrointestinal endoscopy to reduce anxiety, induce brief amnesia, and produce muscle relaxation. Diazepam freely and rapidly crosses the placenta and accumulates in the fetal circulation at levels equal to or higher than those of maternal serum. The FDAapproved labeling for diazepam carries the following warning about use in pregnancy: “An increased risk of congenital malformations associated with the use of minor tranquilizers during the first trimester of pregnancy has been suggested in several studies. Because use of these drugs is rarely a matter of urgency, their use during this period should almost always be avoided.” Diazepam is not recommended by the American Academy of Pediatrics because it, along with its metabolite N-demethyldiazepam, can accumulate in breast-fed infants. Many endoscopists prefer midazolam to diazepam for endoscopic premedication because of faster onset and recovery time, more intense transient antegrade amnesia, and lower risk of thrombophlebitis. Midazolam crosses the human placenta, but fetal serum levels increase to only about one-third to two-thirds of those of maternal serum after oral, intramuscular, or intravenous administration in the mother. Midazolam appears to be preferable to diazepam for endoscopy during pregnancy because of the potential association between diazepam and oral clefts and neonatal neurobehavioral abnormalities. Because the mechanism of action is similar to that of diazepam, midazolam should be used cautiously and in low doses during pregnancy, particularly during the first trimester. Dosages should be titrated carefully to an end point of relaxation and calmness but not somnolence.

The FDA classifies glucagon as a category B drug during pregnancy. However, no adequate and well-controlled studies have involved pregnant women. Thus, this drug should be used during pregnancy only if clearly needed. Although fetal risk has not been characterized completely, the administration of glucagon appears to be justified to decrease motility to help reduce procedure time and aid in cannulation of the bile duct and sphincterotomy during therapeutic ERCP, performed because of the high risk of untreated maternal cholangitis. The grounding pad should be positioned so that the uterus is not directly between the electrical catheter and grounding pad. Antibiotics

Ampicillin: The FDA classifies ampicillin as a category B drug during pregnancy. The American Heart Association recommends ampicillin as intravenous antibiotic prophylaxis for patients at high and medium risk of endocarditis (ie, artificial heart valves and certain congenital heart defects) who undergo ERCP in the presence of biliary obstruction. Ampicillin, a penicillin antibiotic, rapidly crosses the placenta, and fetal serum levels equilibrate with those of maternal serum within 3 hours after the drug is administered to the mother. Gentamicin: This FDA category C drug is recommended by the American Heart Association as part of a prophylactic antibiotic regimen for patients at high and medium risk of endocarditis who undergo ERCP in the presence of biliary obstruction. Gentamicin, an aminoglycoside antibiotic, rapidly crosses the placenta, and fetal serum levels peak at about one-half those of maternal levels after administration. However, the drug should be administered if required to treat known biliary sepsis.

234

Section V Colon

Colonic Lavage Preparations

clinical setting, the risk of maternal cholangitis will likely outweigh the theoretical risk of transient fetal hypothyroidism.

Polyethylene glycol electrolyte solution has not been studied extensively in pregnancy, and it is not known whether it can cause fetal harm. A study of 225 patients demonstrated the agent was safe when used to treat constipation. Because full colonoscopy rarely is indicated during pregnancy, tap water enemas are recommended as bowel preparation for lower endoscopy. Lidocaine

Lidocaine, an FDA category B drug, often is applied topically to the oropharynx before upper endoscopy and ERCP. No fetal harm was noted during parturition in the Collaborative Perinatal Project, in which 293 infants were exposed in the first trimester. The pregnant patient who is administered topical lidocaine should be instructed to gargle and spit out, rather than swallow, the preparation to minimize systemic absorption. Therapeutic Agents for Hemostasis

Epinephrine is injected during endoscopy to achieve hemostasis of actively bleeding lesions. During therapeutic endoscopy, it is used to stop active bleeding, and in this clinical scenario, the benefit outweighs the potential risk of its use. Electricity is transferred readily across the uterus because amniotic fluid is an excellent conductor. Fetal risk depends on the voltage and current amplitude, duration, and frequency time as well as the location on the body. Fetal mortality is rare from electroconvulsive therapy or direct current cardioversion during pregnancy. During endoscopy, cap electrocautery should be used because no grounding pad is necessary. Contrast Dye

Diatrizoate, a contrast agent injected into the biliary tree, has been used in diagnostic and therapeutic amniography without harming the fetus. Although it has been documented to impair fetal thyroid function and is categorized as a class D medication, the risk of its use for cholangiography is less than for amniography because of the doses used. In the appropriate

Nausea and Vomiting

Nausea and vomiting are extremely common during pregnancy and have multiple causes. Most women can be supported through episodes without the use of antiemetics. However, for women who have a protracted course or underlying conditions that may predispose to nausea and vomiting, medical therapy is warranted to prevent complications from volume depletion. The use of antiemetic medications during pregnancy is summarized in Table 21.3. Metoclopramide

The use of metoclopramide as an antiemetic usually is confined to the first trimester, but it also is used to enhance gastric emptying throughout pregnancy. A Danish study compared, over a 5-year period, 309 women with singleton pregnancies and prescriptions for metoclopramide with 13,327 controls and reported no major differences in the risk for malformations (odds ratio [OR] 1.11, 0.6–2.1), low birth weight (OR 1.79, 0.8–3.9), or preterm delivery (OR 1.02, 0.6–1.7). Prochlorperazine

Prochlorperazine, a category C drug, readily crosses the placenta. However, most studies have not found an increased risk of adverse outcomes in pregnancy. Promethazine

Promethazine is also a category C drug. It is an antihistamine that is used occasionally as an antiemetic during pregnancy and also as adjunctive therapy for narcotics during labor. Trimethobenzamide

Trimethobenzamide is a category C drug. Three studies have followed outcomes in women who took trimethobenzamide

Table 21.3 Medications Used for Nausea and Vomiting Drug

FDA Category

Recommendations for Pregnancy

Recommendations for Breastfeeding

No human data: probably compatible Limited human data: probably compatible No human data: probably compatible Limited human data: potential toxicity No human data: probably compatible No human data: potential toxicity No human data: probably compatible No human data: probably compatible

Dolasetron Domperidone

B C

No human studies Safety unknown

Granisetron Metoclopramide Ondansetron Prochlorperazine Promethazine Trimethobenzamide

B B B C C C

No human studies No teratogenicity, low risk (population-based study) No teratogenicity, low risk (controlled trial) No teratogenicity, low risk (large database study) No teratogenicity, low risk (large database study) No teratogenicity, low risk (case series)

21 Gastrointestinal Disease and Pregnancy

during their first trimester for nausea and vomiting. In all three studies, there was no increase in the incidence of malformations with the use of trimethobenzamide.

235

up to 146 times the human dose did not demonstrate any adverse outcomes. Domperidone

Ondansetron

Ondansetron, a category B drug, is used for the prevention and treatment of chemotherapy-induced nausea and vomiting and for hyperemesis gravidarum. A randomized, double-blind study compared intravenous ondansetron with promethazine for hyperemesis. Ondansetron was well tolerated and efficacious with no adverse effects; however, infant outcomes were not reported by the study. Results from the Teratogen Information Services database do not demonstrate an increase in major malformations in comparison with exposure to other antiemetics or normal controls. Granisetron and Dolasetron

Both of these agents are category B drugs. There have not been any studies on pregnant women exposed to these agents. However, studies of pregnant rats and rabbits that were given

Domperidone, a category C drug, is a dopamine antagonist used for short-term treatment of nausea and vomiting and for its prokinetic properties. Currently, it is not available in the United States by prescription. Whether it crosses the placenta is not known, but its bioavailability after oral ingestion is low.

Gastroesophageal Reflux Disease

Heartburn is estimated to occur in 30% to 50% of pregnancies. For mild symptoms, only lifestyle and dietary modifications may be required. Medications for treating gastroesophageal reflux disease have not been tested routinely in randomized, controlled trials in pregnant women. The medications used to treat gastroesophageal reflux disease and peptic ulcer disease are summarized in Table 21.4.

Table 21.4 Medications Used for Gastroesophageal Reflux and Peptic Ulcer Disease Drug

FDA Category

Recommendations for Pregnancy

Aluminum-containing Calcium-containing Magnesium-containing Magnesium trisilicates Sodium bicarbonate

None None None None None

Most safe: minimal absorption Most safe: minimal absorption Most safe: minimal absorption Avoid long term or high doses Not safe: alkalosis

Sucralfate

B

Safe

Recommendations for Breastfeeding

Antacids

Low risk Low risk Low risk Low risk Low risk

Mucosal protectants

No human data: probably compatible

Histamine2 receptor antagonists

Cimetidine Famotidine Nizatidine

B B B

Ranitidine

B

Controlled data: low risk Paucity of safety data Limited human data (low risk in animals) Low risk

Compatible Limited human data: probably compatible Limited human data: probably compatible Limited human data: probably compatible

Proton pump inhibitors

Esomeprazole Lansoprazole Omeprazole Pantoprazole Rabeprazole

B B C B B

Limited data: low risk Limited data: low risk Embryonic and fetal toxicity Limited data: low risk Limited data: low risk

Cisapride

C

Metoclopromide

B

Controlled study: low risk, limited availability Low risk

No human data: potential toxicity No human data: potential toxicity Limited human data: potential toxicity No human data: potential toxicity No human data: potential toxicity

Promotility agents

Limited human data: probably compatible Limited human data: potential toxicity

Treatment of Helicobacter pylori

Amoxicillin Bismuth Clarithromycin Metronidazole Tetracycline

B C C B D

Safe Not safe: teratogenicity Avoid in 1st trimester Low risk, avoid in 1st trimester Not safe: teratogenicity

Compatible No human data: potential toxicity No human data: probably compatible Limited human data: potential toxicity Compatible

236

Section V Colon

Antacids

Antacids that contain magnesium, aluminum, or calcium are not teratogenic in animal studies. Although one case-control study reported a significant increase in major and minor congenital abnormalities in infants exposed to antacids during the first trimester of pregnancy, no analysis of individual agents was done and, currently, most antacids at normal therapeutic doses are considered acceptable during pregnancy. Magnesium trisilicate, found in alginic acid, can lead to fetal nephrolithiasis, hypotonia, and respiratory distress if used long term and in high doses. Antacids containing sodium bicarbonate should not be used because they can cause maternal or fetal metabolic alkalosis and fluid overload. Excessive intake of calcium carbonate can result in milk-alkali syndrome, characterized by hypercalcemia, renal impairment, and metabolic alkalosis. None of the antacids have been shown to concentrate in breast milk and are acceptable when nursing.

dose, reported more abortions, lower fetal weight, and fewer live fetuses than controls. Another study showed a higher rate of abortions in rabbits treated with large doses of the drug. Promotility Agents Metoclopramide

Metoclopramide is a category B drug. No congenital malformations or other neonatal toxicities have been reported in humans with the use of metoclopramide. Reproductive studies in mice, rats, and rabbits that received up to 250 times the recommended human dose have not demonstrated any increase in fetal toxicity. Metoclopramide has been used as a lactation stimulant, and the total daily dose that would be consumed by a nursing infant if the mother took 30 mg/day is much less than the maximum daily dose of 500 μg/kg recommend for infants. Therefore, maternal doses of 45 mg or less per day should not have adverse effects on a nursing infant.

Sucralfate Cisapride

Sucralfate, a category B drug, is a nonabsorbable drug that exerts a local rather than systemic effect and has been tested in a prospective randomized controlled trial. Women with gastroesophageal reflux disease treated with sucralfate had a higher frequency of symptomatic remission than controls (90% vs 43%, P30 days). Although most episodes of diarrhea are self-limited and treatment is not required, certain pathogens require treatment. The common medications used to treat infectious diarrhea are summarized in Table 21.7. Albendazole

Albendazole, a category C drug, is used in the treatment of microsporidia infection, cysticercosis, helminth infection, and hydatid disease. The drug is embryotoxic and teratogenic (skeletal malformations) in rats and rabbits. Human data are limited. Albendazole therapy for the eradication of helminths during pregnancy is associated with significantly less maternal anemia and no increase in adverse pregnancy outcomes,

Azithromycin

Azithromycin, a macrolide antibiotic, is pregnancy category B and is a second-line treatment for cryptosporidia and Entamoeba histolytica infections. A study of 20 women who received the drug for Chlamydia trachomatis infection noted that 40% complained of moderate to severe gastrointestinal side effects. A trial of 94 pregnant women with Trichomonas vaginalis treated with a combination of azithromycin, cefixime, and metronidazole demonstrated increased rates of infant low birth weight, preterm birth, and 2-year mortality compared to the children of 112 infected mothers who were not treated for the same infection. Doxycycline and Tetracycline

Doxycycline and tetracycline are both category D drugs. Doxycycline is used as second-line treatment for infections with Vibrio cholera, Campylobacter, and enterotoxigenic Escherichia coli. Similar to tetracycline, this class of medications

Table 21.7 Medications Used for the Treatment of Infectious Diarrhea Drug

FDA Category

Recommendations for Pregnancy

Recommendations for Breastfeeding

Embryotoxic in animals Avoid in 1st trimester Human data support improved pregnancy outcomes with helminth eradication Safe Low risk Potential toxicity to cartilage: avoid

No human data: probably compatible

Compatible No human data: potential toxicity

Albendazole

C

Ampicillin Azithromycin Ciprofloxacin (all quinolones) Doxycycline Furazolidone

B B C D C

Metronidazole

B

Rifaximin

C

Tetracycline Tinidazole

D C

Trimethoprimsulfamethoxazole Vancomycin

C

Contraindicated: teratogenic Low risk Limited data Low risk: question of increased risk of cleft lip/palate Animal teratogen No human data Not safe: teratogenicity Low risk Limited data Teratogenic

C

Low risk

Compatible Limited human data: probably compatible Limited human data: probably compatible

Limited human data: potential toxicity No human data: probably compatible Compatible Unsafe Compatible Limited human data: probably compatible

244

Section V Colon

crosses the placenta and is bound by chelating with calcium in developing bone and teeth. This results in discoloration of the teeth, hypoplasia of enamel, and inhibition of skeletal growth. A population-based study found a higher rate of congenital anomalies in the infants of mothers who used doxycycline during pregnancy; however, the case-control pair analysis did not show a significantly higher rate of doxycycline treatment in the second and third months of gestation in any group of congenital abnormalities. Furazolidone

Furazolidone, a category C drug, is second-line treatment for giardiasis. Few data are available about its safety in pregnancy, but the Collaborative Perinatal Project monitored 50,282 mother-child pairs and 132 had exposure to furazolidone in the first trimester, with no association with congenital malformations. Metronidazole

Metronidazole, a category B drug, is used to treat Clostridium difficile infection, amebiasis, and giardiasis. Multiple studies have suggested that exposure to metronidazole prenatally is not associated with birth defects. These studies include two meta-analyses, two retrospective cohort studies, and a prospective controlled study of 228 women exposed to metronidazole during pregnancy. A population-based case-control study found that overall teratogenic risk was low, but infants of women exposed to metronidazole in the second to third months of pregnancy had higher rates of cleft lip, with or without cleft palate. This increase was slight and not thought to be clinically significant. Quinolones

Quinolones (eg, ciprofloxacin, levofloxacin, and norfloxacin), category C drugs, are used to treat infections with Shigella, Campylobacter, Yersinia, entertoxigenic and enteroinvasive E coli, and Vibrio cholera. Quinolones have a high affinity for bone tissue and cartilage and may cause arthropathies in children. The manufacturer reports damage to cartilage in weightbearing joints after quinolone exposure in immature rats and dogs. However, a prospective controlled study of 200 women exposed to quinolones and a population-based cohort study of 57 women exposed to quinolones did not find an increased risk of congenital malformations. Overall, the risk is thought to be minimal, but because safer alternatives are available, quinolones should be avoided in pregnancy.

complications in rats and rabbits. However, other studies have noted teratogenicity in rats and rabbits, including cleft palate and incomplete ossification. Tinidazole

Tinidazole, a category C drug, is a second-line treatment for giardiasis and amebiasis. Placental transfer of tinidazole occurs early in pregnancy, raising concern about its use in the first trimester. A population-based study from Hungary did not note an increased rate of congenital malformations when the drug was administered during pregnancy; however, the number of women treated with tinidazole was small. Trimethoprim-Sulfamethoxazole

Trimethoprim-sulfamethoxazole, a category C drug, is firstline treatment for infections with Isospora and Cyclospora and second-line treatment of infections with Shigella, Yersinia, and enterotoxigenic E coli. Trimethoprim has antifolate effects, increasing the potential of congenital anomalies. A study of 2,296 Michigan Medicaid recipients with first-trimester exposure to trimethoprim noted an increased risk of birth defects, particularly cardiovascular defects. A population-based casecontrol study in Hungary noted a higher rate of multiple congenital anomalies and cardiovascular malformations. On the basis of these data, trimethoprim-sulfamethoxazole should be avoided in pregnancy. Vancomycin

Vancomycin, a category C drug, is used in the treatment of C difficile colitis refractory to treatment with metronidazole. Studies in rats and rabbits have not demonstrated teratogenic effects. No cases of congenital defects attributable to vancomycin have been located, and the drug is considered low risk in pregnancy.

Inflammatory Bowel Disease

For patients with Crohn’s disease and ulcerative colitis, disease activity at the time of conception can be associated with a higher risk of spontaneous abortion and disease activity during the course of pregnancy can be associated with higher rates of low birth weight and premature infants. It is advisable that patients be in remission when considering pregnancy, and for the majority, this requires continuing the medications. Medications used to treat inflammatory bowel disease are summarized in Table 21.8.

Rifaximin Aminosalicylates

Rifaximin, a category C drug, is used to treat traveler’s diarrhea. Little information exists about its safety in pregnancy. Rifaximin has not been found to affect fertility or pregnancy outcome in rats or, in one study, to cause teratogenic

All aminosalicylates (sulfasalazine, mesalamine, and balsalazide) are category B drugs except olsalazine, which is category C. A population-based study using the Hungarian Case

21 Gastrointestinal Disease and Pregnancy

245

Table 21.8 Medications Used in the Treatment of Inflammatory Bowel Disease (IBD) Drug

FDA Category

Adalimumab Amoxicillin–clavulinic acid Azathioprine/6mercaptopurine Balsalazide Certolizumab pegol Ciprofloxacin Corticosteroids

B B D

Cyclosporine Fish oil supplements

C –

Infliximab Mesalamine Methotrexate Metronidazole

B B X B

Natalizumab Olsalazine Rifaximin

B C C

Sulfasalazine

B

Tacrolimus Thalidomide

C X

B B C C

Recommendations for Pregnancy

Recommendations for Breastfeeding

Limited human data: low risk Low risk Data in IBD & transplant literature suggest low risk Low risk Low risk Avoid: potential toxicity to cartilage Low risk: possible increased risk of cleft palate, adrenal insufficiency, premature rupture of membranes Low risk Safe Possible benefit Low risk Low risk Contraindicated: teratogenic With limited efficacy in IBD & risk of cleft palate, would avoid Low risk Low risk Animal teratogen No human data Considered safe Give folate 2 mg/d Use if mother’s health mandates Contraindicated: teratogenic

No human data: probably compatible Probably compatible No human data: potential toxicity

Control Surveillance of Congenital Abnormalities database did not find a significant increase in the prevalence of congenital abnormalities in the children of women treated with sulfasalazine. Because of concern about potential antifolate effects of sulfasalazine, it is recommended that women take folic acid 1 mg twice daily in the prenatal period and throughout pregnancy. Unlike with other sulfonamides, bilirubin displacement and, thus, kernicterus do not occur in the infant. Sulfasalazine has been clearly associated with infertility in men. Abnormalities in sperm number, motility, and morphology have been noted. This effect appears to be reversible: when mesalamine was substituted for sulfasalazine, semen quality returned to normal. An association has been described between sulfasalazine use in the parent and congenital malformations in the progeny. Because the lifespan of sperm is 120 days, men considering conception should either stop taking sulfasalazine or switch to mesalamine at least 3 months before attempting conception. Case series of mesalamine use in pregnancy do not suggest an increased risk to the fetus. This has been supported by a prospective controlled trial of 165 women exposed to mesalamine who were compared with matched controls with no exposure and by a population-based cohort study from Denmark.

No human data: potential diarrhea No human data Limited human data: probably compatible Compatible

Limited human data: potential toxicity No human data No human data: probably compatible Limited human data: potential diarrhea Contraindicated Limited human data: potential toxicity No human data Limited human data: potential diarrhea No human data: probably compatible Limited human data: potential diarrhea Limited human data: potential toxicity No human data: potential toxicity

Antibiotics

Metronidazole, the quinolones, and rifaximin are covered in the section on infectious diarrhea. Because of the limited evidence of the effectiveness of these agents in treating inflammatory bowel disease and the extended duration of use in the treatment of Crohn’s disease and ulcerative colitis, these drugs should be avoided during pregnancy. Short courses for the treatment of pouchitis can be considered on the basis of the safety data presented above. An alternative antibiotic for pouchitis is amoxicillin-clavulanic acid, a category B drug. Corticosteroids

Corticosteroids are category C drugs. A case-control study of corticosteroid use during the first trimester of pregnancy noted an increased risk of oral clefts in the newborn. This was confirmed by a large case-control study and a meta-analysis, which reported a summary OR for case-control studies examining the risk of oral clefts (OR 3.35 [95% CI 1.97, 5.69]). However, the population was treated for various conditions, and the overall risk of major malformations was low (OR 1.45 [95% CI 0.80, 2.60]). A prospective controlled study of 311 women who received glucocorticosteroids during the first

246

Section V Colon

trimester did not note an increased rate of major anomalies, and cases of oral cleft were not noted. The study was powered to find a 2.5-fold increase in the overall rate of major anomalies. An increased risk of premature rupture of membranes and adrenal insufficiency in the newborn has been reported in the transplant setting. Overall, the use of corticosteroids poses a small risk to the developing infant, and the mother needs to be informed about both the benefits and risks of therapy. A small retrospective review of patients with inflammatory bowel disease treated with budesonide during pregnancy did not document congenital malformations or an increase in adverse outcomes. Bisphosphonates

The bisphosphonates, alendronate and risedronate, are category C drugs. Their safety in breastfeeding is not known. For many patients, treatment of inflammatory bowel disease starts with these medications in conjunction with corticosteroids to prevent bone loss. Both agents should be avoided in pregnancy because animal studies have shown that alendronate crosses the placenta and is stored in fetal bone, causing anatomical changes. The effects on human fetal bone development are not known. The half-life of alendronate is longer than 10 years, and it accumulates in bone. The concern with giving this agent to a woman of child-bearing potential is that the drug is released slowly from bone and may produce low-level, continuous exposure to the fetus throughout gestation. Risedronate has a half-life of 20 days and may be the better choice for a woman contemplating future pregnancy. However, five half-lives of the drug are needed to excrete 97% of it, so women should wait at least 100 days from the last dose before considering conception. Immunomodulators

The immunomodulators are the most controversial agents used to treat inflammatory bowel disease in pregnant women. Methotrexate

Methotrexate, a category X drug, is clearly teratogenic and should not be administered to women or men who are considering conception. Methotrexate is a folic acid antagonist, and its use during the critical period of organogenesis (6–8 weeks postconception) is associated with multiple congenital anomalies collectively called methotrexate embryopathy or fetal aminopterin-methotrexate syndrome. This syndrome is characterized by intrauterine growth retardation; decreased ossification of the calvarium; hypoplastic supraorbital ridges; small, low-set ears; micrognathia; limb abnormalities; and, occasionally, mental retardation. Exposure in the second and third trimesters may be associated with fetal toxicity and mortality. Methotrexate may cause reversible oligospermia in men. No case reports have been published on congenital anomalies occurring in the offspring of men receiving methotrexate therapy. Methotrexate may persist in tissues for long periods,

and it has been suggested that patients wait at least 3 to 6 months after discontinuation of treatment with the drug before attempting conception. Azathioprine and 6-Mercaptopurine

6-Mercaptopurine and its prodrug azathioprine are category D drugs. Animal studies have demonstrated teratogenicity, with increased frequencies of cleft palate and open-eye and skeletal anomalies in mice exposed to azathioprine and cleft palate and skeletal and urogenital anomalies in rats. Transplacental and transamniotic transmission of azathioprine and its metabolites from the mother to the fetus can occur. The oral bioavailability of azathioprine (47%) and 6-mercaptopurine (16%) is low, and the early fetal liver lacks the enzyme inosinate pyrophosphorylase needed to convert azathioprine to 6-mercaptopurine. Both features may protect the fetus from toxic drug exposure during the crucial period of organogenesis. The largest evidence on safety comes from transplantation studies, in which rates of anomalies ranged from zero to 11.8%, and no evidence emerged for recurrent patterns of congenital anomalies. Multiple case series of inflammatory bowel disease have not noted an increase in congenital anomalies. On the basis of the large experience in transplantation patients and the body of evidence in inflammatory bowel disease, the drugs are often continued during pregnancy to maintain the mother in remission. A flare of disease during pregnancy may be more deleterious to neonatal outcome than any potential risk from the medication. Cyclosporine and Tacrolimus

These agents are considered in the section on liver transplantation. For severe steroid-refractory ulcerative colitis, cyclosporine may be a better treatment option than colectomy, which is associated with a considerable morbidity rate for mother and fetus. Thalidomide

Thalidomide, a category X drug, has some anti-tumor necrosis factor effects and has been used successfully for the treatment of Crohn’s disease. However, its teratogenicity has been documented extensively and includes limb defects, central nervous system effects, and abnormalities of the respiratory, cardiovascular, gastrointestinal, and genitourinary systems. Thalidomide is contraindicated during pregnancy and for women of childbearing age who are not using two reliable methods of contraception for 1 month before starting therapy, during therapy, and for 1 month after stopping therapy. Biologic Therapy Infliximab

Infliximab, a category B drug, is used in the management of Crohn’s disease and ulcerative colitis. A growing body of

21 Gastrointestinal Disease and Pregnancy

evidence suggests that infliximab is low risk in pregnancy. The two largest studies are from the TREAT registry and the Infliximab Safety Database maintained by Centocor (Malvern, PA). The TREAT registry is a prospective registry of patients with Crohn’s disease, and patients may or may not be treated with infliximab. Sixty-six pregnancies were reported among the 5,807 patients enrolled, and 36 of the 66 had prior exposure to infliximab. Fetal malformations have not occurred in any of the pregnancies. The rates of miscarriage (11.1% vs 7.1%, P=.53) and neonatal complications (8.3% vs 7.1%, P=.78) are not significantly different between infliximabtreated and infliximab-naïve patients, respectively. In the Infliximab Safety Database, a retrospective data collection, pregnancy outcome data are available for 96 women who had direct exposure to infliximab. The 96 pregnancies resulted in 100 births. The expected versus observed outcomes for women exposed to infliximab were not different from those of the general population. In a series of 10 women who received maintenance therapy with infliximab throughout pregnancy, all 10 pregnancies ended in live births and no congenital malformations were reported. Infliximab probably crosses the placenta, beginning at approximately week 20. Currently, if maternal health warrants infliximab therapy, it is continued through pregnancy. Adalimumab

Adalimumab, a category B drug, has been shown recently to be safe and effective for the induction of remission in Crohn’s disease. A case report has documented a successful pregnancy in a woman with long-standing Crohn’s disease who began treatment with adalimumab 1 month before conception and received a total of 38 doses during her pregnancy. It is unclear if the drug crosses the placenta but is assumed to do so, starting at about week 20. Adalimumab therapy during pregnancy is warranted for active disease or maintenance of remission. Certolizumab Pegol

Certolizumab is a pegylated Fab′ fragment of IgG1 antibody against tumor necrosis factor-α. Early data suggest that negligible amounts occur in cord blood of infants born to mothers treated with this agent during pregnancy. Because of its small molecular size, certolizumab pegol may have an advantage for use in pregnancy. Certolizumab pegol therapy is warranted during pregnancy for active inflammatory bowel disease or maintenance of remission. Natalizumab

Natalizumab is a monoclonal antibody of the IgG4 class directed against alpha integrins. It carries an FDA category B rating. On the basis of animal and trial data, it is listed as a category B drug. Few data are available about its use in pregnancy in the postmarketing era. Currently, there are no firm

247

recommendations about its use treating inflammatory bowel disease in pregnant women. Fish Oil Supplements

Many patients who have inflammatory bowel disease take fish oil supplements as an adjunct to standard medical therapy. Because fish oil is a supplement and not a drug, it is not rated by the FDA. A randomized controlled trial of fish oil supplementation demonstrated a prolongation of pregnancy without detrimental effects on the growth of the fetus or the course of labor. Fish oil supplementation may help prevent miscarriage associated with the antiphospholipid antibody syndrome. In women with inflammatory bowel disease who may be at increased risk for preterm birth and miscarriage, fish oil supplementation is not harmful and may be of some benefit. SUGGESTED READING

American Academy of Pediatrics Committee on Drugs. Transfer of drugs and other chemicals into human milk. Pediatrics. 2001 Sep;108(3):776–89. Antiretroviral Pregnancy Registry Steering Committee. Antiretroviral Pregnancy Registry International Interium Report for January 1, 1989 through January 31, 2010. Wilmington (NC): Registry Coordinating Center; c2010. Available from http://www. APRegistry.com. Armenti VT, Radomski JS, Moritz MJ, Gaughan WJ, McGrory CH, Coscia LA. Report from the National Transplantation Pregnancy Registry (NTPR): outcomes of pregnancy after transplantation. Clin Transpl. 2003:131–41. Briggs GG, Freeman RK, Yaffe SJ. Drugs in pregnancy and lactation. 7th ed. Philadelphia (PA): Lippincott Williams & Wilkins; c2005. Cappell MS, Colon VJ, Sidhom OA. A study at 10 medical centers of the safety and efficacy of 48 flexible sigmoidoscopies and 8 colonoscopies during pregnancy with follow-up of fetal outcome and with comparison to control groups. Dig Dis Sci. 1996 Dec;41(12):2353–61. Cappell MS, Colon VJ, Sidhom OA. A study of eight medical centers of the safety and clinical efficacy of esophagogastroduodenoscopy in 83 pregnant females with follow-up of fetal outcome with comparison control groups. Am J Gastroenterol. 1996 Feb;91(2):348–54. Dailland P, Cockshott ID, Lirzin JD, Jacquinot P, Jorrot JC, Devery J, et al. Intravenous propofol during cesarean section: placental transfer, concentrations in breast milk, and neonatal effects: a preliminary study. Anesthesiology. 1989 Dec;71(6):827–34. Diav-Citrin O, Arnon J, Shechtman S, Schaefer C, van Tonningen MR, Clementi M, et al. The safety of proton pump inhibitors in pregnancy: a multicentre prospective controlled study. Aliment Pharmacol Ther. 2005 Feb 1;21(3):269–75. Diav-Citrin O, Shechtman S, Gotteiner T, Arnon J, Ornoy A. Pregnancy outcome after gestational exposure to metronidazole: a prospective controlled cohort study. Teratology. 2001 May; 63(5):186–92. Garbis H, Elefant E, Diav-Citrin O, Mastroiacovo P, Schaefer C, Vial T, et al. Pregnancy outcome after exposure to ranitidine and other H2-blockers: a collaborative study of the European Network

248

Section V Colon

of Teratology Information Services. Reprod Toxicol. 2005 MarApr;19(4):453–8. Gossard AA, Lindor KD. Pregnancy in a patient with primary sclerosing cholangitis. J Clin Gastroenterol. 2002 Oct;35(4):353–5. Hasler WL. The irritable bowel syndrome during pregnancy. Gastroenterol Clin North Am. 2003 Mar;32(1):385–406. Lu EJ, Curet MJ, El-Sayed YY, Kirkwood KS. Medical versus surgical management of biliary tract disease in pregnancy. Am J Surg. 2004 Dec;188(6):755–9. Mahadevan U, Kane S. American Gastroenterological Association Institute technical review on the use of gastrointestinal medications in pregnancy. Gastroenterology. 2006 Jul;131(1):283–311.

Ramin KD, Ramsey PS. Disease of the gallbladder and pancreas in pregnancy. Obstet Gynecol Clin North Am. 2001 Sep;28(3): 571–80. Su GG, Pan KH, Zhao NF, Fang SH, Yang DH, Zhou Y. Efficacy and safety of lamivudine treatment for chronic hepatitis B in pregnancy. World J Gastroenterol. 2004 Mar 15;10(6):910–2. Tytgat GN, Heading RC, Muller-Lissner S, Kamm MA, Scholmerich J, Berstad A, et al. Contemporary understanding and management of reflux and constipation in the general population and pregnancy: a consensus meeting. Aliment Pharmacol Ther. 2003 Aug 1;18(3):291–301.

Colon Questions and Answers

Questions Abbreviations used:

ALT, alanine aminotransferase ASCA, anti-Saccharomyces cerevisiae antibody CT, computed tomography HCV, hepatitis C virus MRI, magnetic resonance imaging NSAID, nonsteroidal antiinflammatory drug pANCA, perinuclear antineutrophil cytoplasmic antibody TPMT, thiopurine methyltransferase TNF, tumor necrosis factor TSH, thyroid-stimulating hormone Multiple Choice (choose the best answer)

V.1. A 29-year-old man with a 5-year history of Crohn’s disease presents for evaluation. He has a history of disease involvement of both the ileum and right colon. He has not had any previous surgical treatment. He now has fever, nausea, right lower quadrant pain, and four to six loose bowel movements daily. Which test would not be appropriate at this time? a. CT enterography b. Colonoscopy c. C-reactive protein d. Capsule endoscopy e. Stool testing for Clostridium difficile V.2. A 24-year-old woman presents for evaluation of chronic diarrhea. She has had two to five loose, bloody bowel movements daily for the last 8 weeks. She has not recently traveled, had contact with sick persons, or taken NSAIDs. Stool microbial assessment is negative. Colonoscopy shows inflammation in the cecum (periappendiceal) and normal ascending colon, transverse

colon, and descending colon, with mild inflammation in the sigmoid and rectum. Biopsy specimens from the inflamed regions show mild chronic active colitis. The patient is pANCA-positive, ASCA-negative. What is the most likely diagnosis? a. Crohn’s colitis b. Undisclosed NSAID usage c. Ischemic colitis d. Ulcerative colitis e. Infectious colitis V.3. A 41-year-old man is a new patient who has recently moved because of a job change. When he was 27 years old, chronic ulcerative colitis was diagnosed. He has done well with mesalamine (Asacol) therapy, 2.4 g daily in divided dosing. Colonoscopy performed 4 years ago showed only quiescent disease. There is no personal or family history of colon cancer or polyps. Which of the following should be recommended? a. Colonoscopy with surveillance biopsies every 1 to 2 years starting now b. Colonoscopy with surveillance biopsies every 1 to 2 years starting 5 years after the last endoscopic examination c. Follow-up colonoscopy only if symptoms flare d. Colonoscopy with surveillance biopsies every 3 to 5 years starting now e. Colonoscopy with surveillance biopsies every 3 to 5 years starting 5 years after the last endoscopic examination V.4. A 19-year-old woman with recently diagnosed Crohn’s disease is referred by her primary care physician. She has been researching the disease on the Internet and has multiple questions. Which of following statements is true? a. Patients with Crohn’s disease are not at increased risk for colorectal cancer 249

250

Section V Colon

b. Inflammatory bowel disease is more common in warm climates c. Smoking cigarettes increases the risk of symptomatic recurrence of Crohn’s disease d. Sulfasalazine has been linked to irreversible infertility in men e. Inflammatory bowel disease is more common in developing countries V.5. A 30-year-old woman with established Crohn’s colitis returns after new symptoms develop. Two weeks ago, perianal pain developed and she thought it was related to her hemorrhoids. One week ago, a small “pimple” developed near the anus that was painful to touch, until it began draining cloudy fluid 2 days ago. She also has noted air bubbles in her urine over the last month. Perianal examination shows multiple external skin tags without any areas of fluctuance, tenderness, or obvious fistulous openings. Which test is most likely to establish the diagnosis? a. CT pelvis b. MRI pelvis c. CT enterography d. Colonoscopy e. Capsule endoscopy V.6. A 35-year-old man with Crohn’s disease of the terminal ileum and right colon comes for medical care after moving from another city. Currently, he takes 15 mg of prednisone daily, which controls his symptoms. He states that when he gets below 5 mg daily, he has abdominal pain, diarrhea, and arthralgias. The joint symptoms, more than the diarrhea, appear to limit his function at work. What is an appropriate therapeutic option for this patient? a. Start azathioprine at 5 mg/kg after checking the TPMT level b. Start treatment with methotrexate at 25 mg subcutaneously every week, with a steroid taper c. Add a cyclooxygenase-2 inhibitor for joint pains d. Start high-dose 5-aminosalicylic acid for steroid sparing e. Add budesonide and taper the prednisone dose V.7. A 42-year-old man with a history of steroid-dependent Crohn’s disease has been receiving infliximab therapy for 3 years. Slowly, he has required interval changes, and now he is receiving 10 mg/kg every 6 weeks. He complains that the symptoms have returned after 5 weeks of receiving an infusion. What is the next best step? a. Add azathioprine after checking the TPMT level b. Decrease the dosing interval to every 5 weeks c. Begin premedication with diphenhydramine and acetaminophen d. Change to another anti-TNF agent e. Change to natalizumab

V.8. A 27-year-old man with a 2-year history of left-sided ulcerative colitis is hospitalized for a flare of disease. He has been receiving corticosteroids intravenously for 3 days, without much of a therapeutic response. The most appropriate therapeutic plan at this point is: a. Continue intravenous corticosteroid therapy for the next 5 days b. Start cyclosporine at 2 mg/kg intravenously c. Start cyclosporine at 5 mg/kg intravenously d. Start infliximab at 10 mg/kg intravenously e. Start azathioprine at 5 mg/kg orally V.9. A 30-year-old man with a history of refractory Crohn’s disease of the small intestine undergoes laparoscopic resection of 10 cm of the terminal ileum and right colon, with a primary anastomosis. He has received prednisone therapy in the past but not treatment with immunomodulators. What is the next best step to prevent his disease from recurring? a. Cyclosporine at 2 mg/kg orally b. A low dose of 5-aminosalicylic acid c. 6-Mercaptopurine at 1 mg/kg after checking the TPMT level d. Long-term course of antibiotic therapy e. Methotrexate V.10. A 28-year-old man with a history of pan ulcerative colitis is prescribed 4.8 g mesalamine and has a good therapeutic response. He returns for follow-up 6 months later and reports having one formed bowel movement daily, without bleeding, urgency, or cramping. What is the next appropriate recommendation for maintenance therapy? a. Stop all medications because the patient’s disease is in remission b. Maintain treatment with 4.8 g mesalamine c. Change to azathioprine after checking the TPMT level d. Decrease the dose of mesalamine to 2.4 g/day e. Change regimen to nightly enema therapy V.11. A patient who has Crohn’s disease has been in clinical remission while receiving maintenance infliximab infusions for several years. Over the last several infliximab cycles, the patient has noted the development of arthralgias about 5 days after the infusion. These are becoming progressively more severe. Antinuclear antibodies are positive at 1 to 640. Tests for antibodies to double-stranded DNA and histones are negative. The most likely diagnosis is: a. Drug-induced lupus-like reaction to infliximab b. Incompletely treated Crohn’s disease with related spondyloarthropathy c. Delayed hypersensitivity reaction to infliximab d. Psoriatic arthritis e. Fibromyalgia

Questions and Answers

V.12. A 32-year-old man was diagnosed 6 months ago with extensive ulcerative colitis. He has no family history of colorectal cancer. No dysplastic changes or polyps were noted at initial colonoscopy. He is in clinical remission while receiving a moderate dose of oral mesalamine. He had been noted to have persistently increased serum levels of alkaline phosphatase. Magnetic resonance cholangiopancreatography shows alternate narrowing and dilatation of both the intrahepatic and extrahepatic bile ducts, consistent with primary sclerosing cholangitis. He should have his next colonoscopy in: a. Seven and one-half years b. Three years c. Six months d. Five years e. Nine and one-half years V.13. All the following are indications for bone mineral densitometry in patients with inflammatory bowel disease except: a. Postmenopausal state b. Morbid obesity c. More than 6 months of cumulative corticosteroid use d. Vitamin D deficiency e. Hypogonadism V.14. Which of the following statements about pyoderma gangrenosum in inflammatory bowel disease is correct? a. It typically starts as an erythematous tender lesion on the shins b. It is often an ulcer with undermining borders and exhibits pathergy c. Pyoderma gangrenosum has very characteristic, even pathognomonic, histologic features d. The clinical course of pyoderma gangrenosum always mirrors the course of the underlying inflammatory bowel disease e. Infliximab therapy is contraindicated for patients with pyoderma gangrenosum V.15. Treatment with sulfasalazine and 5-aminosalicylates has been associated with rare idiosyncratic reactions, including all the following except: a. Interstitial nephritis b. Pancreatitis c. Pneumonitis d. Pericarditis e. Collagenous colitis V.16. A 35-year-old man had a 48-hour history of vomiting and watery diarrhea. The symptoms began abruptly. He had eaten oysters at his favorite seafood restaurant 24 hours before the symptoms began. He has also had muscle aches and a mild headache. He believed he had a low-grade fever but did not document his temperature. On clinical examination, his

251

vital signs are normal and he appears well. Stool examination is negative for fecal leukocytes. What is the most likely cause of this man’s symptoms? a. Vibrio vulnificus b. Yersinia enterocolitica c. Rotavirus d. Norovirus e. Campylobacter V.17. A 50-year-old woman is evaluated for a 5-day history of diarrhea. She recently was treated for a urinary tract infection with ciprofloxacin. Two weeks later, diarrhea developed, and she was found to be positive for Clostridium difficile toxin. Her diarrhea resolved after a 10-day course of metronidazole, which was completed 1 week before the current presentation. The patient appears well clinically. Stool analysis for C difficile toxin is positive. What treatment would you recommend for this patient? a. Vancomycin b. Metronidazole c. Cholestyramine d. Rifaximin e. Saccharomyces boulardii V.18. A 75-year-old man has a 5-day history of watery diarrhea. He has not seen blood in the stool. He has had a low-grade fever of 38.1°C. He recently attended a family picnic at which he had eaten chicken and wondered if the chicken was undercooked. His past history is significant only for an asymptomatic aortic aneurysm that is being monitored. Physical examination findings are unremarkable. Stool culture indicates nontyphoid Salmonella. What would you recommend for this patient? a. Observation b. Intravenous fluids c. Intravenous antibiotics d. Oral antibiotics e. Strict isolation V.19. A 55-year-old man has a 5-day history of diarrhea that was watery initially but has become bloody in the past 24 hours. He became concerned with the appearance of blood. He has also experienced significant midabdominal cramping pain. He has had a low-grade fever, headache, and myalgia. The patient is otherwise well and takes no medications. Physical examination findings include normal vital signs and mild nonspecific periumbilical tenderness. Stool analysis shows the presence of multiple fecal leukocytes. What is the most likely cause of this patient’s symptoms? a. Escherichia coli O157:H7 b. Salmonella c. Campylobacter d. Rotavirus e. Staphylococcus aureus

252

Section V Colon

V.20. A 50-year-old man arrives at the emergency department with left lower abdominal pain. He has not had a fever or change in bowel habits. He is eating without difficulty. He has never had similar symptoms in the past. He has not undergone colon cancer screening. He has no medical comorbid conditions. On physical examination, mild tenderness is noted in the left lower abdomen, without peritoneal signs. Laboratory studies show a leukocyte count of 12.5×109/L. CT shows changes consistent with diverticulitis without abscess. What would you recommend as the next step for this patient? a. Hospital admission, bowel rest, intravenous antibiotics b. Outpatient antibiotics c. Colonoscopy d. Surgical consultation V.21.

A 35-year-old man presents with two high-grade cecal cancers without associated polyps. His brother at age 40 years, father at 60 years, and a paternal aunt at 52 years had colon cancer. Another paternal uncle had both stomach and ureteral cancer. The patient and his kindred would be at increased risk for each of the following cancers except: a. Breast b. Endometrial c. Sebaceous gland d. Ovarian e. Gastric

V.22. A 40-year-old adopted woman presented with fatigue. She is found to be iron deficient. Colonoscopy demonstrates multiple small, nonhemorrhagic juvenile polyps in both her stomach and colon. Menses recently have been heavier than her historical baseline. Except for thyroid cancer treated surgically 8 years earlier, she has not been followed by a physician. Physical examination showed a fibroma on her gum line and several small skin papillomas, which were found on histologic examination to be tricholemmomas. Each of the following represents appropriate management except: a. Genetic consultation and PTEN gene testing b. Careful gynecologic examination with endometrial ultrasonography c. Bilateral mammography d. Intensive gastroscopic and colonoscopic surveillance because of increased cancer risks e. Thyroid ultrasonography V.23. Life-threatening conditions associated with familial adenomatous polyposis include all the following except: a. Duodenal cancer b. Osteomas c. Thyroid cancer

d. Early childhood hepatoblastoma e. Colorectal cancer V.24. A 40-year-old woman with freckles on her lips and buccal mucosa comes from a family with known Peutz-Jeghers syndrome. She has not been evaluated previously. What type of the following cancers associated with Peutz-Jeghers syndrome would she be at highest risk for? a. Pancreas b. Colorectal c. Gastric d. Ovarian e. Breast V.25. A 46-year-old man is referred to you for a treatment recommendation. Six weeks ago, he had exploratory laparotomy for evaluation of chronic abdominal pain, postprandial abdominal bloating, and irregular bowel movements. The laparotomy findings were normal, and cholecystectomy and appendectomy were performed. A pathology examination of the gallbladder was normal, but the appendix had a 1-cm carcinoid tumor and no inflammation. Preoperatively, CT of the abdomen and pelvis showed no abnormality. Postoperatively, the patient’s symptoms improved, and colonoscopy showed a normal colon and terminal ileum. What is the most appropriate treatment recommendation? a. Exploratory laparotomy b. Laparoscopy c. Right hemicolectomy d. Octreotide e. Observation V.26. A 26-year-old woman presents with a 5-year history of abdominal pain. The pain is worst immediately before she has a loose, urgent bowel movement, which improves the pain. She has to do this several times each morning. The best recommendation is: a. Reassurance and symptom-based care b. Pelvic floor retraining c. Colectomy d. Psychiatry consultation e. A low-dose tricyclic agent V.27. A 31-year-old woman has constipation. She has a bowel movement every other day but with difficulty. She has to strain to get the stool out. She has bloating. Sometimes, she sees blood on the toilet paper. Findings on flexible sigmoidoscopy are negative. She has tried laxatives, but they only work if they cause her to have diarrhea. The best recommendation is: a. Reassurance and symptom-based care b. Pelvic floor retraining c. Colectomy d. Psychiatry consultation e. A low-dose tricyclic agent

Questions and Answers

V.28. A 45-year-old woman has constipation. She can go 2 weeks without a bowel movement and does not feel fullness or pain. She eventually becomes bloated and may pass a liquid stool. Otherwise, she feels well. She has no marked vomiting, and her weight is stable. She has tried several laxatives without success. She had anorectal manometry and was able to expel the balloon. Next, a whole-gut transit study was performed. At 2 hours, 54% had left the stomach and at 4 hours 90%. The geometric center at 4 hours was 1.3 and at 24 hours 1.5. The test was continued for 48 hours, and the geometric center was 1.8. The best recommendation is: a. Reassurance and symptom-based care b. Pelvic floor retraining c. Colectomy d. Tissue transglutaminase e. A low-dose tricyclic agent V.29. A 55-year-old woman presents with diarrhea and weight loss. She has diarrhea day and night and has lost 25 lb in weight. She has had a rash on her leg. She does not have abdominal pain or discomfort. Other members of her family have had a similar problem but did not seek medical attention for it. The best recommendation is: a. Reassurance and symptom-based care b. Pelvic floor retraining c. Colectomy d. Gluten-free diet e. A low-dose tricyclic agent V.30. A 38-year-old woman presents with abdominal pain not associated with the bowels or eating. She just has pain all the time. It never goes away. She notices that the pain might change when she moves or coughs. She has a history of constipation with some straining. On physical examination, she is tender over the rectus muscle. This tenderness worsens when she flexes her abdominal muscles. The best recommendation is: a. Reassurance and symptom-based care b. Pelvic floor retraining c. Colectomy d. Gluten-free diet e. Abdominal wall injection V.31. A 55-year-old woman has had constipation for 3 years. Despite straining excessively, applying manual abdominal pressure, and bending over, she requires 15 minutes or longer to evacuate stool from the rectum. She has abdominal bloating and uses enemas. Dietary fiber supplements have not been effective. Currently, she takes lubiprostone (8 mg twice daily) and milk of magnesia twice weekly, and small, semiformed liquid stools are expelled the day after she has milk of magnesia. Her appetite and

253

weight are stable, and she has no history of gastrointestinal tract bleeding. Physical examination findings are unremarkable except for the digital rectal examination, which disclosed normal anal resting tone, hesitant squeeze to voluntary command, and reduced perineal descent during simulated evacuation. Results of a complete blood count, routine chemistry panel, and sensitive TSH are normal. Colonoscopy results were normal 2 years ago. The most appropriate next step to evaluate this patient’s symptoms is: a. Repeat colonoscopy b. Assess colonic transit with radiopaque markers or scintigraphy and assess anorectal function with manometry c. Refer to a surgeon for subtotal colectomy d. Increase dietary fiber content V.32. Which of the following statements about the assessment of anorectal function in patients with chronic constipation is correct? a. Digital rectal examination is not useful for identifying defecatory disorders b. The rectal balloon expulsion test is highly sensitive and specific for diagnosing defecatory disorders c. Dyssynergia documented on anal manometry is diagnostic of a defecatory disorder d. Barium or magnetic resonance proctography has no role in the diagnosis of defecatory disorders V.33. Which of the following statements about colonic motor activity is inaccurate? a. Colonic motor activity increases after a meal b. High-amplitude propagated contractions occur an average of 5 times daily and often precede defecation c. The sympathetic nervous system stimulates colonic motor activity via adrenergic α1 receptors d. Rectal distention can induce colocolonic and colointestinal inhibitory reflexes, which can delay intestinal and colonic transit e. Serotonin induces peristalsis (ie, coordinated contraction proximal to and relaxation distal to the point of intestinal distention) V.34. Which of the following statements about the clinical features of chronic constipation is inaccurate? a. Constipation is defined not only by decreased stool frequency but also by other symptoms (eg, straining, lumpy or hard stools, or sensation of incomplete evacuations) b. Stool frequency is an accurate surrogate marker of colonic transit c. Extremes of stool form correlate strongly with colonic transit d. A digital rectal examination is useful for identifying pelvic floor dysfunction

254

Section V Colon

e. Certain symptoms (eg, sense of anorectal blockage during defecation and anal digitation) suggest pelvic floor dysfunction V.35. Which of the following statements about agents used to treat chronic constipation is false? a. Polyethylene glycol is safe and effective b. In controlled studies, dietary fiber supplements have increased stool weight and improved symptoms c. Bisacodyl induces high-amplitude propagated contractions d. Lubiprostone increases intestinal chloride secretion e. Colchicine and misprostol are preferred agents V.36. A 24-year-old woman who is 14 weeks’ pregnant comes to the emergency department with a 1-day history of melena. She has a history of migraines and has been taking a large amount of aspirin over the last few weeks for her symptoms. In the emergency department, she is orthostatic, with a hemoglobin level of 10.2 g/dL (recent value at last obstetrics’ appointment, 13.5 g/dL). Rectal examination shows frank melena. She continues to be tachycardic after rapid infusion of 3 L of fluid intravenously. You are asked to assess the need for endoscopy. Which of the following would you suggest? a. Admit to the obstetrics floor and observe b. Admit to the obstetrics floor and start proton pump inhibitor therapy c. Admit to the intensive care unit and observe d. Admit to the intensive care unit and perform upper endoscopy with midazolam e. Admit to the intensive care unit and consult surgery V.37. A 30-year-old woman in her first trimester of pregnancy is referred to you by her obstetrician after being found to be positive for hepatitis C antibody on initial laboratory work-up. The serum level of ALT is less than 1.5 times normal, and she is HCV RNA positive. Which of the following would you recommend? a. Initiate interferon and ribavirin therapy b. Initiate interferon therapy alone c. Monitor liver enzyme levels throughout pregnancy d. Recommend therapeutic termination e. Initiate lamivudine therapy V.38. A 27-year-old woman with a history of Crohn’s disease of the terminal ileum and right colon who has been well for 2 years while receiving treatment with

infliximab, 5 mg/kg every 8 weeks, tells you that she is pregnant. The last laboratory test results (hemoglobin level, chemistry panel, and iron, folate, and vitamin B12 levels) were all within normal limits. What would you recommend? a. Stop treatment with infliximab now b. Increase infliximab dose to 10 mg/kg to maintain remission during pregnancy c. Stop treatment with infliximab and start treatment with corticosteroids to maintain remission d. Continue infliximab at current dose e. Start treatment with azathioprine to decrease risk of antibody formation during pregnancy V.39. A 25-year-old woman with irritable bowel syndrome controlled with fiber and dietary restrictions consults you about her worsening constipation. Currently, she is 30 weeks’ pregnant and complains of bloating and abdominal discomfort. She now has a bowel movement once every 3 days (baseline is daily). She is also straining to evacuate and is concerned about causing hemorrhoids. Physical examination findings are remarkable only for her gravid uterus. Which of the following would you recommend? a. Suggest more dietary restrictions b. Suggest polyethylene glycol once daily c. Suggest castor oil to reduce straining d. Suggest mineral oil to reduce straining e. Suggest an over-the-counter senna preparation V.40. A 23-year-old woman is referred to you for worsening heartburn symptoms. Currently, she is 25 weeks’ pregnant with a twin gestation. She never had heartburn before but noted it around week 12 of pregnancy. She has been taking over-the-counter antacids with calcium, which helped for a little while but finds that she has to carefully watch her intake. She feels better when she sleeps sitting upright in a chair at night. She has been taking a histamine blocker for a few days without any noticeable relief. She says she does not have any dysphagia or hematemesis. Physical examination findings are remarkable only for her gravid abdomen. What would you recommend? a. Endoscopy to assess for esophagitis b. Continue with histamine blockers and antireflux measures c. Double the dose of histamine blockers d. Proton pump inhibitor therapy e. Prokinetic therapy

Answers

V.1. Answer d. Capsule endoscopy would not be appropriate because of the possibility of stricturing disease in a patient with Crohn’s disease. The concern would be capsule retention. All the other diagnostic tests would be reasonable for a patient with Crohn’s disease who has the presenting complaints described in this case. V.2. Answer d. The patient has evidence of a “cecal patch,” or periappendiceal inflammation, together with left-sided colitis. This can occur in ulcerative colitis and should not be confused with Crohn’s colitis. Serologic testing further supports the diagnosis of ulcerative colitis. V.3. Answer a. Patients with extensive colitis (pancolitis) are at increased risk for developing colorectal cancer after 8 to 10 years of disease. Colonoscopy with surveillance biopsies for detecting dysplasia is recommended every 1 to 2 years. V.4. Answer c. Smoking increases symptomatic recurrences of Crohn’s disease. This is often a concern postoperatively. Patients with Crohn’s colitis affecting more than one-third of the colon are believed to be at increased risk for colon cancer after 8 to 10 years of disease. Inflammatory bowel disease is more common in cold climates and in developed (rather than developing) countries. Sulfasalazine can cause reversible infertility. V.5. Answer b. On the basis of the patient’s history, penetrating perianal disease, possibly with an enterovesical fistula (pneumaturia), is suspected. External fistula openings can be difficult to identify. MRI of the pelvis is the preferred diagnostic modality for assessing perianal and pelvic disease.

V.6. Answer b. This patient has ileocolitis and arthralgias. Although azathioprine is appropriate for steroid sparing, it will not be as effective as methotrexate for the joint symptoms, which are more debilitating to him than the gastrointestinal symptoms. Methotrexate has been shown to have steroid-sparing properties as well as an effect on arthralgias. Cyclooxygenase-2 inhibitors have been shown to be safe short term for arthritis, but not long term for arthralgias associated with inflammatory bowel disease. 5-Aminosalicylic acid agents have no proven steroid-sparing capacity. Budesonide has been shown to be successful in tapering patients off prednisone, but it is not effective in treating joint symptoms. V.7. Answer d. Controlled trials have demonstrated that remission can be induced again by an anti-TNF agent in patients who originally had a response to one of these agents; thus, it is reasonable to switch to another anti-TNF agent. Starting azathioprine now will be too late for this patient, who has lost response. Premedication helps to prevent hypersensitivity reactions, but it does not help with loss of response. V.8. Answer b. Studies have shown that if patients with severely active ulcerative colitis do not demonstrate any response at all to intravenous corticosteroid therapy after 3 days, there is little chance of significant improvement with continued therapy. Originally, cyclosporine was given at a dose of 4 mg/kg intravenously over 24 hours, but controlled data have shown equivalent efficacy and few side effects with a dose of 2 mg/kg intravenously. If infliximab is to be considered in this setting, the dose is 5 mg/kg intravenously. Azathioprine does have steroid-sparing properties but is not appropriate for steroid-refractory disease

255

256

Section V Colon

and takes too long to take effect to be helpful in a hospitalized patient. V.9. Answer c. Cyclosporine is used for active steroid-refractory ulcerative colitis and has little role in the management of Crohn’s disease. A low dose of 5-aminosalicylic acid has not been shown to be superior to placebo to treat active disease or to maintain remission. Trials with a dose of 4 g have reported some success. Controlled data with 6-mercaptopurine have demonstrated effectiveness in maintaining surgically induced remission better than a high dose of 5-aminosalicylic acid or placebo. Short-term antibiotic therapy has been shown to have some benefit, but not long term. Methotrexate has not been administered to patients postoperatively. V.10. Answer d. The patient requires long-term therapy to maintain remission. Controlled trials have demonstrated that doses of mesalamine 1.6 to 2.4 g/day are adequate for maintaining remission. The patient, at this time, does not require more aggressive therapy with an immunomodulator. The patient is asymptomatic and does not require topical therapy with enemas and can continue receiving oral therapy. V.11. Answer c. Antinuclear antibodies are frequently positive in patients with Crohn’s disease treated with infliximab, but true drug-induced lupus is rare and is almost always associated with antibodies to either doublestranded DNA or histones. If the arthralgias had worsened at the end of each infliximab cycle and then improved with treatment, incompletely treated disease would have been the most likely answer. This patient has a classic presentation for delayed hypersensitivity reaction, with arthralgias occurring a week after each infusion. V.12. Answer c. Most guidelines recommend that surveillance colonoscopy with biopsies begin 8 to 10 years after diagnosis in patients with left-sided and extensive ulcerative colitis. However, patients who have primary sclerosing cholangitis are at increased risk for colorectal dysplasia and cancer and should be enrolled in yearly colonoscopic surveillance immediately. Because it has been 6 months since his last examination, he should return in 6 months for the next procedure. V.13. Answer b. Most patients with inflammatory bowel disease who have osteoporosis have one of the usual risk factors, such as postmenopausal state, low body mass index, or hypogonadism. Prolonged corticosteroid use and vitamin D deficiency are usual risk factors. Obesity is not a risk factor for osteoporosis.

V.14. Answer b. Erythema nodosum is typically a raised tender lesion on the shins. Pyoderma gangrenosum ulcerates with undermining borders and exhibits pathergy or a tendency to worsen with trauma. It has no pathognomonic histologic features. The course of pyoderma gangrenosum is often independent of that of the underlying inflammatory bowel disease. Infliximab and other anti-TNF agents have been shown to be useful in healing pyoderma gangrenosum, and they are not contraindicated in this setting. V.15. Answer e. Sulfasalazine and mesalamine have been associated with, among others, interstitial nephritis, pancreatitis, pneumonitis, and pericarditis. Cases of hypersensitivity colitis have been described with these drugs. However, these drugs have not been implicated as one of the drugs associated with either collagenous or lymphocytic colitis. V.16. Answer d. The acute gastroenteritis in this patient most likely has a viral cause. Caliciviruses, including Norovirus (also known as Norwalk-like virus), are the most important cause of viral gastroenteritis in adults. Contaminated shellfish, particularly oysters, is a well-recognized source of Norovirus infection. The patient may feel ill but appear quite well. Fecal leukocytes are typically not present on stool analysis in cases of viral gastroenteritis. Rotavirus is a common cause of acute viral gastroenteritis in infants but is less common in adults. Vibrio vulnificus infection may cause a diarrheal illness in patients with cirrhosis after ingestion of contaminated shellfish. The clinical presentation is less likely to be due to Yersinia or Campylobacter infection. V.17. Answer b. The patient has a first relapse of C difficile infection, for which a repeat course of metronidazole is the recommended therapy. If the patient had not had a response to metronidazole initially, or this was a second relapse, then treatment with vancomycin would be reasonable. Cholestyramine can help decrease symptoms in mild disease, but when used as monotherapy, results have been disappointing. Rifaximin has been reported to be useful in cases of refractory infection, after a standard course of vancomycin. Saccharomyces boulardii may decrease the likelihood of recurrent C difficile infection but has not been proved effective as primary monotherapy. V.18. Answer d. Bacteremia complicates nontyphoid salmonellosis in 8% of normal individuals. The risk is higher at the extremes of age (65 years), with corticosteroid use, and in patients with inflammatory bowel disease or other forms of immunosuppression. In these

Questions and Answers

circumstances antibiotic therapy is recommended. The presence of an aortic aneurysm is also a risk factor for persistent infection and is an indication for treatment independent of the aforementioned risk factors. Oral antibiotic therapy is favored, and strict isolation would not be deemed necessary for this patient. V.19. Answer c. The sudden onset of watery diarrhea with subsequent development of bloody diarrhea and the presence of fecal leukocytes suggest a bacterial cause for the clinical presentation. Campylobacter is a more common cause of dysentery than Salmonella or E coli O157:H7. Rotavirus and staphylococcal infections do not cause bloody diarrhea. V.20. Answer b. This patient has a first case of uncomplicated diverticulitis. Because he is tolerating oral intake without difficulty, outpatient management may be pursued. If the patient had evidence of complication such as abscess or was not able to tolerate oral intake, hospitalization would be necessary. Because this is the patient’s first episode of diverticulitis and it is uncomplicated, surgery is not indicated. Colonoscopy is contraindicated in the setting of acute diverticulitis. For this man, it would be reasonable to perform colonoscopy 2 to 4 weeks after resolution of the acute symptoms, because he has not had colon cancer screening previously. V.21. Answer a. Breast cancer incidence and mortality in hereditary nonpolyposis colorectal cancer (Lynch syndrome) are not increased above that of the general population. V.22. Answer d. Cowden syndrome is characterized by mucocutaneous fibromas, cutaneous tricholemmomas, prominent forehead, and upper and lower gastrointestinal tract hamartomas, most commonly juvenile polyps. Breast cancers occur in up to 50% of those with germline PTEN mutation. Both the uterine and thyroid cancer risk is increased in this syndrome. In contrast to juvenile polyposis, the gastrointestinal cancer risk does not appear to be increased in Cowden syndrome. Thus, PTEN testing is important to direct surveillance and kindred screening. V.23. Answer b. Osteomas are essentially always benign and rarely symptomatic. V.24. Answer e. Breast cancer occurs in 50% of women with PeutzJeghers syndrome, compared with a 40% risk for colorectal cancer, 40% risk for gastroduodenal cancer, 35% risk for pancreatic cancer, and 20% risk for ovarian cancer.

257

V.25. Answer e. Carcinoid tumors are the most common gastrointestinal tract endocrine tumor, and the most common intestinal carcinoid tumors are located in the appendix. Appendiceal carcinoids are usually asymptomatic and found incidentally at operation. Of appendiceal carcinoids, 98% are solitary, benign, and smaller than 1 cm. Appendectomy is considered curative for all appendiceal carcinoids smaller than 2 cm. If an appendiceal carcinoid is larger than 2 cm or if there is evidence of malignancy and possible invasion, right hemicolectomy is recommended. V.26. Answer a. This patient meets the criteria for irritable bowel syndrome. Initial management should be noninvasive and symptom driven. V.27. Answer b. This patient has features of pelvic floor dysfunction. Testing, such as an anorectal manometry with balloon expulsion, is needed. V.28. Answer c. True colonic inertia is rare, but such patients are best served by colectomy with ileorectal anastomosis. The key points are 1) the absence of pelvic floor dysfunction, 2) slow colon transit with normal upper gut transit, and 3) lack of response to therapy. The medical option may still exist, but when the patient’s condition is refractory to medical options, surgery needs to be considered. V.29. Answer d. This patient does not meet the criteria for irritable bowel syndrome. Celiac disease is high on the list of possibilities. The rash may reflect dermatitis herpetiformis. Evaluation with antibody testing and, if positive, small-bowel biopsy is indicated. If the results of these tests suggest the diagnosis of celiac disease, a gluten-free diet is recommended. V.30. Answer e. The constant nature of the pain and the changes with movement raise the question of pain arising outside the gastrointestinal tract, and the abdominal wall is a possible site. V.31. Answer b. After a secondary cause for constipation has been excluded, colonic transit and anorectal functions should be assessed when constipation does not respond to simple measures. Patients with slow-transit constipation unresponsive to laxatives and without pelvic floor dysfunction may require subtotal colectomy. V.32. Answer b. A digital rectal examination is very useful in identifying defecatory disorders. Although clinical features are

258

Section V Colon

helpful, the evidence suggests that anorectal testing is necessary to confirm or exclude defecatory disorders. A rectal balloon expulsion test is highly sensitive and specific for identifying defecatory disorders. Because up to 20% of healthy subjects have dyssynergia detected on anorectal manometry, this finding cannot be interpreted in isolation but in context with clinical features and the results of other tests. Barium or magnetic resonance proctography should be considered when a structural disorder (eg, a clinically significant rectocele) is suspected, when there is discrepancy between the results of anorectal manometry and the rectal balloon expulsion test, or when there is discrepancy between the clinical impression and the results of these tests. V.33. Answer c. The sympathetic nervous system inhibits gastrointestinal and colonic motor activity via adrenergic α2 receptors. V.34. Answer b. Stool frequency does not reflect colonic transit. V.35. Answer e. Colchicine and misoprostol generally should not be used to manage constipation because they are associated with potentially significant adverse effects, including neuromyopathy for colchicine and pronounced bloating for misoprostol. V.36. Answer d. A pregnant patient with appropriate indications should undergo endoscopy. This patient has evidence of active bleeding of the gastrointestinal tract that may require therapeutic intervention. It would be inappropriate to admit and observe or to start antisecretory therapy alone. Endoscopy in a controlled setting with midazolam for comfort would be appropriate. Surgical consultation without endoscopy is premature. V.37. Answer c. Most women with hepatitis C do not require treatment during pregnancy. The toxicity of interferon warrants

careful consideration of its benefits because most women will not experience liver failure or the decompensation of hepatitis during pregnancy. Ribavirin is absolutely contraindicated during pregnancy. It would be inappropriate to recommend therapeutic termination on the basis of a positive serologic test. Lamivudine is not used in the treatment of hepatitis C infection. V.38. Answer d. Infliximab is a category B drug, and although it crosses the placenta, it does not do so readily until early in the third trimester. Maintaining remission is the most important way to minimize an adverse birth outcome. Because the patient is doing well with infliximab therapy, there is no reason to increase the dose or to switch to corticosteroids. There is no evidence that the development of antibodies to any biologic occurs more often in a pregnant patient than in a nonpregnant one; thus, the initiation of azathioprine therapy is unnecessary. V.39. Answer b. Constipation is common in pregnancy, particularly in the third trimester. A patient with underlying irritable bowel syndrome–constipation may experience even more symptoms than a healthy control. The first-line therapy after fiber to treat constipation is polyethylene glycol. Castor oil has been associated with uterine rupture and should not be used. Mineral oil will not help with increasing stool frequency. Dietary restrictions for constipation during pregnancy are not appropriate. Senna may lead to worse cramping and bloating. V.40. Answer d. Many women experience heartburn during pregnancy, especially in the second and third trimesters. Proton pump inhibitors are low risk and effective in controlling heartburn symptoms when over-the-counter antacids and histamine blockers fail. Without red flag signs, endoscopy is not warranted and empiric therapy is appropriate. Prokinetics are unlikely to be as helpful as proton pump inhibitor therapy.

Section VI Liver

This page intentionally left blank

22 Approach to the Patient With Abnormal Liver Tests and Acute Liver Failure John J. Poterucha, MD

This chapter includes the following topics: 1. General discussion of commonly used liver tests 2. Differential diagnosis and discussion of diseases characterized by an increase in hepatocellular enzyme levels 3. Differential diagnosis and discussion of diseases characterized by an increase in cholestatic enzyme levels 4. Evaluation of patients who have jaundice 5. Diagnostic algorithms for evaluating patients who have abnormal liver tests 6. Management of patients who have acute liver failure

Commonly Used Liver Testsa Aminotransferases (Alanine and Aspartate Aminotransferases)

The aminotransferases (also referred to as transaminases) are located in hepatocytes and, thus, are markers of liver cell injury (hepatocellular disease). Injury of the hepatocyte membrane allows these enzymes to “leak” out of hepatocytes, and within a few hours after liver injury, the serum levels of the enzymes increase. Aminotransferases consist of alanine aminotransferase (ALT) and aspartate aminotransferase (AST). ALT is relatively specific for liver injury, whereas AST is found not only in hepatocytes but also in skeletal and cardiac muscle and in other organs. ALT has a longer half-life than AST; thus, improvements in ALT levels lag behind those of AST.

Abbreviations: ALT, alanine aminotransferase; AST, aspartate aminotransferase; ERCP, endoscopic retrograde cholangiopancreatography; MRCP, magnetic resonance cholangiopancreatography

Marked muscle injury can produce striking increases in AST levels and, to a lesser extent, in ALT levels. Alkaline Phosphatase

Alkaline phosphatase is an enzyme located on the hepatocyte membrane bordering bile canaliculi. Because alkaline phosphatase is found also in bone and placenta, an increase in its level without other indication of liver disease should prompt further testing to discover if the increase is from liver or other tissues. One way of doing this is to determine the concentration of alkaline phosphatase isoenzymes. Another way is to determine the level of γ-glutamyltransferase, an enzyme of intrahepatic biliary canaliculi. Other than to confirm the liver origin of an increased level of alkaline phosphatase, γ-glutamyltransferase is of little use in the evaluation of diseases of the liver because its synthesis can be induced by many medications, thus decreasing its specificity for clinically important liver disease. Bilirubin

Bilirubin is the water-insoluble product of heme metabolism that is taken up by hepatocytes and conjugated with glucuronic acid to form monoglucuronides and diglucuronides. Conjugation makes bilirubin water-soluble, allowing it to be excreted in bile. The serum concentration of bilirubin is measured in direct (conjugated) and indirect (unconjugated) fractions. Diseases characterized by overproduction of bilirubin, such as hemolysis or resorption of a hematoma, are characterized by hyperbilirubinemia that is 20% or less conjugated bilirubin. Hepatocyte dysfunction or impaired bile flow produces hyperbilirubinemia that is usually 20% to 80% or more conjugated bilirubin. Patients with an inherited

261

262

Section VI Liver

disorder of bilirubin excretion into the canaliculus, such as Dubin-Johnson syndrome or Rotor’s syndrome, have hyperbilirubinemia that is more than 80% conjugated. Because conjugated bilirubin is water-soluble and may be excreted in urine, patients with conjugated hyperbilirubinemia may note dark urine. In these patients, the stools are lighter in color because of the absence of bilirubin pigments. Prothrombin Time and Albumin

Prothrombin time and serum albumin are commonly used markers of liver synthetic function. Abnormalities of prothrombin time and albumin imply severe liver disease and should prompt immediate evaluation. Prothrombin time is a measure of the activity of factors II, V, VII, and X, all of which are synthesized in the liver. These factors are dependent also on vitamin K for synthesis. Vitamin K deficiency may be produced by antibiotics, prolonged fasting, small-bowel mucosal disorders such as celiac disease, or severe cholestasis with an inability to absorb fat-soluble vitamins. Hepatocellular dysfunction is characterized by an inability to synthesize clotting factors despite adequate stores of vitamin K. A simple way to differentiate vitamin K deficiency from hepatocellular dysfunction in a patient with a prolonged prothrombin time is to administer vitamin K. Administration of vitamin K improves prothrombin time within 2 days in a vitamin K-deficient patient but has little effect if the prolonged prothrombin time is due to liver disease with poor synthetic function. Because albumin has a half-life of 21 days, decreases due to liver dysfunction do not occur acutely; however, the serum level of albumin can decrease relatively quickly in a patient who has a severe systemic illness such as bacteremia. This rapid decrease likely is caused by the release of cytokines, which accelerate the metabolism of albumin. Other causes of hypoalbuminemia include urinary or gastrointestinal tract losses, and these should be considered in a patient who has hypoalbuminemia but not overt liver disease.

Table 22.1 Common Causes of Acute Hepatitis Disease

Clinical Clue

Diagnostic Test

Hepatitis A Hepatitis B Drug-induced

Exposure history Risk factors Compatible medication History of alcohol excess AST:ALT >2 AST 3 months) increases in aminotransferase levels are included in the category of chronic hepatitis. The increase in aminotransferase levels generally is more modest (1.5–5 times the upper limit of normal) than that in acute hepatitis. Although patients may be asymptomatic, they occasionally complain of fatigue and right upper quadrant pain. The most important and common disorders that cause chronic hepatitis are listed in Table 22.2.

Hepatocellular Disorders

Diseases that affect primarily hepatocytes are characterized predominantly by increased levels of aminotransferases. The disorders are best categorized as acute (generally 2

Anti-HCV, HCV RNA HBsAg Ultrasonography, liver biopsy

Autoimmune hepatitis

Liver biopsy, improvement with abstinence ALT 200–1,500 U/L, Antinuclear or usually female, other anti-smooth muscle autoimmune disease antibody, biopsy

Abbreviations: ALT, alanine aminotransferase; AST, aspartate aminotransferase; HBsAg, hepatitis B surface antigen; HCV, hepatitis C virus.

22 Approach to the Patient With Abnormal Liver Tests and Acute Liver Failure

Risk factors for hepatitis C include a history of intravenous drug use or exposure to blood products. Most patients with hepatitis B are from an endemic area such as parts of Asia or Africa or have a history of illegal drug use or multiple sexual contacts. Patients with nonalcoholic fatty liver disease are usually obese and have diabetes mellitus or hyperlipidemia. A complete history is needed to help diagnose drug-induced or alcohol-induced liver disease. Autoimmune hepatitis may manifest as acute or chronic hepatitis. Patients usually have an ALT serum level of 200 to 800 U/L, higher than that in other disorders that cause chronic hepatitis. Autoantibodies, hypergammaglobulinemia, and other autoimmune disorders are helpful clues to the diagnosis of autoimmune hepatitis.

263

obstruction from a stone may produce marked increases in aminotransferase levels. Intrahepatic mass lesions should be considered if a patient has an increased level of alkaline phosphatase and a history of malignancy. Infiltrative disorders such as amyloidosis, sarcoidosis, or lymphoma may produce a markedly increased alkaline phosphatase level with a normal bilirubin concentration. Any systemic inflammatory process such as infection or immune disorder may produce nonspecific liver test abnormalities. The abnormalities usually are a mixed cholestatic (alkaline phosphatase) and hepatocellular (ALT or AST) pattern.

Jaundice Cholestatic Disorders

Diseases that affect predominantly the biliary system are termed cholestatic diseases. These can affect the microscopic ducts (eg, primary biliary cirrhosis), large bile ducts (eg, pancreatic cancer obstructing the common bile duct), or both (eg, primary sclerosing cholangitis). In these disorders, the predominant abnormality is generally the alkaline phosphatase level. Although diseases that produce increased bilirubin levels are often called “cholestatic,” it is important to remember that severe hepatocellular injury, as in acute hepatitis, also produces hyperbilirubinemia because of hepatocellular dysfunction. Causes of cholestasis are listed in Table 22.3. Primary biliary cirrhosis usually occurs in women and can cause fatigue or pruritus. Of patients with primary sclerosing cholangitis, 70% to 80% also have ulcerative colitis. Patients with primary biliary cirrhosis or primary sclerosing cholangitis often are asymptomatic but may have jaundice, fatigue, or pruritus. Large bile duct obstruction often is due to stones or benign or malignant strictures. Remember that acute bile duct Table 22.3 Common Causes of Cholestasis Disease

Clinical Clue

Diagnostic Test

Primary biliary cirrhosis Primary sclerosing cholangitis Large bile duct obstruction Drug-induced

Middle-aged woman

Antimitochondrial antibody ERCP, MRCP

Association with ulcerative colitis

Jaundice and pain are common Compatible medication/ timing Infiltrative History of malignancy, disorder amyloidosis, sarcoidosis Inflammation- Symptoms of underlying associated inflammatory disorder

Ultrasonography, ERCP, MRCP Improvement after agent withdrawn Ultrasonography, computed tomography Blood cultures, appropriate antibody tests

Abbreviations: ERCP, endoscopic retrograde cholangiopancreatography; MRCP, magnetic resonance cholangiopancreatography.

Jaundice is visibly evident hyperbilirubinemia and occurs when the bilirubin concentration is more than 2.5 mg/dL. A common disorder that produces unconjugated hyperbilirubinemia (but not usually jaundice) is Gilbert’s syndrome. Total bilirubin is generally less than 3.0 mg/dL, whereas direct bilirubin is 0.3 mg/dL or less. The level of bilirubin usually is highest when a patient is ill or fasting. A presumptive diagnosis of Gilbert’s syndrome can be made in an otherwise well patient who has unconjugated hyperbilirubinemia, normal liver enzyme values, and a normal concentration of hemoglobin (to exclude hemolysis). Patients with direct hyperbilirubinemia can be categorized as those with a nonobstructive condition and those with an obstructive condition. Abdominal pain, fever, or a palpable gallbladder (or a combination of these) is suggestive of obstruction. Risk factors for viral hepatitis, recent ingestion of a potentially hepatotoxic drug, bilirubin concentration more than 15 mg/dL, and persistently high aminotransferase levels suggest that the jaundice is due to hepatocellular dysfunction. In patients with resolving acute hepatitis, improvement in bilirubin concentration often lags behind the improvement in aminotransferase levels. In diseases resulting in large bile duct obstruction, extrahepatic and intrahepatic biliary dilatation can be identified on imaging studies, especially if the bilirubin concentration is more than 10 mg/dL and the patient has had jaundice for more than 2 weeks. Acute large bile duct obstruction, usually from a stone, may not cause dilatation of the bile ducts, and if the clinical suspicion is strong for bile duct obstruction despite normal-sized bile ducts seen on ultrasonography or computed tomography, the biliary tree should be imaged with endoscopic retrograde cholangiopancreatography (ERCP), magnetic resonance cholangiopancreatography (MRCP), or endoscopic ultrasonography.

General Approach to Abnormal Liver Tests

When a patient has abnormal liver tests at presentation, it is helpful to classify the patient’s condition as one of the clinical syndromes listed in Table 22.4, although the overlap among these categories is considerable. Patients with acute hepatitis

264

Section VI Liver

Table 22.4 Abnormal Liver Tests: Clinical Syndromes

“First-time” increase in liver enzymes Acute hepatitis Chronic hepatitis Cholestasis without hepatitis or jaundice Jaundice Cirrhosis or portal hypertension

or cirrhosis often have jaundice. The approach to patients with acute hepatitis, chronic hepatitis, cholestasis, and jaundice is outlined above. Patients with a “first-time,” often incidental, increase in liver enzyme levels are usually asymptomatic. As long as 1) no risk factors for liver disease are identified, 2) liver enzyme levels are less than three times normal, 3) liver function is preserved, and 4) the patient feels well, observation is reasonable, with the test repeated in a few months. If the repeat test results are still abnormal, the patient’s condition fits the category of chronic hepatitis or cholestasis and appropriate evaluation should be initiated. A similar approach can be taken for patients with incidentally discovered abnormal liver tests who are taking medications that only rarely cause liver disease. Patients also may present with cirrhosis or portal hypertension. Most patients with portal hypertension have cirrhosis,

although occasionally patients present with noncirrhotic portal hypertension that is idiopathic or due to portal vein thrombosis. The evaluation of a patient with cirrhosis is similar to that of a patient with chronic hepatitis and cholestasis (as shown above). Alpha1-antitrypsin deficiency, genetic hemochromatosis, alcoholic liver disease, and nonalcoholic fatty liver disease frequently have cirrhosis as the first manifestation of liver disease. If a patient has clinical and imaging features that strongly suggest cirrhosis, confirmatory liver biopsy is not necessary.

Algorithms for Evaluating Patients With Abnormal Liver Tests

Algorithms for evaluating patients who have abnormal liver tests are, at best, guidelines, and, at worst, misleading. Always remember that in evaluating (or not evaluating) abnormal liver tests, the patient’s clinical presentation should be considered. Generally, a patient with liver test abnormalities that are less than twice normal may be followed as long as the patient is asymptomatic and the albumin level, prothrombin time, and bilirubin concentration are normal. Also, persistent abnormalities should be evaluated. Algorithms for evaluating increased levels of ALT, alkaline phosphatase, and conjugated bilirubin are shown in Figures 22.1 to 22.3.

Abnormal ALT level

Persistent increase or patient symptomatic or impaired liver function

Acute hepatitis

Chronic hepatitis

US, IgM anti-HAV HBsAg, IgM anti-HBc HCV RNA, ANA Ceruloplasmin if 17.5 mg/dL Abbreviation: INR, international normalized ratio. a

Any one of the three criteria.

contraindicated, yet still allow time for spontaneous recovery. The most well known and widely used model is the King’s College criteria (Table 22.6). Liver transplantation likely improves mortality, although improved outcomes have been assessed only by comparison with historical controls. The appearance of encephalopathy precedes cerebral edema; therefore, patients with acute hepatitis and evidence of liver failure need to be monitored carefully for mental status changes. Patients with encephalopathy should receive lactulose, although this agent is not as effective in acute liver failure as in chronic liver disease and may not prevent cerebral edema from developing later. Patients with stage II encephalopathy usually are admitted to an intensive care unit for close monitoring of mental status and vital signs. Sedatives should be avoided initially to allow close monitoring of mental status. Also, at most centers, computed tomography of the head is performed to exclude an alternative cause of mental status changes. Patients who reach stage III encephalopathy are at considerable risk for progression to cerebral edema. Because clinical signs and computed tomography are insensitive for detecting increased intracranial pressure, many centers institute intracranial pressure monitoring when patients reach stage III encephalopathy. Endotracheal intubation and mechanical ventilation usually precede placement of the intracranial pressure monitor. Various such monitors are used, all of which can be complicated by infection and bleeding. The goal of intracranial pressure monitoring is to allow treatment of high pressure and also to identify which patient is too ill for liver transplantation because of a prolonged period of excessively high intracranial pressure. Generally, the goal is to maintain intracranial pressure less than 40 mm Hg and cerebral perfusion pressure (the difference between mean arterial pressure and intracranial pressure) between 60 and 100 mm Hg. Excessively high cerebral perfusion pressures (>120 mm Hg) can increase cerebral edema. Maneuvers that cause straining, including tracheal suctioning, should be avoided or limited. Paralyzing agents and sedatives may be necessary, although they may limit further

268

Section VI Liver

assessment of neurologic status. For intracranial pressure more than 20 mm Hg or cerebral perfusion pressure less than 60 mm Hg, elevation of the head to 20 degrees, hyperventilation to a Paco2 of 25 mm Hg, and mannitol (if renal function is intact) are advised. Barbiturate-induced coma can be used for refractory cases. A prolonged increase in intracranial pressure above mean arterial pressure may signify brain death and generally is a contraindication to liver transplantation. A sudden decrease in intracranial pressure may indicate brain herniation. In patients with acute liver failure, the prolonged prothrombin time is a simple noninvasive measure to follow, and coagulopathy is not corrected unless there is bleeding or an intervention such as placement of a monitoring device is planned. If bleeding occurs or an invasive procedure is necessary, fresh frozen plasma usually is administered first, although platelets and fibrinogen may be necessary in certain circumstances. Continuous infusion of 5% or 10% dextrose is used to keep the plasma glucose level between 100 and 200 mg/dL. The plasma glucose level should be monitored at least twice daily. Both bacteremia and fungemia are sufficiently frequent that periodic blood cultures are advised and prophylaxis with antimicrobials may be initiated, although this practice has not been shown to affect survival. Acute liver failure is the indication for 6% of liver transplants in the United States. Even though survival with transplantation for acute liver failure is lower than that for transplantation for other indications, outcomes are an improvement over the dismal survival rates for patients with acute liver failure who meet poor prognostic markers such as the King’s College criteria. Transplantation should be performed when a poor outcome is anticipated, yet before the patient has uncontrolled

sepsis or prolonged periods of increased intracranial pressure that prevent recovery even with a functioning transplanted liver. SUGGESTED READING

Green RM, Flamm S. AGA technical review on the evaluation of liver chemistry tests. Gastroenterology. 2002 Oct;123(4):1367–84. Larson AM, Polson J, Fontana RJ, Davern TJ, Lalani E, Hynan LS, et al; Acute Liver Failure Study Group. Acetaminophen-induced acute liver failure: results of a United States multicenter, prospective study. Hepatology. 2005 Dec;42(6):1364–72. Lee WM, Squires RH Jr, Nyberg SL, Doo E, Hoofnagle JH. Acute liver failure: summary of a workshop. Hepatology. 2008 Apr;47(4): 1401–15. O’Grady JG, Alexander GJ, Hayllar KM, Williams R. Early indicators of prognosis in fulminant hepatic failure. Gastroenterology. 1989 Aug;97(2):439–45. Ostapowicz G, Fontana RJ, Schiodt FV, Larson A, Davern TJ, Han SH, et al; U.S. Acute Liver Failure Study Group. Results of a prospective study of acute liver failure at 17 tertiary care centers in the United States. Ann Intern Med. 2002 Dec 17;137(12):947–54. Polson J, Lee WM; American Association for the Study of Liver Disease. AASLD position paper: the management of acute liver failure. Hepatology. 2005 May;41(5):1179–97. Rockey DC, Caldwell SH, Goodman ZD, Nelson RC, Smith AD; American Association for the Study of Liver Diseases. Liver biopsy. Hepatology. 2009 Mar;49(3):1017–44. Sass DA, Shakil AO. Fulminant hepatic failure. Liver Transpl. 2005 Jun;11(6):594–605. Sedlack RE, Loftus CG, Viggiano TR, Poterucha JJ. Gastroenterology and hepatology. In: Ghosh AK, editor. Mayo Clinic internal medicine review. 8th ed. Rochester (MN): Mayo Clinic Scientific Press and Florence (KY): Informa Healthcare. c2008. p. 257–328.

23 Chronic Viral Hepatitis John J. Poterucha, MD

Viral infections are important causes of liver disease worldwide. The five primary hepatitis viruses that have been identified are A, B, C, D (or delta), and E. Other viruses such as cytomegalovirus and Epstein-Barr virus also can result in hepatitis as part of a systemic infection. In addition, medications, toxins, autoimmune hepatitis, or Wilson’s disease may cause acute or chronic hepatitis. It is useful to divide hepatitis syndromes into acute and chronic forms. Acute hepatitis can last from weeks up to 6 months and is often accompanied by jaundice. Symptoms of acute hepatitis tend to be similar regardless of cause and include anorexia, malaise, dark urine, fever, and mild abdominal pain. In chronic hepatitis, patients are often asymptomatic but may complain of fatigue. Occasionally, they have manifestations of advanced liver disease (ascites, variceal bleeding, or encephalopathy) as the initial presentation of chronic hepatitis. Each hepatitis virus causes acute hepatitis, but only hepatitis B, C, and D viruses can cause chronic hepatitis. The purpose of this chapter is to review the primary hepatitis viruses. A more comprehensive discussion of acute hepatitis is found in other chapters. The four common hepatitis viruses are compared in Table 23.1, and the disease burden of the three most important viruses in the United States is summarized in Table 23.2.

Table 23.1 Comparison of the Four Primary Hepatitis Viruses Feature

HAV

HBV

HDV

HCV

Incubation, days Jaundice

15–50

30–160

Unknown

14–160

Common

Uncommon

Acute or chronic Parenteral Anti-HDV

Acute or chronic Parenteral HCV RNA

30% of patients Course Acute Acute or chronic Transmission Fecal-oral Parenteral Test for IgM HBsAg diagnosis anti-HAV Common

Abbreviations: HAV, hepatitis A virus; HBV, hepatitis B virus; HBsAg, hepatitis B surface antigen; HCV, hepatitis C virus; HDV, hepatitis D virus.

Table 23.2 Clinical Effect of Hepatitis Viruses in the United Statesa

New infections per year Fulminant, deaths/year Chronic infections Chronic liver disease, deaths/year

HAV

HBV

HCV

25,000

43,000

17,000

50

100

Rare

0

800,000–1.4 million

2.7–3.9 million

0

3,000

12,000

Abbreviations: HAV, hepatitis A virus; HBV, hepatitis B virus; HCV, hepatitis C virus.

Abbreviations: ALT, alanine aminotransferase; anti-HAV, antibody to hepatitis A virus; anti-HBc, antibody to hepatitis B core; anti-HBe, antibody to hepatitis B e; anti-HBs, antibody to hepatitis B surface; anti-HCV, antibody to hepatitis C virus; anti-HDV, antibody to hepatitis D virus; HAV, hepatitis A virus; HBeAg, hepatitis B e antigen; HBsAg, hepatitis B surface antigen; HBV, hepatitis B virus; HCV, hepatitis C virus; HDV, hepatitis D virus; HIV, human immunodeficiency virus; PCR, polymerase chain reaction; SSRI, selective serotonin reuptake inhibitor

a

Data from The Centers for Disease Control and Prevention 2007 estimates.

Hepatitis A Epidemiology

The incidence of acute hepatitis A virus (HAV) infection is decreasing in the United States. Common routes of transmission 269

270

Section VI Liver

of HAV are ingestion of contaminated food or water and contact with an infected person. Groups at particularly high risk include people living in or traveling to underdeveloped countries, children in day care centers, homosexual men, and perhaps persons who ingest raw shellfish. Outbreaks of HAV infection in communities are recognized frequently, although an exact source may not be found. The incubation period for HAV is 2 to 6 weeks.

also to travelers to areas with an intermediate or high prevalence of hepatitis A, men who have sex with men, intravenous drug users, recipients of clotting factor concentrates, and patients with chronic liver disease. Widespread vaccination of health care workers or food handlers has not been advised.

Hepatitis B Clinical Presentation and Natural History Epidemiology

The most important determinant of the severity of acute hepatitis A is the age at which infection occurs. Persons infected when younger than 6 years have nonspecific symptoms that rarely include jaundice. Adolescents or adults who acquire HAV infection usually have jaundice. Hepatitis A is almost always a self-limited infection. There may be a prolonged cholestatic phase characterized by persistence of jaundice for up to 6 months. Rarely, acute hepatitis A manifests as acute liver failure that may require liver transplantation. HAV does not cause chronic infection and should not be in the differential diagnosis of chronic hepatitis. Diagnostic Tests

The diagnosis of acute hepatitis A is established by the presence of IgM hepatitis A antibody (anti-HAV), which appears at the onset of the acute phase of the illness and disappears in 3 to 6 months. The IgG anti-HAV also becomes positive during the acute phase, but it persists for decades and is a marker of immunity from further infection. A patient with IgG anti-HAV, but not IgM anti-HAV, has had an infection in the remote past or has been vaccinated. Treatment and Prevention

The treatment of acute hepatitis A is supportive. Immune serum globulin should be administered to all household and intimate (including day care) contacts within 2 weeks after exposure. Postexposure vaccination is also advised. Immunization with hepatitis A vaccine is recommended for all children at 12 months of age. Hepatitis A vaccine should be offered

Hepatitis B virus (HBV) is a DNA virus that causes about 30% of cases of acute viral hepatitis and 15% of cases of chronic viral hepatitis in the United States. Most chronically infected persons in the United States are immigrants from Asia and Africa, where infection is acquired at the time of birth or in early childhood. Major risk factors for adult disease acquisition in the United States are sexual promiscuity and intravenous drug use. Diagnostic Tests

A brief guide to serologic markers for hepatitis B is provided in Table 23.3. The interpretation of serologic patterns is found in Table 23.4. The best serologic test for acute hepatitis B is

Table 23.3 Hepatitis B Serologic Markers Test

Significance

Hepatitis B surface antigen (HBsAg) Antibody to hepatitis B surface (anti-HBs) IgM antibody to hepatitis B core (IgM anti-HBc)

Current infection Immunity (immunization or resolved infection) Recent infection or “reactivation” of chronic infection Remote infection

IgG antibody to hepatitis B core (IgG anti-HBc) Hepatitis B e antigen (HBeAg) and/or HBV DNA >104 IU/mL

Active viral replication (high infectivity)

Table 23.4 Interpretation of Hepatitis B Serologic Patterns HBsAg

Anti-HBs

IgM anti-HBc

IgG anti-HBc

HBeAg

Anti-HBe

HBV DNA, IU/mL

+



+



+



+

– – + + +

+ + – – –

– – – – –

+ + + + +

– – – + –

± – + – +

– – 104 >104

Interpretation

Acute infection or, less commonly, acute flare of chronic hepatitis B Previous infection with immunity Vaccination with immunity Hepatitis B inactive carrier state Chronic hepatitis B HBeAg-negative chronic hepatitis B (often “precore” or “core promoter” variants)

Abbreviations: Anti-HBe, antibody to hepatitis B e; anti-HBs, antibody to hepatitis B surface; HBeAg, hepatitis B e antigen; HBsAg, hepatitis B surface antigen; HBV, hepatitis B virus; IgG anti-HBc, IgG antibody to hepatitis B core; IgM anti-HBc, IgM antibody to hepatitis B core.

23 Chronic Viral Hepatitis

IgM antibody to hepatitis B core (anti-HBc). Occasionally, a patient with acute hepatitis B (usually with a severe presentation such as acute liver failure) lacks hepatitis B surface antigen (HBsAg) and has only IgM anti-HBc as the marker for recent infection. Patients with an acute flare of chronic hepatitis B may redevelop IgM anti-HBc. Most patients with HBsAg have detectable HBV DNA in serum. HBV DNA levels greater than 104 IU/mL generally are considered to indicate active viral replication. Any detectable level in patients with cirrhosis is clinically important. Occasionally, patients have IgG anti-HBc as the only positive hepatitis B serologic marker. A common explanation in a population without risk factors for disease acquisition is a false-positive test (although the test is often repeatedly positive). Another explanation is a previous, resolved HBV infection in which the antibody to hepatitis B surface (anti-HBs) has decreased below the limit of detection. This can be documented indirectly by demonstrating an anamnestic type of response to hepatitis B vaccine. Rarely, patients with hepatitis B may have HBsAg levels that are below the level of detection, so that IgG anti-HBc is the only marker of infection. Although the significance of this low-level infection is unclear, these patients can be identified by the presence of HBV DNA in serum or liver. The accuracy of serologic and nucleic acid tests obviates the need for liver biopsy in the diagnosis of hepatitis B; however, liver biopsy is useful for grading inflammatory activity and determining the stage of fibrosis. Histologic features of hepatitis B are inflammation that is usually around the portal tract, variable fibrosis that initially is also portocentric, and the presence of ground-glass hepatocytes. Ground-glass hepatocytes are hepatocytes with cytoplasm that has a hazy, eosinophilic appearance. With immunostaining, these cells are positive for HBsAg (Fig. 23.1). Even though liver biopsy is the “gold standard” for diagnosing cirrhosis, it generally is not necessary for patients who have other features of cirrhosis, such as portal hypertension. Clinical Presentation and Natural History

The incubation period after HBV infection ranges from 30 to 160 days, and the clinical outcome varies. Acute hepatitis B in an adolescent or adult is icteric in about 30% of cases. Complete recovery with subsequent life-long immunity occurs in 95% of infected adults. About 5% of infected adults have persistence of HBsAg for longer than 6 months and are referred to as chronically infected. Immunosuppressed persons with acute HBV infection are more likely to become chronically infected, presumably because of an insufficient immune response against the virus. Patients with chronic hepatitis B infection may present in one of four phases (Fig. 23.2). An immune tolerant phase is recognized in many Asian patients who are infected perinatally. This phase is characterized by the presence of hepatitis B e antigen (HBeAg) and very high HBV DNA levels, yet normal levels of alanine aminotransferase (ALT). Generally, liver

271

A

B

Figure 23.1 Liver biopsy specimen from patient with hepatitis B. A,

Ground-glass hepatocyte (arrow). (Hematoxylin-eosin.) B, Immunostain for hepatitis B surface antigen showing positive staining of hepatocyte cytoplasm.

biopsy specimens from these patients show minimal changes except for ground-glass hepatocytes. The immune tolerant phase can last up to the age of 40 years and generally evolves under immune pressure into the HBeAg-positive chronic hepatitis B phase, characterized by increased ALT levels, the presence of HBeAg, and more than 104 IU/mL of HBV DNA. During this phase, active inflammation and often fibrosis are seen in liver biopsy specimens, indicating progressive liver injury. At a rate of about 10% per year, patients mount enough of an immune response to achieve a decrease in ALT levels, clearance of HBeAg and development of antibody to hepatitis B e (anti-HBe) (seroconversion), and a decrease in HBV DNA to less than 104 IU/mL. The resulting inactive carrier phase usually is not accompanied by progressive liver damage. In cross-sectional studies, about 60% of patients with chronic hepatitis B are in the inactive carrier phase. Many patients remain in this phase for many years and have a better prognosis than those with active liver inflammation and viral replication.

272

Section VI Liver

Immune tolerance Positive HBeAg DNA Normal ALT HBeAg-negative chronic hepatitis Negative HBeAg DNA Abnormal ALT

Precore mutation

Progression to cirrhosis

Inactive carrier Negative HBeAg DNA Normal ALT

HBeAg-positive chronic hepatitis Positive HBeAg DNA Abnormal ALT

HBeAg seroconversion

Figure 23.2 Phases of chronic hepatitis B virus infection. Black arrows, changes of histopathology; gray arrows, changes in serologic markers between phases. Up- and down-facing arrows, an increase or decrease of DNA level (↑ = low increase; ↑↑ = moderate increase; ↓↓ = moderate decrease; ↑↑↑ = high increase). ALT indicates alanine aminotransferase; HBeAg, hepatitis B e antigen. (Adapted from Pungpapong S, Kim WR, Poterucha JJ. Natural history of hepatitis B virus infection: an update for clinicians. Mayo Clin Proc. 2007 Aug;82[8]:967–75. Used with permission of Mayo Foundation for Medical Education and Research.)

About one-third of inactive carriers have a reactivation of chronic hepatitis characterized by abnormal ALT levels and HBV DNA of more than 104 IU/mL. This may be associated with a reversion to the HBeAg-positive state, but more commonly it is due to a precore or core promoter variant. This HBeAg-negative chronic hepatitis B phase is associated with progression of liver damage. Patients with HBeAg-negative chronic hepatitis B are more likely to have lower DNA levels and a fluctuating course than patients with HBeAg-positive chronic hepatitis B. Also, the patients generally are older and have more advanced fibrosis because HBeAg-negative chronic hepatitis B tends to occur later in the course of infection. Patients with chronic hepatitis B may experience spontaneous flares of disease characterized by markedly abnormal ALT levels, deterioration in liver function, and often seroconversion of HBeAg. The differential diagnosis for acute hepatitis in patients with chronic hepatitis B is listed in Table 23.5. Because disease activity changes in patients with chronic hepatitis B, even after years of senescence, periodic monitoring with liver tests and hepatitis B markers is necessary. HBsAg clears spontaneously in about 1% of chronically infected patients annually, although this rate is lower in endemic areas where HBV is acquired at a very young age. Overall, about 15% to 40% of patients with chronic HBV infection develop serious sequelae of the disease, either decompensated liver disease or hepatocellular carcinoma. Factors associated with the development of cirrhosis are older age; infection with hepatitis C virus (HCV), human immunodeficiency virus (HIV), or hepatitis D virus (HDV); genotype C;

Table 23.5 Causes of Acute Hepatitis in Patients With Chronic Hepatitis B Cause

Clinical Clues

Spontaneous “reactivation” Seroconversion of HBeAg, of hepatitis B reappearance of IgM anti-HBc Flare due to immune Chemotherapy, antirejection therapy, suppression corticosteroids, anti-tumor necrosis factor agents Induced by antiviral therapy Interferon (common), oral agents (rare) Exposure to hepatitis D (usually due to Superimposed infection illicit drug use), A, or C with other viruses, especially hepatitis D virus Other causes of acute History of alcohol excess, medications, hepatitis illegal drugs Abbreviations: Anti-HBc, antibody to hepatitis B core; HBeAg, hepatitis B e antigen.

longer duration of infection; high HBV DNA levels; and alcohol abuse. Many of these factors, including HBV DNA level, are associated also with an increased risk for the development of hepatocellular carcinoma. Patients with chronic hepatitis B and cirrhosis are at high risk for the development of hepatocellular carcinoma, and surveillance every 6 to 12 months is advised. Surveillance also is advised for patients without cirrhosis who meet one of the following criteria: family history of hepatocellular carcinoma,

23 Chronic Viral Hepatitis

Asian male older than 40 years, Asian female older than 50 years, black African older than 20 years, and persistent increase in ALT level together with an HBV DNA value of more than 104 IU/mL. The method of surveillance can vary among centers, but many use ultrasonography with or without alpha fetoprotein at 6-month intervals. Eight hepatitis B genotypes, labeled A through H, have been identified, and the genotype for a patient is determined largely by the country in which infection is acquired. All genotypes have been identified in the United States. In Asian patients, genotype B has a better prognosis than genotype C, including a higher rate of clearance of HBeAg, a slower rate of progression to cirrhosis, and a lower likelihood of the development of hepatocellular cancer. Genotypes A and B have a better response to peginterferon therapy than other genotypes, and genotyping should be considered if peginterferon treatment of hepatitis B is contemplated. Treatment

Generally, hepatitis B is treated if the patient is at risk for disease progression. This includes patients in the HBeAg-positive or HBeAg-negative chronic hepatitis B phases. Patients with compensated cirrhosis should be treated if the HBV DNA level is more than 2,000 IU/mL, and patients with decompensated cirrhosis should be treated if there is any detectable HBV level. Pretreatment liver biopsy is not necessary for all patients but can be helpful in patients who otherwise do not meet clear criteria for treatment. Liver biopsy also can be used to diagnose advanced fibrosis, which could mandate a change in management, such as evaluation for esophageal varices or surveillance for hepatocellular carcinoma. Hepatitis B can be treated with peginterferon or one of the oral agents (lamivudine, adefovir, entecavir, telbivudine, or tenofovir). Peginterferon has replaced standard interferon because of once-weekly dosing and perhaps better efficacy. One year of treatment generally is advised. Seroconversion may occur months or even years after completion of treatment. Predictors of a greater likelihood of response to peginterferon

include higher ALT level, lower HBV DNA level, shorter duration of disease, genotype A or B, and female sex. Patients treated with peginterferon may experience a flare of hepatitis (likely due to immune system activation) about 4 to 8 weeks after beginning treatment. Treatment should be continued despite this flare unless there is clinical or biochemical evidence of decompensation. Patients with Child-Pugh class B or C cirrhosis should not be treated with interferons because of the risk of precipitating decompensation with this flare. Adverse effects of peginterferon are common and are considered below (Hepatitis C). The oral agents are the drugs prescribed most frequently for the treatment of chronic hepatitis B. They are compared in Table 23.6. These drugs are well tolerated. The oral agents are useful particularly in patients with decompensated cirrhosis, because these drugs may improve liver function. The flare of hepatitis that may occur during interferon therapy is unusual with the oral agents. Treatment with lamivudine or telbivudine is complicated by resistant mutations at a rate of about 10% to 15% per year. Resistance to adefovir occurs at a rate of about 1% to 4% per year. Resistance to entecavir is uncommon unless the patient has previously developed resistance to lamivudine, although data for long-term treatment are needed. Clinical resistance to tenofovir has not yet been described. Because of their lower rates of resistance, tenofovir and entecavir are now considered the oral agents of choice. About 15% to 20% of HBeAg-positive patients treated with oral agents have seroconversion to anti-HBe positive after 1 to 2 years of therapy; treatment should be continued for at least 6 months after seroconversion. Patients without seroconversion of HBeAg need to continue treatment indefinitely. Seroconversion of HBsAg with the oral agents is unusual; therefore, most patients with HBeAg-negative chronic hepatitis will require prolonged or even indefinite therapy. The choice of therapeutic agent for hepatitis B depends on several factors. As noted above, only active hepatitis should be treated. Peginterferon is reasonable for patients without cirrhosis who have an ALT level greater than 200 U/L and who are able to tolerate the numerous adverse effects of the drug. Peginterferon is most effective against hepatitis B genotypes

Table 23.6 Oral Agents for Treatment of Hepatitis B Feature

Lamivudine

Adefovir

Entecavir

Telbivudine

Tenofovir

Charge/month, $US HBeAg seroconversion, % of patients Loss of HBV DNA, % of patients Resistance, % of patients

750 20 40 20 @ 1 year 70 @ 5 years

950 12 21 0 @ 1 year 29 @ 5 years

750 22 60 4 @ 1 year 21 @ 2 years

725 20 90 None

Durability of response, % of responding patients

50–80

90

800 21 67 Naive: 1 cm) found during surveillance to arise in a cirrhotic liver shows typical arterial enhancement and venous washout on triphasic CT or MRI. The major rationale for noninvasive diagnosis in early-stage hepatocellular carcinoma is to prevent tumor seeding and recurrence after potentially curative treatment, including liver transplant. Patients who present with newly discovered liver masses in the absence of cirrhosis should have a biopsy study to histologically confirm hepatocellular carcinoma because conditions such as lymphomas and metastases from other primary sites not infrequently masquerade as hepatocellular carcinoma in a noncirrhotic liver. Determination of the alpha fetoprotein level may obviate the need for biopsy if the level is markedly increased, but it is important to consider that malignancies at other primary sites, notably esophageal and gastric carcinomas, also can be associated with a high level of alpha fetoprotein. Management Liver Transplantation

Liver resection and transplantation offer the greatest chance of cure for patients with hepatocellular carcinoma. The selection of resection versus transplantation is based on a careful evaluation of the comorbid conditions of the patient, liver function,

24 Clinical Approach to Liver Mass Lesions

A

B

289

cirrhosis who have well-preserved liver function and little or no portal hypertension. For patients without cirrhosis, major liver resection can be performed with low mortality rates (3× upper limit of normal). He drinks two or three glasses of wine on weekends. He does not smoke. He has chronic low-grade abdominal pain, often exacerbated by rich foods. CT shows a normal-appearing pancreas. EUS shows lobularity and stranding of the parenchyma, ectatic side branches of the pancreatic duct, and small shadowing calculi in the main pancreatic duct. Which of the following suggests a definitive diagnosis of chronic pancreatitis? a. Chronic abdominal pain in a patient who had an episode of documented acute pancreatitis 6 months earlier b. Abnormal EUS findings in an alcoholic patient who does not have abdominal pain or pancreatitis c. Intraductal calculi in a patient with recurrent pancreatitis d. Chronic abdominal pain with persistent mildly increased (